You are on page 1of 131

SEARCH { CYBERWORLDFORU.BLOGSPOT.

COM } ON GOOGE

ALL STUDY MATERIALS IN FREE OF COST


HERE YOU WILL GET FOLLOWING MATERIALS :-
 HANDWRITTEN NOTES
 PRINTED NOTES { DETAILED , REVISION , SHORT }
 IMPORTANT QUESTIONS
 BOOKS PDF
 BOOKS SOLUTIONS
 TEST PAPERS
 CHAPTERWISH PREVIOUS YEAR PAPERS
 SAMPLE PAPERS
 CONCEPT MAPS
 HOT QUESTIONS
 NEWS AND UPDATES JOIN @iitwale in Telegram

MATERIALS ARE AVILABLE FOR FOLLOWING :-


 CLASS 9TH
 CLASS 10TH
 CLASS 11TH
 CLASS 12TH
 IIT-JEE
 NEET
 NTSE
 KVPY
 OTHER COMPETITIVE EXAMS
Number System

QUESTIONS

1. For P  N, 34P  24P is always divisible by ____


(a) 15 (b) 5
(c) 13 (d) Both (b) and (c)
(e) None of these
2. The greatest number of 5 digits exactly divisible by 15, 24 and 36 is ____
(a) 99620 (b) 99720
(c) 99968 (d) 99960
(e) None of these
3. The greatest number of 6 digits exactly divisible by all the numbers between 1 and 10 (both inclusive)
is _________
(a) 997920 (b) 999768
(c) 999660 (d) 999760
(e) None of these
4. The smallest three digit number which leaves remainders 8 and 12 when divided by 28 and 32
respectively, is _________
(a) 102 (b) 222 JOIN @iitwale in Telegram
(c) 202 (d) 204
(e) None of these

5. If P = n - 1+ n+1 where n is a positive integer then the value of P is


(a) a rational number
(b) not a rational number
(c) an integer
(d) a natural number
(e) None of these
6. The largest number that will divide 398, 606 and 474 leaving remainders 7, 11 and 15 respectively is
________
(a) 52 (b) 26
(c) 17 (d) 18
(e) None of these
7. Which one among the following statements is true?
(a) The remainder when the square of any number is divided by 4 is 1 or 0.
(b) There is no natural number for which 4 ends with digit zero.

(c) A positive integer n is prime, if no prime p less than or equal to n divides n.


(d) All the above
(e) None of these
8. The unit value of 6100 - 5100 is ______
(a) 0 (b) 1
(c) 2 (d) 3
(e) None of these
4n 4n
9. For any odd natural number n, ( 3) +( 2) is always divisible by ______

(a) 5 (b) 7
(c) 17 (d) 13
(e) None of these
10. If I is a positive integer then (I )2 will be in the form of ————
(a) 4m for some integer m
(b) 8m for some integer m
(c) 4m+1 for some integer m
(d) Both (a) and (c)
(e) None of these
11. Which among the following statements is not true?
(a) The square of any odd integer is of the form 4q + 1, for some integer q.
2
(b) For any odd integer p, p 2 - 1 is divisible by 8. (c) If p and q are both odd JOIN
positive integers/
@iitwale then p q 2 is even
inTelegram
and divisible by 4.
(d) For any natural number n, 12n cannot end with the digit 0 or 5.
(e) None of these
12. For any natural number n, (2n  1)2  1 is always divisible by ______
(a) 2 (b) 4
(c) 8 (d) All the above
(e) None of these
13. Which of the following statements is always true?
(a) The sum or difference of a rational and an irrational number is rational.
(b) Every irrational number is a surd.
(c) The product or quotient of a non-zero rational number and an irrational number is irrational.
(d) All the above
(e) None of these
14. The value of (27)3p - (13)3p ends in ______ (where p is a natural number)
(a) 0 (b) 4
(c) 6 (d) Either (b) or (c)
(e) None of these

3
15. If P  1  3  5  7.............21 and Q  2  4  6  8  10............22 then HCF of P and Q is ________

(a) 12375 (b) 14175


(c) 825 (d) 925
(e) None of these
16. In a seminar, the numbers of participants in science, English and Mathematics are 144, 180 and 192
respectively. Find the minimum number of rooms required if in each room the same number of
participants are to be seated and all of them being in the same subject.
(a) 38 (b) 40
(c) 43 (d) 45
(e) None of these
17. Without actually performing the Song division, choose which among the following rational numbers
will not have a terminating decimal expansion.
123 351
(a) (b)
16 2  58  718
7

32 833
(c) (d)
28  59 49  27
(e) None of these
18. The largest number that divides 588, 1999 and 1650 leaving 3, 10 JOIN
and 12 respectively
@iitwale isin______
Telegram
(a) 117 (b) 109
(c) 27 (d) 43
(e) None of these
12879
19. The decimal expansion of the rational number will terminate after:
1250
(a) One decimal places
(b) Two decimal places
(c) Three decimal places
(d) Four decimal places
(e) None of these
20. Find the greatest prime factor in 527527.
(a) 17 (b) 11
(c) 13 (d) 31
(e) None of these
21. If p is a single digit natural number and the unit digits of p4 and p are same, then how many
possibilities p can assume?
(a) 2 (b) 3
(c) 4 (d) 5
(e) None of these

4
22. The sum of LCM and HCF of two numbers is 29610. If their LCM is 140 times v the HCF of the
numbers then which among the following can be one of the numbers?
(a) 330 (b) 1470
(c) 525 (d) 462
(e) None of these
3m 3m
23. The value of  22   28  ends in ______ {M  N} .

(a) 8 (b) 2
(c) 6 (d) 0
(e) None of these
24. If LCM and HCF of two numbers are 324 and 18 respectively, then how many such pairs of numbers
are possible?
(a) 0 (b) 1
(c) 2 (d) 3
(e) None of these

25. If p = 11+ 5, q = 14 + 2 and r = 13 + 3 then which one of the following holds true?

(a) p > q > r (b) p < q < r


(c) p > r > q (d) p < r < q JOIN @iitwale in Telegram
(e) None of these
26. The number of ways, in which 360 can be resolved in two factors, is ______
(a) 24 (b) 18
(c) 12 (d) 15
(e) None of these

27. If u  16 7  16 5, v  7  5, w  8 7  8 5, x  16 7  16 5, and y  4 7  4 5, then which one of the following


is a rational number?
(a) uvxy (b) uvwxy
(c) uxwy (d) vwxy
(e) None of these
28. A mason has to fit two bathrooms with square marble tiles of the largest possible size. The dimensions
of each such bathroom are 12 fts and 10 fts. If the size of the tiles in inches has to be taken then
number of such tiles required is ____
(a) 15 (b) 30
(c) 60 (d) 80
(e) None of these
29. If HCF of 374 and 255 is H and H = 255m + 374n then the value of m - n is equal to _____
(a) 3 (b) 4
(c) 5 (d) 1
(e) None of these

5
30. Choose which one among the following statement is incorrect?
(a) HCF of two co-primes a and b is 1.
(b) LCM of two co-primes m and n is mn.
(c) By using Euclid's division lemma for two numbers 155 and 345, we get 345 = 155 × 2 + 35.
(d) The remainder, when the square of any prime number greater than 3 is divided by 6, is 1.
(e) None of these
31. If LCM and HCF of two numbers are 3003 and 21 respectively, then how many such numbers are
possible?
(a) 0 (b) 1
(c) 2 (d) 3
(e) None of these
32. The largest number which divides 1288 and 2915 and leaves the remainders 1 and 8 respectively, is
H and it satisfies the expression, H = 45m + 288n. Find the value of m + n.
(a) 11 (b) 15
(c) 13 (d) 10
(e) None of these
33. The smallest number, which when increased by 19 is exactly divisible by both 2079 and 1404, is
_______ JOIN @iitwale in Telegram
(a) 6200 (b) 625
(c) 6218 (d) 3208
(e) None of these

7+ 4 3
34. equals to ________
2

2 2 6
(a) 2 6 (b)
2

2 6 32
(c) (d)
2 4
(e) None of these
35. If HCF of the numbers (3600, x) = 20 then how many values are possible for x? (where it is assumed
that x is a product of a power of 2 and a power of 5 only)
(a) One (b) Two
(c) Three (d) Four
(e) None of these
36. The number of ways, in which 576 can be resolved into two factors, is ________
(a) 8 (b) 9
(c) 10 (d) 11
(e) None of these

6
37. Four runners P, Q, R and S start running around a circular track simultaneously. If they complete one
round in 16, 12, 24, 18 minutes respectively, after how much time they will meet next?
(a) 2 hours 20 minutes (b) 2 hours
(c) 3 hours 18 minutes (d) 2 hours 24 minutes
(e) None of these
38. If LCM and HCF of two numbers are equal, then the numbers will be _________
(a) Composite (b) Prime
(c) Equal (d) Co-prime
(e) None of these
39. If the product of two numbers is 149058 and HCF of these numbers is 21 then how many pairs of
these numbers are possible?
(a) 1 (b) 2
(c) 3 (d) 4
(e) None of these
40. If A  14  (1  2  3  4  5  ................10  14) and B  19  (1  2  3  4  5 ................10  19) then which one

of the following is/are correct?


(i) B  A is a prime number.
(ii) B  A is a composite number. JOIN @iitwale in Telegram
(iii) A is a composite number.
(iv) B is a prime number.
(a) Both (i) and (ii)
(b) Both (ii) and (iii)
(c) Both (iii) and (iv)
(d) All (i), (ii), (iii) and (iv)
(e) None of these

7
ANSWER - KEY

1. (d) 2. (b) 3. (a) 4. (d) 5. (b)

6. (c) 7. (d) 8. (b) 9. (d) 10. (d)

11. (c) 12. (d) 13. (c) 14. (d) 15. (b)

16. (c) 17. (b) 18. (a) 19. (d) 20. (d)

21. (b) 22. (b) 23. (d) 24. (c) 25. (c)

26. (c) 27. (b) 28. (c) 29. (c) 30. (e)

31. (c) 32. (a) 33. (c) 34. (b) 35. (b)

36. (d) 37. (d) 38. (c) 39. (b) 40. (b)
JOIN @iitwale in Telegram

8
SEARCH { CYBERWORLDFORU.BLOGSPOT.COM } ON GOOGE

ALL STUDY MATERIALS IN FREE OF COST


HERE YOU WILL GET FOLLOWING MATERIALS :-
 HANDWRITTEN NOTES
 PRINTED NOTES { DETAILED , REVISION , SHORT }
 IMPORTANT QUESTIONS
 BOOKS PDF
 BOOKS SOLUTIONS
 TEST PAPERS
 CHAPTERWISH PREVIOUS YEAR PAPERS
 SAMPLE PAPERS
 CONCEPT MAPS
 HOT QUESTIONS
 NEWS AND UPDATES JOIN @iitwale in Telegram

MATERIALS ARE AVILABLE FOR FOLLOWING :-


 CLASS 9TH
 CLASS 10TH
 CLASS 11TH
 CLASS 12TH
 IIT-JEE
 NEET
 NTSE
 KVPY
 OTHER COMPETITIVE EXAMS
Polynomials

QUESTIONS

1. If the zeroes of the quadratic polynomial x2   a  3 x  b are 3 and - 4, then

(a) a  2, b  6 (b) a   2, b  12

(c) a  3, b  4 (d) a  4, b   3

(e) None of these


2. See the graph given below:

(i) (ii)

(iii)
JOIN @iitwale in Telegram
Based on these graphs, identify the correct statement among the following:
(a) In case ii, the quadratic polynomial a x 2  bx  c has two distinct zeroes.

(b) In case i, the quadratic polynomial ax 2  bx  c has two equal zeroes.


(c) In case iii, the quadratic polynomial has no zero.
(d) All the above
(e) None of these
3. Which one among the following statements is incorrect?
(a) Graph of a linear polynomial is a straight line whereas the graph of a quadratic polynomial has one of the two

shapes of parabola either open upwards or open downwards .


(b) The shape of the parabola depends on the value of 'a' of the quadratic polynomial ax 2  bx  c .
(c) The zeroes of a quadratic polynomial ax 2  bx  c , a  0 are y coordinates of the points where the parabola
y  ax 2  bx  c intersects the y-axis.
(d) A real number m is a zero of the polynomial p(x) if p (m) = 0
(e) None of these
4. A polynomial of degree n has ________
(a) two zeroes (b) n zeroes
(c) atleast n zeroes (d) atmost n zeroes
(e) None of these
5. If one zero of the quadratic polynomial x 2  5 x  k is 3 then second zero of this polynomial is ______
(a) 5 (b)  3
(c)  5 (d)  8
(e) None of these
6. If the zeroes of a quadratic polynomial ax 2  bx  c are both negative, then
(a) a is positive., b and c are negative
(b) a is negative/ b and c are positive
(c) a and c are negative, b is positive
(d) a, b and c all have the same sign
(e) None of these

7. If (3 + 3 ) is one of the zeroes of the quadratic polynomial x 2 + mx + 6 then find the second zero.

(a)  3 (b) 3  3

(c) 3  3 (d) 3
(e) None of these

4+ 2
8. For a quadratic polynomial 2 x 2 - 8x + b, sum of its roots is 4 and one of the roots is , then
2
the value of b is______
JOIN @iitwale in Telegram
(a) 3 (b) 6
(c) 7 (d) 8
(e) None of these
9. If the zeroes of the quadratic polynomial p(x)  abx 2  (b2  ac)x  bc are  &  , then

b 2  c2 a b
(a)   and   (b)   and  
a b b c
b c a c
(c)   and   (d)   and  
a b b b
(e) None of these
 
10. If  and  are zeroes of the quadratic polynomial p  x   ax 2  bx  c, then the value of  is
 
_________ .

b 2  ac b 2  ac
(a) (b)
ac ac
b 2  2ac c 2  2ac
(c) (d)
ac ac
(e) None of these

3
3 5
11. If the zeroes of the quadratic polynomial ax 2  x  b are and , then
2 3
(a) a = 15, b = 6 (b) a = 6, b = 15
(c) a = 12, b = 4 (d) a = 4, b = 12
(e) None of these
12. If p(x)  25 x 2  15 x  a where  and  are the zeroes of the polynomial, also if it is given that

63
3  3  , then
125
1 4
(a) a = 5 (b) roots are and
5 5
1 2
(c) a = 3 (d) roots are and
5 5
(e) None of these
13. If two zeroes of the cubic polynomial px 2  qx 2  rx  s are 0, then the third zero is _________

p p
(a) (b)
q q

p
(c)  (d) 0
q
JOIN @iitwale in Telegram
(e) None of these
14. If one of the zeroes of a cubic polynomial of the form x 3  ax 2  bx  c is the negative of the other, then
(a) a is of negative sign and b and c are of positive sign
(b) b is of negative sign and a and c are of positive sign
(c) a and c are of opposite signs and b is of negative sign
(d) a and b are of opposite signs and c is of positive sign
(e) None of these
15. If all the zeroes of the cubic polynomial x 3  c x 2  d x  b are equal, then

(a) cd = 9 b (b) bd = 8 b
(c) cd = 6 b (d) bd = 8 b
(e) None of these
1 1
16. If p and q are the of the polynomial bx 2  cx  a, value of +
p3 q 3

3abc  c 3 3abc  c 3
(a) (b)
ab 2 ab 2
3abc  c 3 3abc  c 3
(c) (d)
a 2b a 2b
(e) None of these

4
17. If the zeroes of the polynomial 6 x 2  7 3 x  15  0 are  &  , then

 3 5 3
(a)   & 
2 3
(b)    3 &   5 3

3 5 3
(c)   & 
2 3
(d)   5 3 &    3

(e) None of these


18. If  &  are the zeroes of the quadratic polynomial 3 x 2  11x  6, then find the polynomial whose

zeroes are (2   ) and (  2 )

 270 
(a) k  x 2  5 x  , k is any non-zero real number
 9 

 260 
(b) k  x 2  11x  , k is any non-zero real number
 9 

(c) k (3 x 2  3 x  26), k is any non-zero real number

(d) k (2x 2  5 x  27), k is any non-zero real number

(e) None of these


JOIN @iitwale in Telegram
19. If  ,  &  are the roots of the equation x 3  4 x 2  53x  168 then the relation between their roots is

_______
(a) 3    2  (b) 3  4   4 

(c) 3    4  (d)   2  

(e) None of these


20. What must be subtracted from 6 x 4  16 x 3  15 x 2  8 x  9, so that it is exactly divisible by 3x 2  5 x  2 ?
(a) 19 x  15 (b) 19 x  16
(b) 13x  19 (d) 19 x  15
(e) None of these
21. If p(x)  x 3  10 x 2  31x  30 and q(x)  x 3  12x 2  41x  42 , then find the LCM of the polynomials p(x)
and q(x).
(a) x 4  17x 3  101x 2  247x  210

(b) x 3  36x 2  90x  105


(c) x 4  18x 3  95x 2  234x  119
(d) x 3  18x 2  108x  114
(e) None of these

5
1 1
22. What should be added to 2
to get 2
x - 12x +32 x - 11x +30

2 x 2  25 x  96 2 x 2  25 x  66
(a) (b)
(x  6)(x  5)(x  4)(x  8) (x  6)(x  5)(x  4)(x  8)

2 x 2  25 x  66 2
(c) (d)
(x  6)(x  5)(x  4)(x  8) (x  6)(x  5)(x  4)(x  8)
(e) None of these
23. If p(x)  x 2  x  1 and q(x)  x 3  x  1, then the HCF of p(a) - p(b) and q(a)

(a) a  b  1 (b) a – b  1
(c) a  b (d) a  b
(e) None of these
24. If (x 2  x  1) is a factor of x 4  9 x 3  qx 2  8 x  5 then find the values of p and q.

(a) p   3, q  4 (b) p  4, q   3

(c) p  2, q   4 (d) p   4, q  2

(e) None of these


3 5 a
25. If the zeroes of the algebraic expression 3ax 2  x(3b  5a)  5b are and , then find the value of .
7 3 b
JOIN @iitwale in Telegram
1 4
(a) (b)
3 5
7
(c) (d) 3
3
(e) None of these
26. If degree of both p(x) and [p(x) + q(x)] is 15 then degree of q(x) can be
(a) 12 (b) 10
(c) 15 (d) any one of the above
(e) None of these
27. If the LCM of p(x) and q(x) is a9  b9 then their HCF can be

(a) (a - b) 
(b) a 2  b2  ab 
(c) a6  b6  a3 b3 (d) All the above
(e) None of these
a 1 a 1
28. If m  and n  , then m2  n2  3mn is equal to
a 1 a 1

a 4  18a 2  1 a 4  9a 2  3
(a) (b)
a 4  2a 2  1 a 4  2a 2  1
a 4  9a 2  3 a 4  16a 2  1
(c) (d)
a 4  2a 2  1 a 4  2a 2  1
(e) None of these

6
p2 (q - r)2 q2 - (p - r)2 r 2 - (p - q)2
29. Solve + +
(p+ r)2 - q2 (p  q)2 - r 2 (q+ r)2 - p2

1
(a) (b) p + q + r
pqr
(c) 0 (d) 1
(e) None of these
30. Find the value of a - b so that 8 x 4  14 x 3  ax 2  bx  2 is exactly divisible by 4 x 2  3x  2 .
(a) 4 (b) 6
(c) 9 (d)  3
(e) None of these

31. If two zeroes of the polynomial f (x)  x 4  2x 3  18 x 2  6 x  45 are  3 and 3 , then find the sum of

other two zeroes.


(a) 0 (b)  1
(c)  2 (d) 1
(e) None of these
32. If the zeroes of the polynomial x 3  15 x 2  66 x  80 are  ,  &  and it is also given that 2    

then
JOIN @iitwale in Telegram
(a)   4 (b)   3

(c)   7 (d)   2

(e) None of these


33. If  &  are the zeroes of the polynomial x 2  6 x  k such that 2    11 then k is equal to

(a) 18 (b)  23
(c) 391 (d)  391
(e) None of these
34. If p and q are the zeroes of the quadratic polynomial f (x)  cx 2  ax  b then the value of p4  q4 is
______

(a 2  2bc)2  b 2c 2 (a 2  2bc)2  2b 2c 2
(a) (b)
c4 c4
(b 2  2ac)2  a 2c 2 (b 2  2ac)2  2a 2 c 2
(c) (d)
c4 c4
(e) None of these
35. If on dividing the polynomial f(x)= x3 - 4x2 +7x - 9 by a polynomial g(x), the quotient q(x) and the

remainder r(x) are (x  3) and (2x  3) respectively, the polynomial g(x) is ____

(a) x 2  x  1 (b) x 2  x  2
(c) 2x 2  x  1 (d) 2x 2  x  2
(e) None of these

7
36. If the zeroes of the polynomial f (x)  ax 3  3bx 2  3cx  d are in A.P. then 2b3  a2d is equal to _______

(a) a 2bc (b) 3abc


2
(c) 2b ac (d) abc
(e) None of these
37. If f(x) = 3x4 + 6x3 - 2x2 - l0x - 5 and two of its zeroes are - 1, - 1, then the other two zeroes are _______

3 3 2 2
(a) , (b) ,
5 5 5 5

5 5 5 5
(c) , (d) ,
3 3 4 4
(e) None of these
1 1 1
38. If  ,  ,  are the zeroes of the polynomial p(x)  x 3  ax 2  bx  c, then    __________
  

a b
(a) (b)
b c
a c
(c) (d)
c d
(e) None of these
39. The graph of a polynomial f(x) is shown below: JOIN @iitwale in Telegram

The number of real zeroes of the polynomial f(x) is _________


(a) 1 (b) 2
(c) 3 (d) 4
(e) None of these
40. The graph of a polynomial p(x)  ax 2  bx  c is shown below:

Based on the above graph which one is correct?

8
(a) a > 0, b > 0, c > 0 (b) a < 0, b < 0, c > 0
(c) a < 0, b < 0, c < 0 (d) a > 0, b < 0, c > 0
(e) None of these

ANSWER – KEY
JOIN @iitwale in Telegram
1. (b) 2. (c) 3. (c) 4. (d) 5. (d)

6. (d) 7. (b) 8. (c) 9. (c) 10. (b)

11. (b) 12. (b) 13. (c) 14. (c) 15. (a)

16. (e) 17. (c) 18. (b) 19. (c) 20. (a)

21. (a) 22. (c) 23. (c) 24. (b) 25. (b)

26. (d) 27. (d) 28. (a) 29. (d) 30. (c)

31. (c) 32. (d) 33. (c) 34. (b) 35. (b)

36. (b) 37. (c) 38. (c) 39. (d) 40. (d)

9
Surface Area and Volume
1. A cuboidal metal of dimensions 9. From a circle of radius 15 cm, a sector
44 cm  30 cm 15 cm was melted and cast central angle 216 is cut and its bounding
into a cylinder of height 28 cm. Find its radii are joined without overlap so as to
radius. form a cone. Find its volume.
(a) 20cm (b) 15cm (a) 1081.3cm3 (b) 1071.3cm3
(c) 10cm (d) 18cm (c) 1018.3cm3 (d) 1061.9cm3

2. The length of the longest rod which can, be


10. A cylindrical vessel of diameter 9 cm has
kept inside a rectangular box is 27 cm. If the
some water in it. A cylindrical iron piece of
length and the breadth of the box are 23 cm
diameter 6 cm and height 4.5 cm is dropped
and 10 cm respectively, find its height.
in it. After it was completely immersed, what
(a) 8cm (b) 10cm
is the rise in the level of water?
(c) 12cm (d) 14cm
(a) 0.8 cm (b) 0.5 cm
(c) 0.1 cm (d) 0.3 cm
3. The volume of a cube is numerically equal
to its surface area. What is the length of its
11. The diameter of a copper sphere is 6 cm. It
side?
is beaten and drawn into a wire of diameter
(a) 6 units (b) 8 units
0.2 cm. What is the length of the wire?
(c) 9 units (d) 10 units
(a) 36cm (b) 360cm
(c) 3600 cm (d) 306 cm
4. The total surface area of a cylinder is 220 sq
cm with height 6.5 cm. Find its volume. JOIN @iitwale in Telegram
12. A cylindrical vessel of diameter 4 cm is
(a) 25.025 cm3 (b) 2.5025 cm3 partly filled with water. 300 lead balls are
(c) 2502.5 cm 3
(d) 250.25 cm3 dropped in it. The raise in water level is 0.8
cm. Find the diameter of each ball.
5. The lateral surface area of a cylinder is 176 (a) 0.8 cm (b) 0.4 cm
cm2 and its base area is 38.5cm2 .What is its (c) 0.2 cm (d) 0.6 cm
volume?
(a) 830 cm3 (b) 380 cm3 13. Three solid glass balls of radii 'r' cm, 6 cm
(c) 308 cm3 (d) 803 cm3 and 8 cm are melted into a solid sphere of
radius 9 cm. Find the value of 'r'.
6. Identify the ratio of lateral surface areas of 1 1
(a) cm (b) cm
two cylinders with equal heights. 4 3
(a) H: h (b) R: r 1
(c) cm (d) 1 cm
(c) 1: 2 (d) 2 : 3 2
7. A cone and a hemisphere have equal bases
and equal volumes. Identify the ratio of their 14. A hemispherical bowl is made of steel of
heights. 0.25 cm thickness. The inner radius of the
(a) 1 : 3 (b) 1 : 2 bowl is 5 cm. What is the volume of steel
(c) 2 : 1 (d) 3 : 1 used?
(a) 42.15 cm3
8. The area of the base of a cone is 616 sq cm. (b) 41.52 cm3
Its height is 48 cm. Find its total surface (c) 41.25cm3
area.
(a) 2816cm2 (b) 2861 cm2 (d) 43.21cm3

(c) 2618cm2 (d) 2681 cm2


15. A hollow sphere of internal and external 21. A cylindrical vessel contains 49.896 litres of
diameters 4 cm and 8 cm respectively is liquid. The cost of painting its curved
melted into a cone of base diameter 8 cm. surface area at 2 paise/sq cm is ` 95.04.
Find the height of the cone. Find its total surface area.
(a) 14 cm (b) 12 cm (a) 5724 cm2 (b) 7524 cm2
(c) 16 cm (d) 18 cm (c) 5742 cm 2
(d) 7254 cm2

16. A circus tent is in the form of a cone over a 22. The ratio of base radius and height of a
cylinder. The diameter of the base is 9 m, cone is 3 :4. If the cost of smoothening the
the height of cylindrical part is 4.8 m and curved surface area at 5 paise/sq cm is `
the total height of the tent is 10.8m. What 115.50, find the volume of cone.
area of canvas is required for the tent? (a) 12963cm3 (b) 12693cm3
(a) 24.184 sq. m (b) 2418.4 sq. m
(c) 12936cm3 (d) 12639cm3
(c) 241.84sq.rn (d) 24.164 sq. m

17. If the radii of the circular ends of a conical 23. A vessel is conical in shape. If its volume is
bucket of height 45 cm are 28 cm and 7 cm, 33.264 litres and height is 72 cm, find the
what is the capacity of the bucket? cost of repairing its curved surface area at?
(a) 48150cm3 (b) 48510cm3 12/sq m.
(a) ` 5.94 (b) ` 6.94
(c) 48105cm3 (d) 48015cm3
(c) ` 7.95 (d) ` 9.65

18. The height of a cone is 30 cm. A small cone 24. The cost of painting the curved surface area
is cut off at the top by a plane parallel to the of a cone at 5 ps / cm2 is ` 35.20, find the
1 JOIN @iitwale in Telegram
base. If its volume is . the volume of volume of the cone if its slant height is 25
27 cm.
cone, what is the height at which the section (a) 1223cm3 (b) 1232cm3
is made?
(a) 10 cm (b) 15 cm (c) 1323cm3 (d) 1332cm3
(c) 20 cm (d) 19 cm
25. A piece of metal pipe is 77 cm long with
19. A top is of the shape of a cone over a inside diameter of the cross section as 4 cm.
hemisphere. The radius of the hemisphere is If the outer diameter is 4.5 cm and the metal
3.5 cm. The total height of the top is 15.5 weighs 8 gm/cu cm, find the weight of pipe.
cm. Find the total area of the top. (a) 2.057 kg (b) 20.57 kg
(a) 214.5cm2 (b) 21.45cm2 (c) 205.7 kg (d) 2.097 kg
(c) 215.4cm2 (d) 21.43cm2
26. A rectangular solid metallic cuboid
2 9 cm  8 cm  2 cm is melted and recast into
20. The base radius of a cylinder is 1 . times its
3 solid cubes each of side 2 cm. How many
height. The cost of painting its curved such solid cubes can be made?
surface area at 2 paise / cm2 is ` 92.40. Find (a) 20 (b) 18
the volume of the liquid. (c) 12 (d) 15
(a) 80850 cm3
(b) 80580 cm3 27. The side of a solid metallic cube is 50 cm. It
is melted and recast into 8000 similar solid
(c) 80508 cm3
cubical dice. Find the side of each die.
(d) 800508 cm3 (a) 2.5 cm (b) 6 cm
(c) 1.9cm (d) 5.2cm

3
28. Water flows at the rate of 5 km per hour 34. 56 circular plates, each of radius 5 cm and
through a pipe of diameter 14 cm into a thickness 0.25 cm, are placed one above the
rectangular tank which is 25 m long and 22 other to form a solid right circular cylinder.
m wide. Find the time in which the level of Find the volume of the cylinder so formed.
water in the tank rises by 21 cm. [Take (a) 900 cm3 (b) 885 cm3
22
  .] (c) 1100 cm 3
(d) 1250cm3
7
(a) 2 hours 35. A semicircular piece of paper of radius 14
(b) 5 hours cm is rolled to form a cone of the largest
(c) 3 hours possible size. Find the capacity of the cone.
(d) 1.5 hours (a) 721.5cm3 (b) 645.10cm3
(c) 449.64 cm3 (d) 622.37 cm3
29. Two right circular cones X and Y are made
such that X has a radius three times the
36. In a solid cylinder of height 10 cm and
radius of Y and Y has a volume half the
diameter 8 cm, two equal conical cavities
volume of X. What is the ratio of heights of
have been made at both its ends. If the
X and Y?
diameter of the cavity is 6 cm and height 4
(a) 5 : 8 (b) 4 : 7
cm, find the volume of the remaining solid
(c) 2 : 9 (d) 3 : 5
in terms of  .
(a) 95 cm3 (b) 136 cm3
30. Rain water from a roof 22 m  20 m drains
into a cylindrical vessel with diameter of (c) 216 cm3 (d) 192  cm3
base 2 m and height 3.5 m. If the vessel is
just full, find the amount of rainfall in cm. 37. A solid sphere
JOINand@iitwale
a solid hemisphere have
in Telegram
(a) 4.5 cm (b) 2.5 cm the same total surface area. Find the ratio of
(c) 3 cm (d) 1 cm their volumes.
(a) 3 :1 (b) 3 3 : 5
31. The diameter of a solid metallic sphere is 16 (c) 3 3 : 4 (d) 1: 3
cm. The sphere is melted and recast into 8
similar solid spherical balls. What is the
38. A sphere, of diameter 12 cm, is dropped
radius of the each of the balls?
into a right circular cylindrical vessel, partly
(a) 2 cm (b) 3 cm
filled with water. If the sphere is completely
(c) 4 cm (d) 6 cm
immersed in water, the water level in the
5
32. A hemispherical bowl of internal diameter cylindrical vessel rises by 3 cm. Find the
36 cm is full of liquid. The liquid is to be 9
filled into cylindrical shaped bottles each of diameter of the cylindrical vessel.
radius 3 cm and height 9 cm. How many (a) 16cm (b) 18cm
bottles are required to empty the bowl? (c) 12cm (d) 13cm
(a) 24 (b) 35
(c) 12 (d) 48 39. The largest right circular cone is carved out
of a cube whose edge is of length 'p' units.
33. If the radius of the base of a right circular Find the volume of the cone.
cylinder is halved, keeping the height same,  p3
(a) cu.units
find the ratio of the volume of the reduced 12
cylinder to that of the original cylinder.  p3
(b) cu.units
(a) 1 : 4 (b) 2 : 3 4
(c) 1 : 2 (d) 5 : 7 (c)  p3cu.units
 p3
(d) cu.units
5

4
40. A right-angled triangle ABC, where B  90o 44. The radii of circular ends of a solid frustum
, is rotated about BC. If BC = 16 cm and of a cone are 33 cm and 27 cm and its slant
AC = 20 cm, find the volume of the right height is 10 cm. Find its total surface area.
circular cone traced out by the triangle. (a) 1564.15cm2 (b) 7599.43cm2
(c) 6542.1 cm2 (d) 8265.14cm2

45. The ratio of the radii of two spheres is 3 : 2.


Find the ratio of their volumes.
(a) 25 : 4 (b) 5 : 16
(c) 9 :14 (d) 27 : 8

(a) 2413cm3 (b) 2311 cm3 46. A cylindrical can whose base is horizontal
(c) 1254cm3 (d) 1725cm3 and of internal radius 3.5 cm contains
sufficient water so that when a solid sphere
is placed in the can, water just covers the
41. A bucket of height 16 cm made up of metal
sphere. Given that the sphere just fits in the
sheet is in the form of frustum of a right
can, find the depth of water in the can
circular cone with radii of its lower and
before the sphere was put into it.
upper ends as 3 cm and 15 cm respectively.
(a) 2.3 cm (b) 5.1 cm
What is the slant height of the bucket?
(c) 1.5 cm (d) 3.2 cm
(a) 15 cm (b) 20 cm
(c) 5 cm (d) 12 cm
47. The internal and external radii of a
hemispherical metallic vessel are 7 cm and
42. A right-angled triangular plate of smaller
sides measuring 5 cm and 12 cm is rotated 10.5 cm JOIN @iitwale
respectively. If 1 cm3in
of Telegram
the metal
about the longest side to trace a double weighs 10 g, find the weight of the vessel.
cone as shown in the figure. Find the ratio (a) 25.15 kg (b) 32.1 kg
of the curved surface areas of the two cones. (c) 17.07 kg (d) 16.27 kg

48. A tent is made in the form of a frustrum A of


a right circular cone surmounted by another
right circular cone B. The diameter of the
ends of the frustrum A are 8 m and 4 m, its
(a) 6 : 1 (b) 5 : 12 height is 3 m and the height of the cone B is
(c) 3 : 14 (d) 6 : 5 2 m. What is the area of the canvas required
for the tent?
43. The largest possible cube is made from a (a) 86.75 m2 (b) 85.77 m2
wooden sphere of radius 6 3 cm cm. Find (c) 86.77 m 2
(d) 87.67 m2
the surface area of the cube.
49. What is the maximum distance between any
(a) 864cm2 (b) 542cm2
two points of a cube of side T units?
(c) 735cm2 (d) 625cm2
(a) ( 2  1)l units (b) 2 l units
(c) 3 l units (d) 2l units

5
Answer - Keys

1. B 2. B 3. A 4. D 5. C 6. B

7. C 8. A 9. C 10. B 11. C 12. B

13. D 14. C 15. A 16. C 17. B 18. C

19. A 20. A 21. B 22. C 23. A 24. B

25. A 26. B 27. A 28. D JOIN


29. @iitwale
C in
30.Telegram
B

31. C 32. D 33. A 34. C 35. D 36. B

37. C 38. B 39. A 40. A 41. B 42. B

43. A 44. B 45. D 46. A 47. C 48. B

49. C

6
Solution
1. (b) Volume of cuboid = Volume of cylinder 7
r 
 lbh   r h2
2
44  30 15  7  V   r 2h  250.25 cm3
 r2 
22  28 Hence, volume of the cylinder  250.25 cm3
 r  15 cm
5. (c) Base area of a cylinder  38.5 cm2
Hence radius of the cylinder is equal to 15
7
cm.   r 2  38.5 cm2  r  cm
2
Lateral surface area  176 cm2
2. (b) Given that the length of the longest rod
 2 rh  176 cm2  h  8 cm
= 27 cm
2
Length of the box, l  23 cm 22  7 
 Volume   r h 
2
  8
Breadth of the box, b = 10 cm 7 2
Let 'h' be the height of the box  308 cm3
Length of the longest rod = Length of the Hence, volume of the cylinder  308 cm3
diagonal  l 2  b2  h2
6. (b) Not available
 27  (23)2  (10)2  h2
Squaring on both sides
7. (c) Not available
 (27)2  (23)2  (10)2  h2 JOIN @iitwale in Telegram
 h  100  10 cm 8. (a) Not available
Hence, the height of the box = 10 cm
9. (c) Not available
3. (a) Volume of a cube  l 3cu. units
10. (b) Not available
Surface area of a cube  6 l 2 sq. units,
where ' l ' is the length of each side of a 11. (c) Diameter of sphere = 6 cm
cube. Diameter of wire =0.2 cm
Given that the volume of a cube is Volume of cylindrical wire = Volume of
numerically equal to its surface area sphere
 l 3cu.units  6 l 2 sq.units 4
  r 2h   R3
l3 3
 6  l  6 units
l2 4 22 7 1
 h    33  
The length of each side of the cube = 6 3 7 22 0.01
units.  3600 cm
 Length of wire is 3600 cm.
4. (d) TSA  2 r (r  h)
22  13  12. (b) Volume of 300 spherical balls
220  2  rr   = Volume of rise in water level
7  2
3
7 4 22  d  22 4
 r  10,  300        2  2 
2 3 7 2 7 5
Since radius cannot be — ve,

7
2   r 2h  49896 cm3
d  0.4 cm
5 9504 2
C.S.A.  cm
2
13. (d) Given that the radii of three solid glass  2 rh  4752
balls is 'r' cm, 6 cm and 8 cm, sum of the  r 2 h 49896
volumes of the three glass balls  
2 rh 4752
4 4 4
  r 3   (6)3   (8)3  r  21and h  36
3 3 3
T.S.A.  2 r (h  r )  7524 cm2
4
  (r 3  63  83 )cm3
3
22. (c) Not available
The volume of the solid sphere of radius
9cm
23. (a) Not available
4
   (9)3  243  4
3
24. (b) Not available
4
 243  4   (r 3  728)
3
25. (a) Not available
 r 1

26. (b) Not available


14. (c) Not available

27. (a) The side of the given metallic cube = 50


15. (a) Not available JOIN @iitwale in Telegram
cm
Its volume  (50)3 cm3  125000 cm3
16. (c) Not available
Let ' x ' cm be the side of a cubical die. Then
17. (b) Not available volume of 8000 dice
 8000  ( x)3 cm3
18. (c) Not available According to the problem, the volume of
8000 cubical dice = The volume of the
19. (a) Height of cone  15.5  3.5  12 cm given solid metallic cube
l  h2  r 2   8000  ( x)3  125000
5
 (3.5)2  (12)2  12.5cm  x  2.5
2
C.S.A.   rl  137.5 sq cm Hence, the side of the cubical die is 2.5 cm.
C.S.A. of hemisphere  2 r 2  77 cm2
Total area of the top = 214.5 sq cm. 28. (d) Not available

22 5h 220h2 29. (c) Not available


20. (a) C.S.A.  2   h 
7 3 21
9240 220h 2 30. (b) Not available
 
2 21
 V   r h  80850 cm3
2 31. (c) Not available

32. (d) Not available


21. (b) V  49.896 10000 cm3

8
33. (a) Not available
45. (d) Not available
34. (c) Not available
46. (a) Let the initial depth of water in the can
35. (d) Not available be x cm

36. (b) Not available

37. (c) Not available

38. (b) Not available


Then, volume of initial water column
39. (a)   r 2h   (3.5)2  cm3
Volume of the sphere
4 4
  r 3   (3.5)2 x cm3
3 3
Now, volume of initial water column +
1
Volume of the cone   r 2 h Volume of the sphere = Volume of the
3
cylinder up to the height of water level
 p3
2
1  p
      p  cu.units 4
   (3.5)2  x cm3   (3.5)3 cm3
3 2 12 JOIN @iitwale
3 in Telegram
2
7
40. (a) Given, h = 16 cm and l  20 cm.     7 cm3
2
4 14
 x   3.5  7  x   7
3 3
14 7
 x 7 
3 3
 r  l 2  h2  202  162 cm
 The depth of water in the can before the
 12cm sphere was put into it =2.3 cm
1
Volume   r 2 h
3 47. (c) Not available
1 22
  122 16 cm3
3 7 48. (b) Not available
16896 3
 cm  2413 cm3
7 49. (c) Not available

41. (b) Not available

42. (b) Not available

43. (a) Not available

44. (b) Not available

9
SEARCH { CYBERWORLDFORU.BLOGSPOT.COM } ON GOOGE

ALL STUDY MATERIALS IN FREE OF COST


HERE YOU WILL GET FOLLOWING MATERIALS :-
 HANDWRITTEN NOTES
 PRINTED NOTES { DETAILED , REVISION , SHORT }
 IMPORTANT QUESTIONS
 BOOKS PDF
 BOOKS SOLUTIONS
 TEST PAPERS
 CHAPTERWISH PREVIOUS YEAR PAPERS
 SAMPLE PAPERS
 CONCEPT MAPS
 HOT QUESTIONS
 NEWS AND UPDATES JOIN @iitwale in Telegram

MATERIALS ARE AVILABLE FOR FOLLOWING :-


 CLASS 9TH
 CLASS 10TH
 CLASS 11TH
 CLASS 12TH
 IIT-JEE
 NEET
 NTSE
 KVPY
 OTHER COMPETITIVE EXAMS
Co-ordinate Geometry

QUESTIONS

1. PQOR is a rectangle whose vertices are Q (0, 7), O (0, 0) and R (- 5, 0). Find the length of its diagonal.

(a) 84 (b) 8

(c) 12 (d) 74
(e) None of these
2. The point which lies on the perpendicular bisector of the line segmen joining the points P (- 3, - 5)
and Q (3, 5) is
(a) (-3, 4) (b) (2, -5)
(c) (0, 0) (d) (-1, 4)
(e) None of these
3. The points (- 5, 0), (0, 4) and (5, 1) are the vertices of a _______
(a) right triangle (b) isosceles triangle
(c) equilateral triangle (d) scalene triangle
(e) None of these
4. The points (4, 5), (6, 7) and (8, 9) are ________
(a) collinear
(b) not collinear
JOIN @iitwale in Telegram
(c) the vertices of a right triangle
(d) vertices of an isosceles triangle
(e) None of these
5. If a point (x, y) is equidistant from the points (p + q, q - p) and (p - q, p + q), then ________
(a) qx  yp  0 (b) qx  yp
(c) px  qy (d) px  qy  0
(e) None of these

 a 2a 
6.  
If M and N are the points whose coordinates are ap2 , 2ap and  2 ,  respectively and S is the
p p 
1 1
point (a, 0), then + is equal to _________
SM SN
(a) a (b) ap
1 1
(c) (d)
a ap

(e) None of these


7. The coordinates of circumcentre of the triangle whose vertices are (3, 6),(4, -4) and (3, -5) are
 3   3 1 
(a)  , 0  (b)  , 
 2   2 2
1  1 3
(c)  ,  3  (d)  , 
2   3 2
(e) None of these
8. The ratio in which the line 2x  3y  9  0 the points (2, 3) and (4, 7) is _________
(a) 3 : 1 (b)  5 : 4
(c)  1 : 5 (d) 2 : 5
(e) None of these
9. The area of a triangle, whose vertices are (a, a - 2), (a + 2, a + 2) and (a + 3, a), is _________
(a) 1 sq unit (b) 2 sq units
(c) 3 sq units (d) 4 sq units
(e) None of these
1 1
10. If the points (p, 0), (0, q) and (1, 1) are collinear, then + equals to _______
p q

(a) 1 unit (b) 2 unit


(c) 3 unit (d) 4 unit
(e) None of these
11. The distance between the points (cos ,sin ,) and (cos ,  sin ,) is _______

(a) 3 (b) 2 JOIN @iitwale in Telegram


(c) 2 (d) 1
(e) None of these
7 m
12. If A  ,  is the midpoint of the line segment joining the points B (3, -2) and C (4, 6), then the value
3 5 
of m is _______
(a) 4 (b) 10
(c)  4 (d)  8
(e) None of these
13. If the point A (-3, m) divides the line segment joining the points B (-8, -5) and C (1, - 4) in the ratio a
: b, then m equals to
5 40
(a) (b)
3 3
40 30
(c) (d)
9 7
(e) None of these
14. The ratio, in which the line 5x + 3y = - 9 divides the segment joining the points (2, 3) and (- 3, 4), is
________
(a) 4 : 3 (b)  4 : 3
(c)  14 : 5 (d)  14 : 3
(e) None of these

3
15. If a point A (-1, 3) divides internally the line segment joining B (3, 4) and ( in ratio 2 : 3, then the
coordinates of point C is ______

 3
(a) (2,-3) (b)  2, 
 2

 3
(c)  1,  (d) (-2, 3)
 2
(e) None of these
16. The points (6, 1), (8, 2), (9, 4) and (7, 3) are the vertices of a
(a) trapezium (b) rectangle
(c) square (d) rhombus
(e) None of these
17. If the coordinates of mid-point of the sides of a triangle are (21, 7), (- 5, 8, 5) and (10, 8.5), then
which among the following is not a vertex of this triangle?
(a) (  16, 10) (b) (7, 6)
(c) (36, 7) (d) (6, 7)
(e) None of these
18. The third vertex of a triangle, if two of its vertices are (- 5, 2) and (1, - 3) and the centroid is at (- 2,
3), is JOIN @iitwale in Telegram
(a) (- 2, 5) (b) (- 2, 10)
(c) (0, 5) (d) (10, - 2)
(e) None of these
19. The vertices of a  PQR are P (1, 2), Q (- 3, 2) and R (5 - 6). Which among the following is a true
statement for this triangle PQR?
(a) Circumcentre of  PQR is (- 1, - 4)
(b) Circumradius of  PQR is 740 unit
(c) The point where perpendicular bisectors of PQ and PR intersects, is the circumcentre of the triangle PQR.
(d) All the above
(e) None of these
20. Find the orthecentre of the triangle ABC having its vertices as A (- 2, - 3), B (2, 1) and C (5, - 2).
(a) (  3, 5) (b) (  4, 6)
(c) (2, 1) (d) (0, 0)
(e) None of these
21. The equation of a line whose inclination is 60 an of 5 units on x-axis, is

(a) 3x  y  3 (b) 3x  y  5 3

(c) x  3y  3 (d) x  3y  5 3
(e) None of these

4
22. The area of a triangle formed by the line 4x  3y  12  0 with the coordinate axes is __________
(a) 3 sq unit (b) 4 sq unit
(c) 5 sq unit (d) 6 sq unit
(e) None of these
23. The equation of a line which is parallel to the line 3x  5y  8  0 and making an intercept - 3 on x-
axis is
(a) 5y  3 x  9 (b) 5y  3 x  9
(c) 3 x  5y  9 (d) 3 x  5y  9
(e) None of these
24. Which among the following can be the vertices of an equilateral triangle?
(a) (2, 3), (5, 4), (7, 8)
(b) (3, - 6), (5,-9), (6, - 12)

(c) (3,-3), (-3, 3), (3 3, 3 3)

(d) All the above


(e) None of these
25. If the vertices of a triangle have integral coordinates, then the triangle cannot be
(a) an equilateral triangle
JOIN @iitwale in Telegram
(b) a right triangle
(c) an isosceles triangle
(d) All the above
(e) None of these
26. If three vertices (in order) of a parallelogram are (p + q, p - q), (2p + q, 2p q) and (p - q, p + q), then
its fourth vertex is _________
(a)  q,  q  (b)  2q,  2q 

(c)   q, q  (d)  p,  p 

(e) None of these


27. The equation of median drawn from the vertex P to the side QR of a  PQR, whose vertices are P (2,
3), Q (- 3, - 5) and R (6, 2), is _______.
(a) 9 x  y  18 (b) 9 x  y  15
(c) 3 x  8y  18 (d) 3 x  8y  15
(e) None of these
28. A line 6x  5y  30 intersects the coordinate axes. Find the length of the smallest side of the triangle
so formed by the axis.
(a) 3 units (b) 4 units
(c) 5 units (d) 6 units
(e) None of these

5
29. The line segment joining the points (3, - 2) and (4, - 6) is divided by the line 2x  3y  11  0 in the
ratio m: n externally at point (a, b). Which among the following is true for the given information?
(a) m = 11 (b) n = 21
89
(c) a  b  (d) All the above
10
(e) None of these
30. If the mid-points of sides of a  ABC are P (2, 5), Q (- 3, 8) and R (6, 12), then its vertices are ________
(a) (13, 10), (  9, 2), (2, 14)
(b) (12, 9), (  8, 1), (1, 15)
(c) (11, 9), (  7, 1), (1, 15)
(d) (  12, 8), (  7, 2), (  2, 15)
(e) None of these
31. The three medians of a  PQR intersect at a point S. If the medians are PM, QN & RT and the area of
 PQR is 90 cm2 , then the area of the quadrilateral MSRN is ________
(a) 45 cm2 (b) 30 cm2
(c) 15 cm2 (d) 60 cm2
(e) None of these
32. JOIN
The length of the side of a at the of the angle of Pythagorean triangle of @iitwale in 17
8 cm, 15 cm Telegram
cm the
hypotenuse is ________
5 4
(a) 5 cm (b) 6 cm
23 23
3 2
(c) 7 cm (d) 4 cm
23 23
(e) None of these
33. The length of the diagonal of a square, which can be inscribed in a right triangle of sides 5 cm, 12 cm
and 13 cm, is ______

30 2 60 2
(a) cm (b) cm
17 17

(c) 15 cm (d) 10 2 cm
(e) None of these
34. The equation of a line, which passes through (3, 4) and the product of whose intercepts on the
coordinate axes is 48, is _________
(a) 6 x  9y  48 (b) 8 x  9y  48
(c) 4 x  3y  24 (d) 3x  4 y  24
(e) None of these

6
35. If the equations of three concurrent lines are px  6y  8  0, qx  5y  8  0 and rx  4y  8  0, then
the value of p + r is __________
(a) q (b) 2q
(c) 3q (d) 4q
(e) None of these
36. The point (p, q) divides the line formed by joining the points (p + q, p + q) and (p - q, q - p) in the
ratio _______
(a) p : q internally (b) q : p externally
(c) 2 : 1 (d) 1 : 1
(e) None of these
37. The radius of the circle which passes through the origin (0, 6) and (6, 0) is
(a) 12cm (b) 6 cm

(c) 3 2 cm (d) 4 2
(e) None of these
38. Area of the region formed by the lines 3| x |  2| y | 6 is

(a) 8 cm2 (b) 10 cm2


(c) 12 cm2 (d) 14 cm2
JOIN @iitwale in Telegram
(e) None of these
39. The orthecentre of the triangle formed by the lines 5x  8y  10, 16x  10y  13 and y-axis, is _____

 102 95   51 89 
(a)  ,  (b)  , 
 89 178   89 95 

 102 95   51 95 
(c)  ,  (d)  , 
 89 178   89 89 
(e) None of these
40. The line which is perpendicular to 5x  2y  3  0 is _________
(a) 2x  5y  8  0 (b) 3 x  5y  9
(c) 4 x  3y  7 (d) 5 x  2y  3
(e) None of these

7
ANSWER - KEY

1. (d) 2. (c) 3. (d) 4. (a) 5. (b)

6. (c) 7. (b) 8. (c) 9. (d) 10. (a)

11. (b) 12. (b) 13. (c) 14. (c) 15. (b)

16. (d) 17. (b) 18. (b) 19. (d) 20. (c)

21. (b) 22. (d) 23. (a) 24. (a) 25. (b)

26. (c) 27. (b) 28. (c) 29. (d) 30. (c)

31. (b) 32. (a) 33. (b) 34. (c) 35. (b)

36. (d) 37. (c) 38. (c) 39. (a) 40. (a)

JOIN @iitwale in Telegram

8
SEARCH { CYBERWORLDFORU.BLOGSPOT.COM } ON GOOGE

ALL STUDY MATERIALS IN FREE OF COST


HERE YOU WILL GET FOLLOWING MATERIALS :-
 HANDWRITTEN NOTES
 PRINTED NOTES { DETAILED , REVISION , SHORT }
 IMPORTANT QUESTIONS
 BOOKS PDF
 BOOKS SOLUTIONS
 TEST PAPERS
 CHAPTERWISH PREVIOUS YEAR PAPERS
 SAMPLE PAPERS
 CONCEPT MAPS
 HOT QUESTIONS
 NEWS AND UPDATES JOIN @iitwale in Telegram

MATERIALS ARE AVILABLE FOR FOLLOWING :-


 CLASS 9TH
 CLASS 10TH
 CLASS 11TH
 CLASS 12TH
 IIT-JEE
 NEET
 NTSE
 KVPY
 OTHER COMPETITIVE EXAMS
Circles
1. PT is a tangent and PAB is a secant of the 6. If the four sides of a quadrilateral ABCD are
circle meeting the circle at A and B. If PA = tangential to a circle, which of the following
4 cm and AB = 5 cm. What is the length of is true?
PT? (a) AC  AD  BD  CD
(a) 8 cm (b) 6 cm (b) AB  CD  BC  AD
(c) 3 cm (d) 1 cm (c) AB  CD  AC  BD
(d) AC  AD  BC  AB
2. In the given figure, AB is a tangent at P.
7. In the given figure, OA and OB are two radii
of the circle. If PA and PB are tangents to
the circle at A and B respectively which is
the correct statement?

Find APD .
(a) PDE (b) EPB
(c) DPE (d) PED

3. Two concentric circle have their centres at


(a) AOB and APB are the
O. OP = 4 cm and OQ = 5 cm. AB is a
supplementary angles.
chord of the circle and a tangent to the inner
(b) PBO and APB are complementary
circle at P.
angles. JOIN @iitwale in Telegram
(c) PBO and AOB are complementary
angles.
(d) Both (a) and (c)

Find the length of AB. 8. AP is a secant and PT is a tangent to a


(a) 6 cm (b) 5 cm circle. Lf PT=15cm and AB = 8BP, find
(c) 8 cm (d) 9 cm AP.

4. Find the value of x.

(a) 45 cm (b) 36 cm
(c) 25 cm (d) 62 cm
5 8 9. In the figure, find ABC .
(a) (b)
2 3
6 9
(c) (d)
5 4

5. What is the number of common tangents


that can be drawn to two intersecting circles
touching at two points? (a) 60 (b) 120
(a) 2 (b) 4
(c) 180 (d) 90
(c) 3 (d) 1
10. If OA and OB are radii and PA and PB are 14. AB is a diameter of a circle with centre O.
tangents to the circle, which of the following Tangent ST touches the circle at C. If
is quadrilateral OAPB?  BAC  40 , find ACT .
(a) Rectangle
(b) Square
(c) Parallelogram
(d) Kite

11. In the given figure, a circle touches the side


BC of ABC at P and touches AB and AC
produced at Q and R respectively. If AQ =
5 cm, find the perimeter of ABC . (a) 50° (b) 30°
(c) 60° (d) 40°

15. In the given figure, OA is a radius and AB is


a tangent to the circle. (Note that BC is the
shortest distance between B and the circle).

(a) 6cm (b) 10cm


(c) 12cm (d) 15cm
lf OA= 12cm and AB = 16 cm, find the
12. A tangent ST at P to a circle at P is parallel distance between B and C.
to a chord QR of the circle. (a) 7cm
JOIN @iitwale
(b) 6cm
in Telegram
(c) 8cm (d) 12cm

16. An isosceles triangle in which AB = AC is


inscribed in a circle.
Which of the following is correct statement?
(a) P is equidistant from the extremities of
the chord.
(b) PT is the tangent parallel to the chord
PQ.
(c) PT is the tangent parallel to the cord PR. Which of the following is true?
(d) S is equidistant from the extremities of (a) Tangent at A is parallel to BC.
the chord. (b) Tangent at A is perpendicular to AB.
(c) Tangent at A is parallel to AC.
13. In the given figure.TP and TQ are tangents (d) Tangent at A is perpendicular to BC.
to the circle. If PAQ  70 , what is PTQ ?
17. If the distance between two circles of radii R
and r is d, find the length of the direct
common tangent.
(a) d 2  ( R  r )2

(a) 30 (b) 45 (b) d 2  ( R  r )2


(c) 60 (d) 40 (c) ( R  r )2  d 2
(d) d 2  ( R  r )2

3
18. A tangent ST, touches a circle at B.AB is a (22-25): OA and OB are radii and PA
chord such that ABT  65 and PB are tangents of the circle, as
shown in the given figure.

Find AOB , where O is the centre of the


circle.
(a) 120 (b) 145
(c) 110 (d) 130 22. Find APB if OPB  25 .
(a) 50 (b) 40
19. In the given figure, two circles with centres A (c) 70 (d) 25
and B of radii 5 cm and 3 cm touch each
other internally. 23. If AP= 15 cm and the radius is 8 cm, find
the distance of P from the centre of the
circle.
(a) 12cm (b) 15cm
(c) 17cm (d) 20cm

24. If PB = 20 cm and OA = 15 cm, find the


shortest distance between the circle and P.
If the perpendicular bisector of segment AB (a) 25 cm
JOIN @iitwale in Telegram
(b) 15 cm
meets the bigger circle in P and Q, find the (c) 30cm (d) 10cm
length of PQ.
(a) 4 6cm (b) 24 cm 25. If APO  18 , find AOB .
(a) 24 (b) 144
(c) 8 3 cm (d) 4 3 cm
(c) 108 (d) 175

20. In the given figure, 0 is the centre of the 26. PQ is a chord of a circle, which on
circle. PQ is the tangent to the circle and producing, meets the tangent TR at R. If PQ
secant PAB passes through the centre O. = 5 cm and QR = 4 cm, find TR.

If PQ =5 cm and PA = 1 cm, find the


radius of the circle. (a) 3 cm (b) 6 cm
(a) 8cm (b) 12cm (c) 9cm (d) 10cm
(c) 10cm (d) 6cm
27. Two circles of radii 8 cm and 2 cm have
21. A If all the sides of a parallelogram touch a direct common tangent of length 10 cm,
circle, what is the parallelogram? find the distance between their centres.
(a) A rectangle (b) A rhombus (a) 8cm (b) 10cm
(c) A square (d) A trapezium (c) 21 cm (d) 200 cm

4
28. BD and CD are two tangents drawn to a 32. With the vertices of a ABC as centres,
circle with centre O. three circles are described, each touching
the other two externally. If the sides of the
triangle are 9 cm, 7 cm and 6 cm, find the
radii of the circles.

If BDC  44 ,find BAC .


(a) 68 (b) 32
(c) 44 (d) 96

29. If an equilateral triangle ABC is inscribed in


a circle, which of the following is formed by
the tangents at their vertices? (a) 4 cm, 7 cm and 3 cm
(a) A scalene triangle (b) 7 cm, 5 cm and 2 cm
(b) An equilateral triangle (c) 5 cm, 4 cm and 3 cm
(c) An isosceles triangle (d) 4 cm, 5 cm and 2 cm
(d) A right triangle
33. A, B and C are three points on a circle. The
30. In the given figure, ABCD is a quadrilateral tangent at C meets BA produced at T.
in which Q  90 . A circle C (O, r) Given that ATC  36 and ACT  48 ,
find the angle subtended by AB at the
touches the sides AB, BC, CD and DA at P,
centre of the circle.
Q, R, S respectively. If BC = 38 cm, CD =
25 cm, and BP = 27 cm, find the value of r.
JOIN @iitwale in Telegram

(a) 90 (b) 96


(c) 108 (d) 120
(a) 14cm (b) 15cm
(c) 10cm (d) 16cm 34. Tangents AP and AQ are drawn to a circle
with centre 0 from an external point A.
31. In the given circle, O is the centre, QP is a
diameter and PT is a tangent.

Identify the correct statement.


(a) PAQ  2OPQ
(b) PAQ  OPQ
If QOS  120 , find PTS .
(c) PQA  QPA
(a) 30 (b) 40
(c) 60 (d) 80 (d) PQA  2OPA

5
35. PT and QT are tangents drawn to the circle 39. A circle with centre 0 and radius 5 cm is
with centre O. If OT= 25 cm and OP = inscribed in an equilateral triangle ABC.
15cm, find QT.

(a) 20cm (b) 10cm


Find the perimeter of ABC .
(c) 5cm (d) 15cm
(a) 15 3 cm (b) 25 2 cm
36. Two circles with centres 0 and P, and radii 8 (c) 14 2 cm (d) 30 3 cm
cm and 4 cm touch each other externally.
Find the length of their common tangent 40. ABC is a right - angled triangle with
QR. B  90 , AB=12cm and AC=13cm.A
circle with centre 0 is inscribed inside the
triangle.

(a) 8cm (b) 7cm JOIN @iitwale in Telegram


(c) 8 2 cm (d) 7 3 cm

37. PQ is a chord of a circle .the tangent XR at Find the radius of the circle.
X on the circle cuts PQ produced at R. If (a) 8cm (b) 7cm
XR  12 cm , PQ  x cm, QR  ( x  2)cm , (c) 4 cm (d) 2 cm
find x in cm.
41. Three circles with centres A,B and C touch
(a) 6 (b) 7
one another externally. The three centres
(c) 10 (d) 14
are joined to one another to form a triangle.
38. A circle with centre 0 is inscribed in an
isosceles triangle PQR in which PQ = PR=
17 cm and QR = 16 cm.

If AB = 7 cm, BC = 11 cm and AC = 6 cm,


find the radii of the three circles.
(a) 2 cm, 7 cm, 9 cm
Find the radius of the circle. (b) 1 cm, 6 cm, 5 cm
(a) 6.2 cm (b) 4.8 cm (c) 12 cm, 5 cm, 6 cm
(c) 5.6 cm (d) 7.4 cm (d) 3cm, 10 cm, 7 cm

6
42. In the given figure, ABC is an isosceles 44. In the given figure, 0 is the centre of in circle
triangle in which AB = AC.A circle through inscribed in ABC .
B touches AC at its mid-point D and
intersects AB at P.

Which of the following is correct? Find BOC if BAC  40 .


3
(a) AP  AB (b) AP  AB
2 (a) 100
4 3 (b) 120
4 1 (c) 90
(c) AP  AB (d) AP  AB
5 4 (d) 110
45. In ABC, B  90 .If a circle drawn with
43. A tangent CQ touches a circle with centre O AB as diameter intersects the hypotenuse
at P. Diameter AB is produced to meet the AC at P, which of the following is true?
tangent at C. If ACP  a and BPC  b ,
find the relation connecting a and b.

JOIN @iitwale in Telegram


(a) The tangent drawn to the circle at P
bisects the side BC.
(b) The tangent drawn to the circle at A
bisects the side AB.
(c) The tangent drawn to the circle at B
(a) a  b  180 (b) a  2b  90 bisects the side AC.
(c) a  b  60 (d) 2a  b  100 (d) The tangent drawn to the circle at C
bisects the side BC.

7
Answer - Keys

1. B 2. D 3. A 4. D 5. A 6. B

7. A 8. A 9. A 10. D 11. B 12. A

13. D 14. A 15. C 16. A 17. D 18. D

19. B 20. B 21. B 22. A 23. C 24. D

25. B 26. B 27. A 28. A JOIN


29. @iitwale
B in
30.Telegram
A

31. C 32. D 33. B 34. A 35. A 36. C

37. C 38. B 39. D 40. D 41. B 42. D

43. B 44. D 45. A

8
Solutions
1. (b) Not available
17. (d) Not available
2. (d) Not available
18. (d) Not available
3. (a) Not available
19. (b) Not available
4. (d) Not available
20. (b) Not available
5. (a) Not available
21. (b) Not available
6. (b) Given sides of a quadrilateral ABCD are
tangential to a circle. 22. (a) Not available

23. (c) OP2  OA2  AP2


 82  152
OP  289 cm  17 cm

 AB  DC  AD  BC 24. (d) OP2  OA2  PA2 (since, PA= PB.)


OP  25 cm
7. (a) Not available Hence, the shortest distance between the
15  10@iitwale
circle is 25JOIN cm . in Telegram
8. (a) Not available
25. (b) AOP  180  [90  18o ]  72
9. (a) Not available
AOB  72 2  144
10. (d) Not available
26. (b) PR  PQ  QR  5cm  4cm  9cm
11. (b) Not available PR  QR  TR 2
 9cm  4cm  TR 2
12. (a) SPQ  PRQ .....(1)  TR  6cm
(Angle between tangent and chord is equal
to the angle in the alternate segment.) 27. (a) Not available
SPQ  PRQ
......(2) 28. (a) Not available
(Alternate angles as ST | | QR .)
 PRQ  PQR ....... (2) 29. (b) Not available
[From statements 1 and 2.]
 PQ = PR  P is equidistant from Q 30. (a) Not available
and R, the extremities of the chord.
31. (c) Not available
13. (d) Not available
32. (d) Not available
14. (a) Not available
33. (b) Not available
15. (c) Not available
34. (a) Not available
16. (a) Not available 35. (a) Not available

9
36. (c) Join 0 to P and Q. Join P to R. Draw
SP  OQ .
Now SP = QR, as they are opposite sides of
rectangle PRQS.
OP  8 cm  4 cm  12 cm
OS  8 cm  4 cm  4 cm
 SP  OP 2  OS 2
OFA  90
 122  42 cm  8 2 cm
In AOF,AOF=70  y
 QR  8 2cm From the figure,
x  x  y  y  z  z  360o
37. (c) RX 2  RP  RQ BOC  110
 122  (2 x  2)  ( x  2)
 x  10  7 45. (a) Join P to B. Now, BPC  90
Hence, the value of  (in cms) = 10 cm. BPM  MPC  90
But BPM  A
38. (b) Not available A  MPC  90
 A  90  MPC
39. (d) Not available (from ABC )
A  90  C
40. (d) Not available  MPC  C
 MP=MC but, MP=MB
41. (b) Not available MC = MB
JOIN @iitwale in Telegram
Hence, the statement in option (a) is true.
42. (d) Not available

43. (b) Given BPC  b and


ACP  a .
Also, OPA  OAP  b (Angles in an
isosceles triangle OAP, angle in deternate
segment.)
CPO  90
 CPA  90  b
In ACP, ACP  180  [(b  90)  b]
 a  2b  90

44. (d) AO is joined.


Since the circle is the in circle for ABC ,
AO, BO and CO are the angle bisectors of
A, B and C respectively.
1
DAO  FAO  BAC  20o
2

10
SEARCH { CYBERWORLDFORU.BLOGSPOT.COM } ON GOOGE

ALL STUDY MATERIALS IN FREE OF COST


HERE YOU WILL GET FOLLOWING MATERIALS :-
 HANDWRITTEN NOTES
 PRINTED NOTES { DETAILED , REVISION , SHORT }
 IMPORTANT QUESTIONS
 BOOKS PDF
 BOOKS SOLUTIONS
 TEST PAPERS
 CHAPTERWISH PREVIOUS YEAR PAPERS
 SAMPLE PAPERS
 CONCEPT MAPS
 HOT QUESTIONS
 NEWS AND UPDATES JOIN @iitwale in Telegram

MATERIALS ARE AVILABLE FOR FOLLOWING :-


 CLASS 9TH
 CLASS 10TH
 CLASS 11TH
 CLASS 12TH
 IIT-JEE
 NEET
 NTSE
 KVPY
 OTHER COMPETITIVE EXAMS
Arithmetic Progressions

1. Find the general term of the series 4, 7, 10, 9. What is the sum of first 'n' odd numbers
13...... starting from 11?
(a) 3n - 7 (b) 3n + 7 (a) n2  10n (b) n2  10n
(c) 3n +1 (d) 3n – 1 (c) 10n  n 2
(d) n 2

2. Which is the first negative term of the 10. How many two digit numbers are divisible
arithmetic progression 35, 30, 25,.....? by 4?
(a) 7th term (b) 5th term (a) 20 (b) 16
(c) 9th term (d) 11th term (c) 25 (d) 22

3. Find the next term of the arithmetic 11. How many terms of the arithmetic
progression 12, 27, 48 progression 1, 9, 17, .... must be taken to
(a) 75 (b) 60 give a sum of 1540?
(a) 10 (b) 40
(c) 80 (d) 90
(c) 20 (d) 15
4. What is the term of an arithmetic 12. What is the sum of 36 terms of the series
progression whose sum to 'n' terms is
whose nth term is 5n + 4?
2n2  3n ? (a) 4347 (b) 3474
(a) 4n - 3 (b) 4n - 5 (c) 4374 (d) 3447
(c) 4n + 5 (d) 5 - 4n JOIN @iitwale in Telegram
13. Divide 21 into three parts that are in
5. Which term of the arithmetic progression arithmetic progression and whose product is
30, 27, 24,.... is O? 168.
(a) 8th term (b) 10th term (a) 3, 6, 9
(c) 13th term (d) 11th term (b) 14, 17, 11
(c) 2, 6, 14
6. The numbers 28, 22, 'x', 'y', 4 are in (d) 2, 7, 12
arithmetic progression. What are the
respective values of 'x' and 'y'? 14. Which term of the arithmetic progression 3,
(a) 10, 16 (b) 20, 18 8, 13, ..... is 55 more than its 20th term?
(c) 18, 16 (d) 16, 10 (a) 29th term
(b) 31st term
7. What is the sum to 'n' terms of the series (c) 25th term
5, 20, 45, 80,.....? (d) 27th term
n(n  1) 5 n(n  1)
(a) (b)
2 2 15. Identify the formula for then nth term of the
n  (n  1)  arithmetic progression whose first term,
(c) (d) n(n  1) 5 (a) is 2 - x, and the common difference,
2  5 
(d) is 'x'.
(a) 2  x  nx
8. What is the sum of the first 'n' even (b) 2  (n  2) x
numbers? (c) 2  x  nx
(a) n2  1 (b) n(n  1) (d) 2  (n  1) x
(c) n2  n (d) n(n  1)
16. Find the number of terms of the arithmetic 23. A piece of wire is used to make circles of the
1 1 2 11 following pattern.
progression , , ,......
3 2 3 6
(a) 8 (b) 10
(c) 12 (d) 13

17. The first, second and last terms of an


arithmetic progression are 5, 9 and 101
respectively. Find the number of terms in The radius of the first circle is 'r' cm and it is
the arithmetic progression. increased by 1 cm successively. Find the
(a) 30 (b) 50 length of the wire that is required to make 5
(c) 25 (d) 75 such circles.
(a) (10  r )cm
18. Find the sum of the first 18 terms, of the
(b) 2 cm
arithmetic progression 12b, 8b, 4b,......
(a) 126b (c) 10 (r  2) cm
(b) -56b (d) 2 (r  2) cm
(c) 256b
(d) - 288b 24. An arithmetic progression has a 3rd term of
13 and a last term of 148. If the common
19. Find the sum of the arithmetic progression, difference is 5, find the number of terms of
x  2 y,2 x  y,3x......11x  8 y . the progression.
(a) 33(2 x  y) (a) 30
(b) 22(2 x  y) (b) 40 JOIN @iitwale in Telegram
(c) 50
(c) 28( x  y)
(d) 75
(d) 33x  11y
25. Given that the sum of the first 'n' terms of an
20. Find the sum from the sixth term to the arithmetic progression is 2n2 + 3n, find the
twelfth term of the arithmetic progression 6, twelfth term.
10, 14, .... (a) 7 2
(a) 176 (b) 36
(b) 232
(c) 625
(c) 266
(d) 184 (d) 56

21. The first term and the common difference of 26. The first two terms of an arithmetic
an arithmetic progression are 13 and 5 progression are 27 and 24 respectively.
respectively. Find the sum from the 6th term How many terms of the progression are to
to the 20th term. be added to get - 30?
(a) 876 (a) 15 (b) 20
(b) 146 (c) 25 (d) 18
(c) 1095
(d) 1080 27. A secondary school had an enrolment of
1620 students in the year 2009 which
22. Find the value of 'n' if the sum of the first 'n' increases by 150 students per year. What
terms of the A.P.I 5, 23, 31, ....... is 708. was the enrolment in the year 2013?
(a) 9 (b) 8 (a) 2170 (b) 2070
(c) 12 (d) 10 (c) 2220 (d) 2150

3
28. A piece of wire is used to make rectangles 33. The given figure shows a pattern of several
with the same height of 'h' cm. The lengths squares drawn consecutively.
of the rectangles are increased by 1 cm
successively.

Find the total length of wire required to


make 7 rectangles in this pattern.
(a) 2[h  b  7] (b) 2b + h + 7 The perimeters of the squares in figure form
(c) 2n + h + 6 (d) 2[b  h  6] an arithmetic progression. The length of the
first square is x cm and the length of the
29. The postal rates in India from town M to other consecutive squares differ from each
town N are tabulated as shown. other by 1 cm. Given the sum of the
perimeters of the first five squares is 160 cm,
Weight 61- 81- 101- 121- find the sum of the perimeters of the first 10
1-60
(g) 80 100 120 140 squares.
Postal (a) 280cm
3.00 3.50 4.00 4.50 5.00
Rate(`) (b) 240cm
Amir posts a 230 g parcel from town M to (c) 420 cm
town N, what is the postal charge? (d) 360 cm

(a) ` 8.00 (b) ` 7.50 34. In an arithmetic progression, the fourth term
(c) ` 7.00 (d) ` 6.00 is 8 and the sum of 12 terms is 156.
JOIN @iitwale in Telegram
Find the value of 'p' if the p th term is 1000.
30. A cineplex has 13 rows of seats with 10
(a) 200
seats in the first row, 12 in the second, 14 in
(b) 500
the third and so on. What is the total
(c) 300
number of seats in the cineplex?
(d) 100
(a) 252 (b) 256
(c) 258 (d) 286
35. A roll of thread 907i cm long is cut into 5
parts to make up 5 circles as shown in the
31. Sulekha intends to read a 200 page book
figure.
within a specific period-She starts by
reading 11 pages on the first day and
increases her rate of reading by reading an
extra 2 pages for each subsequent day How
long will she take to finish reading the book? The radii of the circles increase by 1 cm
(a) 20 consecutively. Find the radius of the smallest
(b) 10 circle.
(c) 15 (a) 7 cm (b) 5 cm
(d) 25 (c) 6 cm (d) 9 cm

32. The 6th term of an arithmetic progression is 36. A circle is divided into 18 sectors such that
30 and the sum of the first six terms is 210. the angles subtended at the centre form an
Find the sum of the next 6 consecutive arithmetic progression. Given that the angle
terms. of the smallest sector is 11.5 , find the angle
(a) 100 (b) 50 of the biggest sector.
(c) 138 (d) 204 (a) 30 (b) 15.5
(c) 45 (d) 28.5

4
37. An arithmetic progression has 10 terms. The 45. The sum to 'n' natural numbers is S1 sum of
sum of the odd terms is 245 whereas the the squares of 'n' natural numbers is S 2 and
sum of the even terms is 305. Find the sum of the cubes of 'n' natural numbers is
common difference.
S3 Which of the following is equal to 9S 22 ?
(a) 2 (b) 7
(c) 3 (d) 1 (a) (1  8S1 ) S2
(b) S3 (1  8S1 )
38. Given an arithmetic progression -10,-5, (c) S1 (1  8S2 )
0,..... Identify three consecutive terms of the (d) S1 (1  8S3 )
progression whose sum is 90.
(a) 26, 31, 36 (b) 25, 30, 35
46. In an arithmetic progression if 7 times the
(c) 20, 30, 40 (d) 18, 23, 28
7th term is equal to 11 times the 11th term,
find its 18th term.
39. The first three terms of a sequence are 8, 'y'
(a) 1 (b) 0
and 18. Find the positive value of 'y' so that
(c) -1 (d) 2
the sequence is an arithmetic progression.
(a) 26 (b) 18
47. Find the sum of the numbers in between 1
(c) 12 (d) 13
and 1000 which are divisible by 9.
(a) 55944 (b) 54954
40. The sum of the first 10 terms of an
(c) 99994 (d) 99894
arithmetic progression is four times the sum
of its first five terms. Find the ratio of the
48. There are 'n' arithmetic means in between 'a'
first term to the common difference.
(a) 1:4 (b) 4:1 and 'b'. Find the common difference.
(c) 2:1 (d) 1:2 b  a JOIN @iitwale b  ain Telegram
(a) (b)
n 1 n 1
41. How many numbers between 100 and 1000 ba ba
(c) (d)
are divisible by 7? n 1 n 1
(a) 7 (b) 128
(c) 132 (d) 127 49. What is the sum of 'n' arithmetic means
between 'a' and 'b'?
42. Find the sum of 14 AM's between 5 and 8. 3 n
(a) (a  2b) (b) [a  b]
(a) 91 (b) 89 2 2
(c) 90 (d) 85 n n
(c) [a  b] (d) [a  b]
2 4
43. If 13  23  .....  m3  3025 , find m.
 1  2  3
(a) 12 50. Find the sum of 1    1    1   ....
(b) 10  n  n  n
(c) 17 terms.
(d) 12 n n 1
(a) (b)
2 2
44. If mth term of an arithmetic progression is 'n' n 1 n(n  1)
(c) (d)
and nth term is 'm', find its (m  n)th term. 2 n2
(a) 0
(b) m  n  p 51. In an arithmetic progression, if
t p =q, t q  p, find t pq .
(c) m  n
mn (a) p  q  pq (b) p  q
(d)
mn (c) p  q  pq (d) pq  p  q

5
52. Find three numbers in arithmetic
progression whose sum is 3 and product is-
35.
(a) 1,5,7 (b) 5,7, 1
(c) 5,1,7 (d) 7,5, 1

53. Which of the following is incorrect?


(a) S n of the arithmetic progression 3,
13,23,.... is 5n2  8n .
(b) Sn  3n2  8n is the sum of an arithmetic
progression whose common difference is 6.
(c) nth term, tn  sn  sn1
(d) The nth term of an arithmetic progression
is tn  a  (n  1)d .

54. Given P=3+5+7+ .... (n terms) and Q = 5


+ 8 +11 +..... (10 terms).What is the value
P
of 'n' if  7 ?
Q
(a) 25 (b) 35
(c) 30 (d) 20

55. What is the value of 'x' if 2 x, x  10 and 3x  2


JOIN @iitwale in Telegram
are in arithmetic progression?
(a) 5 (b) 6
(c) 4 (d) -6

6
Answer - Keys

1. C 2. C 3. A 4. B 5. D 6. D

7. A 8. D 9. A 10. D 11. C 12. B

13. D 14. B 15. B 16. B 17. C 18. D

19. A 20. C 21. C 22. C 23. C 24. A

25. A 26. B 27. C 28. D JOIN


29. @iitwale
B in
30.Telegram
D

31. B 32. C 33. C 34. B 35. A 36. D

37. D 38. B 39. D 40. D 41. B 42. A

43. B 44. A 45. B 46. B 47. A 48. A

49. B 50. B 51. A 52. C 53. A 54. B

55. B

7
Solutions
1. (c) The general term of an arithmetic  n(10  n)  10n  n2
progression is given by
a  (n  1)d where 'a' is the first term and 'd' 10. (d) Not available
is the common difference.
Here, a = 4 and d  13 10  10  7 11. (c) Not available
743. 12. (b) Not available
 Then nth term 13. (d) Not available
 4  (n  1)3
 3n  1 14. (b) Not available

2. (c) The given arithmetic progression is 35, 15. (b) Not available
30, 25, .....
Here, a = 35 and d = (-5). 16. (b) Not available
nth term, tn  a  (n  1)d
 40  5n 17. (c) Not available
For the first negative term, tn  0
18. (d) Not available
 40  5n  0
 8n  n 8
19. JOIN
(a) Not available @iitwale in Telegram
 n9

20. (c) T6  T7  ......  T12  S12  S5


3. (a) Not available
12
 [2  6  (12  1)  4]
4. (b) Not available 2
5
 [2  6  (5  1)  4]  266
5. (d) Not available 2
Thus, the required sum is 266.
6. (d) Not available
21. (c) t6  13  5(5)  13  25  38
7. (a) Not available t20  13  19(5)  13  95  108
15
S15  [38  108]  1095.0
8. (d) The first 'n' even numbers are 2, 4, 6,… 2
2n.
n n
 Sn  (2  2n)  n(n  1) 22. (c) Sn  [2a  (n  1)d ]
2 2
 [4n  59][n 12]  0
9. (a) Odd numbers starting from 11 are 11, 59
13, 15, 17, 19, ....  n or n  12
4
Sum to 'n' odd numbers, where a = 11, d = Hence, the number of terms is n = 12.
2 is
n 23. (c) The circumferences of the successive
Sn  [2a  (n  1)d ]
2 circles (in cm) are

8
2 r,2 (r  1)  2 (r  2),.....
T2  T1  2 (r  1)  2 r  2 36. (d) Since S10  360, a  115 and d = 1.
T3  T2  2 (r  2)  2 (r  1)  2 tn  a  (n  1)d
5 t15  11.5  (18  1)(1)  28.5o
S5  (4 r  4(2 ))
2  The angle of the biggest sector is 28.5o
 10 (r  2)cm
37. (d) Not available
24. (a) Not available
38. (b) Not available
25. (a) Not available
39. (d) Not available
26. (b) Not available
27. (c) Not available 40. (d) Not available

28. (d) Not available 41. (b) Not available

29. (b) Not available 42. (a) a  15d  8


a = 5 (Given)
30. (d) Not available 1
 d
5
31. (b) Not available JOIN @iitwale in Telegram
26 27 39
5, , ,........ ,8
5 5 5
32. (c) Not available 14   26   1 
S14   2    13   
2  5   5 
33. (c) Not available
14  52  13 
  91
34. (b) Given, t4  8; S12  156 2  5 

 a = 2 and d = 2
43. (b) Sum of cubes of 1st 'm' natural numbers
tn  a  ( p  1)d
 m(m  1) 
2
 tn  a  ( p  1)d 
 2   3025
 2  2( p  1)  1000
Hence, m = 10.
 p  500

44. (a) tm  a  (m  1)d ....(1)


35. (a)
tn  a  (n  1)d ....(2)
Subtracting (2) from (1), we get Solving the
two equations, we get
Circumference of 5 circles d  1 and a  n  m 1
 2 r  2 (r  1)  2 (r  2)  t( mn)  a  (m  n  1)d
2 (r  3)  2 (r  4)  90  n  m 1  m  n  1  0
 10 r  70
 Radius of the smallest circle,
r = 7 cm.

9
 n(n  1)(2n  1)  1 2 
2

45. (b) pS  9 
2
    ........' n ' terms 
2
 6   n n 
n2 (n  1)2 (2n  1)2  1  2  3  .....  ' n ' terms 
9  n 
36  n 
n2 (n  1)2 n(n  1)
 [4n2  4n  1] n
4 2n
 S3[4n(n  1)  1] n  1 n 1
n 
 S3 (8S1  1) 2 2

51. (a) Not available


46. (b) 7t7  11t11
 7[a  6d ]  11[a  10d ]
52. (c) Not available
 a  17(d )  0
 t18  0 53. (a) Not available

47. (a) 9, 18, 27, ......999 54. (b) Not available


tn  a  (n  1)d
 999  9  (n 1)9 55. (b) Not available
 111  n
JOIN @iitwale in Telegram
48. (a) t1  a; tn2  b
tn2  a  (n  2  1)d
 b  a  (n  2  1)d
ba
 d
n 1

49. (b) 1st term of 'n' AM.s between 'a' & 'b'
b  a an  b
a 
n 1 n 1
Last term of 'n' A.M.s between 'a' & V
b  a bn  a
b 
n 1 n 1
Sum of the terms
n  na  b bn  a 
 
2  n  1 n  1 
n  (n  1)(a  b) 
  
2 n 1
n
 ( a  b)
2

50. (b) (1 + 1 + ....... 'n' terms)

10
SEARCH { CYBERWORLDFORU.BLOGSPOT.COM } ON GOOGE

ALL STUDY MATERIALS IN FREE OF COST


HERE YOU WILL GET FOLLOWING MATERIALS :-
 HANDWRITTEN NOTES
 PRINTED NOTES { DETAILED , REVISION , SHORT }
 IMPORTANT QUESTIONS
 BOOKS PDF
 BOOKS SOLUTIONS
 TEST PAPERS
 CHAPTERWISH PREVIOUS YEAR PAPERS
 SAMPLE PAPERS
 CONCEPT MAPS
 HOT QUESTIONS
 NEWS AND UPDATES JOIN @iitwale in Telegram

MATERIALS ARE AVILABLE FOR FOLLOWING :-


 CLASS 9TH
 CLASS 10TH
 CLASS 11TH
 CLASS 12TH
 IIT-JEE
 NEET
 NTSE
 KVPY
 OTHER COMPETITIVE EXAMS
Probability
1. What is the probability of getting a prime 7. From a normal pack of cards, a card is
number in a throw of a die? drawn at random. Find the probability of
1 getting a jack or a king.
(a) 2 (b)
2 2 1
(a) (b)
3 52 52
(c) (d) 6
2 2 1
(c) (d)
13 26
2. What is, the probability that a vowel
selected at random in English alphabet is an 8. Two numbers are chosen from 1 to 5. What
"i"? is the probability for the two numbers to be
1 1 consecutive?
(a) (b)
5 26 1 2
(a) (b)
1 5 5
(c) (d) 1
6 1 2
(c) (d)
10 10
3. When two dice are thrown, what is the
probability of always getting a number 9. Two dice are thrown at a time. What is the
greater than 4 on the second dice? probability that the difference of the
1 1 numbers shown on the dice is 1?
(a) (b)
6 3 5 1
(a) (b)
1 1 18 JOIN @iitwale 36 in Telegram
(c) (d)
36 2 1 1
(c) (d)
6 12
4. What is the probability for a leap year to
have 52 Mondays and 53 Sundays? 10. A bag contains 3 white and 5 red balls. If a
1 1 ball is drawn at random, what is the
(a) (b)
366 52 probability that it is red?
2 1 3
(c) (d) (a)
7 7 8
5
(b)
5. Cm In a single throw of two dice, what is the 8
probability of getting a sum of 10? 3
(c)
1 1 15
(a) (b)
12 36 5
(d)
1 1 15
(c) (d)
6 8
11. What is the probability of getting an even
6. Three letters, to each of which corresponds number when a die is rolled?
an addressed envelope are placed in the 1
(a)
envelopes at random. What is the 6
probability that all letters are placed in the 1
right envelopes? (b)
36
1
(a) (b) 1 (c)
1
3 2
1
(c) (d) 0 (d)
1
6 12
12. OA card is drawn from a packet of 100 17. What is the chance that a non-leap year
cards numbered 1 to 100. Find the contains 53 Saturdays?
probability of drawing a number which is a 2 1
(a) (b)
square. 7 7
1 9 2 1
(a) (b) (c) (d)
10 100 365 365
1 2
(c) (d)
100 100 18. From a well shuffled pack of cards, one card
is drawn at random. What is the probability
13. A book containing 100 pages is opened at that the card drawn is a king?
random. What is the probability that a 12 1
(a) (b)
doublet page is found? 13 13
9 3 1
(a) (c) (d)
100 13 2
90
(b)
100 19. An unbiased coin is tossed. What is the
10 probability that neither head nor tail turns
(c)
100 up?
20 1
(d) (a) 1 (b)
100 2
1
(c) 0 (d)
14. If a coin is tossed twice, find the probability 3
of getting at least one head. JOIN @iitwale in Telegram
1 1 20. A box contains 7 red, 3 white and 2 black
(a) (b)
2 4 balls, when a ball is picked at random from
3 1 the box what is the probability that it is not
(c) (d) red?
4 8
1 11
(a) (b)
15. Find the probability of getting a number 12 12
greater than 2 or an even number in a single (c)
7
(d)
5
throw of a fair die. 12 12
1 2
(a) (b)
3 3 21. An unbiased coin is tossed 5 times. What is
5 3 the odds in favour of getting at least one
(c) (d) tail?
6 5
(a) 31 :1 (b) 1 :31
16. Find the probability that in a family of 3 (c) 32:32 (d) 31 :32
children, there is at least one boy.
3 22. A coin is tossed successively three times.
(a) What is the probability of getting exactly one
4
head or two heads?
1
(b) 3 1
8 (a) (b)
4 4
4
(c) 1 2
8 (c) (d)
3 3
5
(d)
8

3
23. Three of the six vertices of a regular 28. A month is randomly selected from a year.
hexagon are chosen at random. What is the An event X is defined as 'the month with 30
probability that the triangle with these days'. Identify the number of outcomes of
vertices is equilateral? event X.
1 2 (a) 1
(a) (b)
5 5 (b) B
1 3 (c) 3
(c) (d) (d) 4
10 10

24. A coin is tossed successively three times. 29. A spinner is spun. What is the number of
What is the probability of getting exactly one possible outcomes of the event that the
head or two heads? arrow will stop in the sector with an odd
number.
3 1
(a) (b)
4 4
1 2
(c) (d)
3 3
1 1 (a) 1 (b) 2
25. If P( A)  , P( B)  and A and B are (c) 3 (d) 4
3 2
mutually exclusive, find P( A ' B ') .
30. Turn the given cards facing down and
5 1
(a) (b) shuffle.
6 6
1 2
(c) (d)
5 5 Turn oneJOIN card @iitwale
facing up. in Telegram
What is the
probability that it shows a circle?
3 2 5
26. The probability that A can win a race is (a) (b)
8 7 7
1 3 1
and the probability that B can win it is . If (c) (d)
6 7 7
both run in a race, what is the probability
that one of them will win the race, assuming 31. A restaurant operator checked a sample of
that both cannot win together? 200 plates and found that 10 of them were
(a)
17
(b)
15 defective. The chef of the restaurant picks a
24 24 plate from this sample. What is the
13 11 probability that he will get a defective plate?
(c) (d)
24 24 (a) 0.5 (b) 0.05
(c) 0.2 (d) 20
27. A chess piece is randomly selected from a
box that contains all the pieces used in the 32. Two dice are rolled at once and the
game of chess. Identify the sample space of numbers shown are added up. What is the
this experiment. probability of getting a total of 14?
(a) {King, Queen, Bishop, Knight} 1
(a)
(b) 1,2,3,4,5,6,7} 2
(c) {Bishop, Castle, King, Pawn, Queen, (b) 1
Knight} (c) 0
(d) {King, Knight, Pawn, Ace, Queen, 2
Castle, Bishop} (d)
3

4
33. The given figure shows 10 alphabet cards. 38. 7 marbles shown are kept in a tin.

If a marble is taken out randomly from the


What is the probability of getting a card
tin, state the probability that the marble has
labelled 'S' when the card is chosen at
the number 2.
random?
2 3
1 2 (a) (b)
(a) (b) 7 7
5 5
5 4
1 1 (c) (d)
(c) (d) 7 7
10 6
39. Two fair dice are thrown. Find the
34. A weather forecast center predicts that it will
probability that both dice show different
rain for 3 days in a duration of 20 days.
numbers.
Find the probability of rain on a particular
1 5
day. (a) (b)
17 3 6 6
(a) (b) 32 29
20 17 (c) (d)
20 3 36 36
(c) (d)
17 20
40. A box contains 24 coloured marbles.
Eighteen of then are yellow and the rest are
35. 90% of the mangoes in a bag are good. If a
either red or blue. A marble is picked at
mango is chosen randomly from the box,
random. Find the probability of picking an
find the probability of getting a bad mango. JOIN @iitwale in Telegram
yellow marble.
9 1
(a) (b) 1
100 100 (a)
4
9 1
(c) (d) 3
10 10 (b)
4
3
36. A fair coin is tossed thrice. Identify the (c)
probability of getting 3 tails as a fraction. 8
1
(a)
1
(b)
3 (d)
8 8 8
7 1
(c) (d) 41. A certain class has 's' students. If a student is
8 4
picked at random, the probability of picking
8
37. The given figure shows two circles such that a boy is . If the class has 24 boys, what is
1 13
the radius of the small shaded circle is . the value of 's'?
3
(a) 26 (b) 39
times the radius of the big circle. A dart is
(c) 52 (d) 60
thrown randomly towards the circle. Find
the probability that the dart hits the shaded
42. A box contains 60 pens which are blue
target.
inked or black-inked. If a pen is picked at
random, the probability of picking a blue-
inked pen is .What is the number of blue-
1 3 inked pens in the box?
(a) (b) (a) 32 (b) 48
8 8
(c) 30 (d) 24
7 1
(c) (d)
8 4

5
43. A certain class has 45 students. If a student 47. A factory has 120 workers in January. 90 of
is picked at random, the probability of them are female workers. In February,
1 another 15 male workers were employed. A
picking a prefect is . How many students in
3 worker is then picked at random. Calculate
the class are not prefects? the probability of picking a female worker.
(a) 60 (b) 15 (a)
3
(b)
4
(c) 30 (d) 70 4 9
2 1
(c) (d)
44. A bag contains several coloured balls. 28 of 3 3
them are red. If a ball is drawn at random,
4
the probability of drawing a red ball is . x 48. A box contains a number of marbles with
9 serial number 18 to 38.A marble is picked at
balls are added into the box. A ball is then a random. Find the probability that it is a
drawn at random. If the probability of multiple of 3.
1 3
drawing a red ball is now ,find the value of (a)
2 5
x. 7
(b)
(a) 4 (b) 6 20
(c) 5 (d) 7 3
(c)
4
45. A bag contains 40 coins, consisting of ` 2, ` 1
5 and `10 denominations. If a coin is drawn (d)
3
at random, the probability of drawing a ` 2
5 JOIN @iitwale in Telegram
coin is .lf x ` 2 coins are removed from the 49. A box contains 40 marbles of red and blue
8 colour. If a marble is picked at random, the
bag and then a coin is drawn at random, the 3
1 probability of picking a blue marble is .
probability of drawing a ` 2 coin is . Find 8
2 Rana takes out one red marble and nine
the value of x . blue marbles and then picks a marble at
(a) 5 (b) 2 random. Find the probability that it is a blue
(c)10 (d) 8 marble.
4 2
46. The given incomplete table shows the (a) (b)
5 5
number of coins in a box.
7 1
50 (c) (d)
Coins `1 `2 `5 10 40 3
p
Amount 6 12 8 10 ?
If a coin is drawn at random. the probability
50. Set P  {x : 5  x  22, x is an integer} . If an
3
of drawing a ` 2 coin is . Find the element from set P is picked at random,
10
calculate the probability that it is a prime
probability of drawing a 50 p coin.
number.
1
(a) 5 1
10 (a) (b)
18 3
2
(b) 7 5
10 (c) (d)
9 6
1
(c)
5
2
(d)
15

6
51. A box contains 32 coloured marbles. Eight 56. A box contains 20 balls bearing numbers
of them are red marbles and the rest are 1,2,3,...,20.A ball is drawn at random from
either blue or green marbles. A marble is the box. What is the probability that the
drawn at random. Calculate the probability number on the balls is not divisible by 10?
of drawing a marble which is not red in 9 1
(a) (b)
colour. 10 10
2 5 9 1
(a) (b) (c) (d)
3 8 5 5
3 7
(c) (d)
4 16 57. A coin is tossed two times. Find the
probability of getting a tail at least once.
52. A Suppose a die is dropped at random on 3 2
(a) (b)
the rectangular region as shown in the 4 3
figure. 3 1
(c) (d)
5 5

58. If a leap year is selected at random what is


the probability that it will contain 53
What is the probability that it will land inside Tuesdays?
the circle with diameter 2 m? (a)
1
(b)
2
  7 7
(a) (b)
8 28 3 4
(c) (d)
1 1 7 JOIN @iitwale 7 in Telegram
(c) (d)
12 18

53. A lottery has a 0.00002 probability of 59. If P( A  B)  0.65, P( A  B)  0.15, find


winning first prize. How many tickets have P( A)  P( B) .
been sold for the lottery? (a) 1.5 (b) 1.4
(a) 2000 (b) 50000 (c) 1.3 (d) 1.2
(c) 2000 (d) 100000
60. If the odds in favour of winning a race by
54. 250 tickets are sold for a raffle. A girl three horses are respectively 1 : 4, 1 : 5 and
calculates that the tickets bought by her 1 :6, find the probability that one of these
family give them a 0.032 probability of horses will win.
winning first prize. How many tickets did the 37 39
family buy? (a) (b)
60 60
(a) 60 (b) 9
41 51
(c) 50 (d) 8 (c) (d)
60 60
55. All the three cards of spades are removed
from a well-shuffled pack of 52 cards. A 61. Two dice are rolled at once. What is the
card is drawn at random from the remaining probability of getting an even number on
pack. Find the probability of getting a the first die or a total of 8?
queen? 4 5
(a) (b)
3 3 9 9
(a) (b) 7 2
52 49 (c) (d)
1 1 9 9
(c) (d)
26 52

7
Answer - Keys

1. B 2. A 3. B 4. D 5. A 6. C

7. C 8. B 9. A 10. B 11. C 12. A

13. A 14. C 15. C 16. A 17. B 18. B

19. C 20. D 21. A 22. A 23. C 24. A

25. B 26. C 27. C 28. D 29. B 30. A

JOIN @iitwale in Telegram


31. B 32. C 33. A 34. D 35. D 36. A

37. C 38. B 39. B 40. B 41. B 42. D

43. C 44. D 45. C 46. A 47. C 48. D

49. D 50. B 51. C 52. B 53. B 54. D

55. B 56. A 57. A 58. B 59. D 60. A

61. B

8
Solutions
1. (b) Not available n( E ) 3
 p( E )  
n( S ) 4
2. (a) Not available
15. (c) E  {2, 3, 4, 5, 6}  n(E)  5
3. (b) Not available S  {1, 2, 3, 4, 5, 6}  n(S)  6
n( E ) 5
4. (d) Not available  p( E )  
n( S ) 6

5. (a) Not available


16. (a) Not available
6. (c) Not available
17. (b) Not available
7. (c) Not available
18. (b) Not available
8. (b) Not available
19. (c) Not available
9. (a) Not available
20. (d) Not available
10. (b) Not available JOIN @iitwale in Telegram
21. (a) Not available
11. (c) E  {2,4,6}  n( E)  3
22. (a) Not available
S  {1,2,3,4,5,6}  n(S )  6
n( E ) 3 1 23. (c) Not available
 p( E )   
n( S ) 6 2
24. (a) Not available
12. (a) E  {1,4,9,16,25,36,49,64,81,100}
n(E)=10 25. (b) Not available
S  {1, 2, 3, ...... 100}  n(S)  100
n( E ) 10 1 26. (c) Let A be the event that A wins and B be
 p( E )    the event that B wins.
n( S ) 100 10
 P (one of them will win)  P( A  B)
13. (a) E  {11,22,33,44, 55,66,77,88,99}  P(A)  P(B) (Since, A and B are mutually
 n( E )  9 exclusive.)
S  {1, 2, 3, ....... 100}  n(s)  100 3 1 13
  
8 6 24
9
 p( E ) 
100
27. (c) The sample space is S = {Bishop,
Castle, King, Pawn, Queen, Knight}.
14. (c) S  {HH, HT, TH, TT}  n(s)  4
E  {HT, TH, HH}  n(E)  3
28. (d) X = {April, June, September,
November}

9
Hence, n(X)  4 41. (b) Not available

29. (b) X  {1,3} 42. (d) Not available


Hence, n(X)=2 .
43. (c) Not available

30. (a) Since there are 7 cards, n(S) =7 .


44. (d) Let n(S )  s, n( R)  28
Let A be the event of picking a card having
n( R ) 4 28
a circle, n(A) = 2 . P( R)    or S  63
n( S ) 9 S
The probability of picking a card having a
28  x 1
n( A) 2 Now, 
circle P(A) =  63  x 2
n( S ) 7
 x7

31. (b) Since there are 200 plates, n(S )  200 .


45. (c) n(S )  40, let n(C )  C
Let B be the event of picking a defective
5 C 5
plate. Then, n( B)  10 . P(C )    or C  25
8 40 8
The probability of picking a defective plate 25  x 1
n( B) 10 Now,   x  10
 P( B)    0.05 40  x 2
n( S ) 200
46. (a) Not available
32. (c) Not available JOIN @iitwale in Telegram
47. (c) Not available
33. (a) Not available
48. (d) Not available
34. (d) Not available
49. (d) Not available
35. (d) Not available
50. (b) Not available
36. (a) Not available
51. (c) n(S )  32, n( R)  8
37. (c) Area of big circle  S   r 2
8 1
Area of shaded circle, p( R)  
32 4
1  r  The probability of drawing a marble
22
A  r 
3  9 which is not red in colour
 1 3
 1 
r2
n( A) 9 1 4 4
 P( A)   
n( S )  r 2 9
52. (b) n( S )  Area of rectangle
38. (b) Not available  7  4  28 m2
n( A)  Area of circle   r 2   (1)2  
39. (b) Not available
n( A) 
 Probability  
n( S ) 28
40. (b) Not available

10
n( A)
53. (b) P( A) 
n( S )
1
0.00002 
n( S )
1
n( S )   50000
0.00002

n( A)
54. (d) 0.032 
250
n( A)  250  0.032
n( A)  8

55. (b) Total number of cards = 52


3 face cards of spades are removed.
Then remaining cards = 49
3
P (a queen) 
49

56. (a) Numbers divisible by 10 from 1 to 20 are


10 and 20
i.e., 18 numbers are not divisible by 10. JOIN @iitwale in Telegram
n( A) 18 9
 P( A)   
n( S ) 20 10

57. (a) Not available

58. (b) Not available

59. (d) Not available

60. (a) Not available

61. (b) Not available

11
Real Numbers
1. The L.C.M. and H.C.F. of marks scored by 6. If 4 divides 1728, which of the following
Ajit and Amar in a math test are 5040 and statements is true?
12 respectively. If Amar's score is 144, what (a) 4 divides 12. (b) 6 divides 1728.
is Ajit's score? (c) 2 divides 1728. (d) 4 divides 144.
(a) 288 (b) 132
(c) 564 (d) 420 7. Dimensions of a rectangle are (25  7)cm and
(2  52  73 )cm . Express the area of the
2. 'p' is the remainder obtained when a perfect
square is divided by 3.What is the value of rectangle in prime factorization form.
'p'? (a) 2  5  7cm2 (b) 2 x 73 cm2
(a) 1 (c) 26  52  74 cm 2
(d) 25  52  73cm2
(b) 0
(c) Either (a) or (b). 8. Choose the irrational number.
(d) Neither (a) nor (b). (a) 2  4 (b) ( 5)2
(c) 9 4 (d) 2 3
3. The factor tree shows the prime factorization
of 1314.
9. Given a  3  2 . and b  3  2 , which of
the following is correct?
(a) a + b is irrational.
(b) a - b is rational.
(c) ab is rational.
JOIN @iitwale in Telegram
a
(d) is rational.
b
Find the respective values of 'a' and 'b'. 10. Euclid's division lemma: For any two
(a) 3, 37 (b) 3, 73 positive integers 'a' and 'b', there exist
(c) 73, 3 (d) 9, 73 unique integers 'q' and 'r' such that a  bq  r
.
4. The following are the first and last steps in
What is the condition that 'r' must satisfy?
finding the H.C.F. of 408 and 1032 using
(a) 0  r  b (b) 0  r  b
Euclid's algorithm.
Step 1: 1032  408  2  216 (c) 0  r  b (d) 0  r  b
Step 2: ___________
Step 3: ___________
11. Which of the following is a non-terminating
Step 4: 192  24  8  0
repeating decimal?
Choose the steps 2 and 3.
24
(i) 408=2161+1921 (a)
(ii) 408=216+180+12 1600
(iii) 216 =192 1 + 24 (b)
171
(iv) 192 = 24 8 + 0 800
(a) (i) and (ii) (b) (i) and (iii) 123
(c) 2 3
(c) (ii) and (iii) (d) (iii) and (iv) 2 5
145
5. For what value of 'x' does 6" end with 5? (d) 3 2 2
2 5 7
(a) 0
(b) 1
(c) 5
(d) Never ends with 5.
12. Choose the terminating decimal. 19. Which of the following is an incorrect
141 17 statement?
(a) (b)
1000 30 (a) If a  b is an irrational number, then
271 53 ab is also an irrational number.
(c) (d)
90 343 (b) The reciprocal of an irrational number is
always an irrational number.
13. Find the number which when divided by 43 (c) There are infinitely many rational
leaves a remainder 32 and gives a quotient numbers between any two irrational
25. numbers.
(a) 1045 (b) 1107 (d) 7 13  13 is a prime number.
(c) 1150 (d) 1105
20. Which of the following is true for two co-
14. By what number must 1789 be divided to prime numbers?
get a quotient 29 and remainder 49? (a) Their H.C.F. is 1.
(a) 60 (b) 61 (b) TheirL.CM.is1.
(c) 59 (d) 52 (c) Their H.C.F. is equal to their product.
(d) Their L.C.M. is twice their H.C.F.
15. What is the L.C.M. of 140 and 605 if their
H.CF. is H? 21. The difference of the L.C.M. and H.C.F. of
(a) 8000 (b) 5500 210 and 55 is expressed as 210 x 6 + 55y.
(c) 8400 (d) 7700 What is the value of y 3 ?
(a) 361 (b) 19
16. 910 blue pens and 1001 red pens are
(c) 55 (d) 6859
distributed to students of class X so that JOIN @iitwale in Telegram
each student gets the same number of pens
22. Choose the methods that can be used to
of each kind. What is the maximum strength
find the H.C.F. of any two numbers.
of the class?
(i) Euclid's division lemma
(a) 91 (b) 80
(ii) Prime factorization
(c) 94 (d) 86
(iii) Division of the numbers
(iv) Product of numbers
17. Books in a library are stacked in such a way
(a) (i) and (iv) only
that they are stored subject wise and the
(b) (i), (ii) and (iii) only
stacks are of the same size. If there are 144
(c) (i), (iii) and (iv) only
Geography books, 384 History books and
(d) (ii), (iii) and (iv) only
240 Economics books, in the library, in how
many stacks can the books be arranged?
23. A positive number 'n' when divided by 8
(a) 18 (b) 14
leaves a remainder 5. What is the
(c) 16 (d) 12
remainder when 2n  4 is divided by 8?
(a) 8 (b) 1
6 2
18. What is the L.C.M. of and ? (c) 6 (d) 0
14 7
3 6 24. The remainder when a number is divided by
(a) (b)
7 7 143 is 31.What is the remainder when the
4 5 same number is divided by 11?
(c) (d)
7 7 (a) 5 (b) 7
(c) 6 (d) 9

3
25. Three ropes are 7 m, 12 m 95 cm and 3 m 31. 96 books of English, 240 books of hindi and
85 cm long. What is the greatest possible 336 books of mathematics have to be
length which can be used to measure these packed in bundles with each bundle
ropes? containing equal number of books of each
(a) 35 cm (b) 55 cm of the subjects. What is the difference of the
(c) 1 m (d) 65 cm largest number of books which can be
packed in each bundle and the least number
26. Three bulbs are connected in such a manner of bundles which can be made?
that they glow for every 24 seconds, 36 (a) 1
seconds and 54 seconds respectively. All of (b) 3
them glow at once at 8 a.m. When will they (c) 34
again glow simultaneously? (d) 48
(a) 8:30:36 a.m. (b) 8:03:36 a.m.
(c) 8:36:03 a.m. (d) 8:36:30 a.m.
32. Which of the following is true about
27. Find The largest number which divides the 17  41 43  61  43 ?
numbers 120, 224 and 256. (a) It is a prime number.
(a) 8 (b) 6 (b) It is a composite number.
(c) 4 (d) 5 (c) It is an odd number.
(d) Both (a) and (c)
28. A book seller purchased 117 books out of
which 45 books are of mathematics and the 33. A circular field has a circumference of 360
remaining 72 books are of physics. Each km. Two cyclists Sumeet and John start
book has the same size. Mathematics and together and
JOIN cycle at speeds in
@iitwale of Telegram
12 km/hr
physics books are to be packed in separate and 15 km/hr respectively, a round the
bundles and each bundle must contain the circular field. After how many hours will
same number of books. Find the least they meet again at the starting point?
number of bundles which can be made of (a) 100 hours (b) 171 hours
these 117 books. (c) 120 hours (d) 140 hours
(a) 8 (b) 11
(c) 13 (d) 9 34. Find the H.C.F. of 6930 and 8085.
(a) 1155 (b) 2205
29. Sandeep donated 75 glucose biscuits and (c) 1515 (d) 2025
45 monaco biscuits to the students of a
class. These are to be packed in identical p
35. If 0.2317 is expressed in the form of
packets. The two type of biscuits are to be q
packed separately and each containing the where 'p' and 'q' are co-prime and also 'q' is
equal number of biscuits. Find the least
in the form 2n  5m what are the values of 'm'
number of glucose and monaco biscuit
and 'n' respectively?
packets respectively.
(a) 4 and 3 (b) 4 and 5
(a) 5, 15 (b) 5, 3
(c) 4 and 4 (d) 3 and 4
(c) 2, 3 (d) 3, 2
p
30. An army contingent of 616 members is to 36. If 0.737373..... is expressed in the form of
march behind an army band of 32 members q
in a parade. The two groups are to march in , where 'p' and 'q' are co-primes, what are
the same number of columns. What is the the prime factors of 'q'?
maximum number of columns in which they (a) 4 and 7 (b) 3 and 11
can march? (c) 7 and 11 (d) 4 and 3
(a) 3 (b) 8
(c) 12 (d) 4

4
37. Which of the following is correct about 44. The L.C.M. of 318 and 477 is expressed as
41 159  p  318 .What is the value of 'p'?
37500 (a) 2 (b) 4
(a) It is a non-terminating repeating decimal. (c) 3 (d) 0
(b) It is a terminating repeating decimal.
(c) It is a terminating and not repeating 45. A rectangular metal piece of dimensions 360
decimal. cm by 280 cm is cut into some identical
(d) It is a non-terminating and not repeating small squares. If the side of each square has
decimal. the largest possible length, find the number
of square pieces formed.
38. Find the L.C.M. of 3465 and 5460. (a) 126 (b) 20
(a) 181080 (b) 180180 (c) 40 (d) 63
(c) 108108 (d) 108801
46. In a school, the duration of a period in
39. If the LCM Of (480,672) = 3360, find junior section is 40 minutes and in senior
H.C.F. of (480,672). section is 60 minutes. If the first bell for each
(a) 75 (b) 69 section rings at 9 a.m., when will the two
(c) 67 (d) 96 bells ring together again?
(a) 10:45 a.m. (b) 10:15 a.m.
40. Find the respective values of H.C.F. and (c) 12:00 p.m. (d) 11:00 a.m.
L.C.M. of 5474, 9775 and 11730.
(a) 391 and 410550 (b) 319 and 401550 47. M The prime factorization of two numbers
(c) 410550 and 319 (d) 405150 and 193 are 32  73 11 and 3  72 113 17 . Which of
the following
JOIN is @iitwale
a common factor of the
in Telegram
41. The circumferences of the front wheel and numbers?
the rear wheels of a tricycle are 120 cm and (a) 1683 (b) 5831
90 cm respectively. Before beginning to ride (c) 1089 (d) 539
the tricycle, Ruth marks the points where the
tyres touch the ground as A and B
respectively on the front and the rear
wheels. How many revolutions do the front
and rear wheel make when both A and B
touch the ground again simultaneously?
(a) 6, 8 (b) 3, 4
(c) 9, 12 (d) 1, 4

42. Which of the following is a correct statement


(a)  is a natural number.
(b)  is an irrational number.
(c)  is not defined.
22
(d) The value of  is .
7

43. The product of L.C.M. and H.C.F. of two


numbers is 88288. If one of the numbers is
248, find the other number.
(a) 356 (b) 635
(c) 365 (d) 653

5
Answer - Keys

1. D 2. C 3. B 4. B 5. D 6. A

7. C 8. D 9. C 10. C 11. D 12. A

13. B 14. A 15. D 16. A 17. C 18. B

19. D 20. A 21. D 22. B 23. C 24. D

25. A 26. B 27. A 28. C JOIN


29. @iitwale
B in
30.Telegram
B

31. C 32. B 33. C 34. A 35. C 36. B

37. A 38. B 39. D 40. A 41. B 42. B

43. A 44. B 45. D 46. D 47. D

6
Solutions
1. (d) The product of two numbers is equal to 7. (c) The dimensions of a rectangle are
the product of their L.C.M. and H.C.F. (25  7)cm and (2  52  73) cm .
Let Ajit's score be ' x ' . Its area  (25  7)(2  52  73 ) cm2
Then 1441 x  5040 12
 26  52  74cm2
5040 12
 x  420
144
8. (d) 2  4  2  2  0
(Rational number)
2. (c) The square of a positive integer 'm' is of
the form 3m or 3m + 1 (for some 'm'). ( 5)2  5 (Rational number)
Hence, the remainder obtained when a 9  4  3 2 1
perfect square is divided by 3 is either 0 or (Rational number)
1. Also, the sum or difference of two irrational
numbers is irrational. Thus, 2  3 is
3. (b) 219  3  657 irrational.
and 3  73  219
Hence, a = 3 and b = 73. 9. (c) Not available

4. (b) Using Euclid's algorithm, H.C.F. of 408 10. (c) Not available
JOIN @iitwale in Telegram
and 1032 is:
Step 1: 1032  408  2  216 11. (d) Not available
Step 2: 408  216  1  192
12. (a) Not available
Step 3: 216  192  1  24
Step 4: 192  24  8  0
13. (b) Not available
Hence, the required steps are (i) and (iii)
only.
14. (a) By Euclid's lemma,
1789  29 x  49 , where ' x ' is the divisor.
5. (d) If 6 x ends with 5, then 6" would contain
 1789  49  29x
the prime 5.
1740
But 6x  (2  3) x  2x  3x .  x  60
29
 The only prime numbers in the
factorization of 6 are 2 and 3.
x
15. (d) The product of two numbers = The
 By uniqueness of fundamental theorem, product of their L.C.M. and H.C.F.
there are no primes other than 2 & 3 in 6 x .  140  605  11 L.C.M .
So, 6" will never end with 5. 140  605
 L.C.M .   7700
11
6. (a) If 'p' divides x 3 , then 'p' divides ' x ' .
Here, 1728 = 12s. 16. (a) The maximum strength of class X
So, 4 divides 1728 means 4 divides 123. = H.C.F. (910, 1001)
Thus, 4 divides 12 is the required statement. 1001  9110  91
910  9110  0

7
Hence the maximum strength of class X is 120  104 1  16
91. 104  16  6  8 and 16  8  2  0
Thus, H.C.F.
17. (c) H.C.F. of 144, 384 and 240 is 48. (224, 120) = 8.
Number of stacks Now, find the H.C.F. of 8 and the third
 144 384 240  number 256.
     3  8  5  16
 48 48 48  256  8  32  0
i.e., H.C.F. (256, 8) = 8
a c L.C.M .(a, c) Hence, the largest required number is 8.
18. (b) L.C.M. of and 
b d H .C.F .(b, d )
28. (c) Not available
6 2
 L.C.M . of and
14 7
29. (b) Not available
L.C.M .(6, 2) 6
 
H .C.F .(14,7) 7
30. (b) Not available

19. (d) 7 13  13  104  23 13 31. (c) Not available


The product of two prime numbers is
composite. 32. (b) Not available
 7 13  13 is not a prime number.
33. (c) Not available
JOIN @iitwale in Telegram
20. (a) The H.C.F. of two co-prime numbers is
1. 34. (a) Not available

21. (d) Not available 35. (c) Not available


36. (b) Not available
22. (b) Not available
37. (a) Not available
23. (c) Not available
38. (b) 3465  32  5  7 11
24. (d) Not available
5460  22  3  5  7  l3
 L. C. M. (3465, 5460)
25. (a) Not available
 22  32  5  7 1113  180180

26. (b) All the three bulbs glow at once at 8 a.m.


39. (d) The two numbers are 480 and 672.
The time when they glow
Their L.C.M. = 3360.
simultaneously again = L.C.M. (24, 36, 54)
Product of the numbers
seconds =216 seconds = 3 minutes 36  H .C.F . 
L.C.M .
seconds
 The time when the three bulbs glow
40. (a) 5474  2  7 17  23
together again is at 8 : 03 : 36 a.m.
9775  52 17  23
27. (a) Let us find H.C.F. of 120 and 224. 11730  2  3  5 17  23
224  120 1  104  H. C. F. (5474, 9775, 11730)
 17  23  391 and

8
L. C. M.(5474, 9775, 11730)
 2  3  52  7  l7  23  410550

41. (b) Not available

42. (b)  is an irrational number.


22 22
is the nearest value of 71. Apart from
7 7
,  also has other nearest values

43. (a) Not available

44. (b) Not available

45. (d) Not available

46. (d) The L.C.M. of 40 and 60 will give the


number of minutes after which the two bells
will ring together again.
Now, 40  23  5
60  22  3  5 JOIN @iitwale in Telegram
L. C. M. (40, 60)
 22  3  5  2  120
Hence, the two bells ring together again
after 120 minutes i.e., after 2 hours i.e., at
11: 00 a.m.

47. (d) Not available

9
SEARCH { CYBERWORLDFORU.BLOGSPOT.COM } ON GOOGE

ALL STUDY MATERIALS IN FREE OF COST


HERE YOU WILL GET FOLLOWING MATERIALS :-
 HANDWRITTEN NOTES
 PRINTED NOTES { DETAILED , REVISION , SHORT }
 IMPORTANT QUESTIONS
 BOOKS PDF
 BOOKS SOLUTIONS
 TEST PAPERS
 CHAPTERWISH PREVIOUS YEAR PAPERS
 SAMPLE PAPERS
 CONCEPT MAPS
 HOT QUESTIONS
 NEWS AND UPDATES JOIN @iitwale in Telegram

MATERIALS ARE AVILABLE FOR FOLLOWING :-


 CLASS 9TH
 CLASS 10TH
 CLASS 11TH
 CLASS 12TH
 IIT-JEE
 NEET
 NTSE
 KVPY
 OTHER COMPETITIVE EXAMS
Pair of Linear Equations in Two Variables
1. Which of the following is a linear equation? 8. What is a system of simultaneous equations
(a) x  2 y  7 (b) x3  1  0 called if its graph has intersecting lines?
6 x 3 (a) Inconsistent system
(c) x   12 (d)   14 (b) Consistent system
x 2 x
(c) Dependent system
(d) Independent system
2. Which of the following is a solution of
2p  3q  5 ?
9. IHI What is the nature of the graphs of a
(a) p  1,q  1 (b) p  1, q  1 dependent system?
(c) p  1,q  1 (d) p  1,q  1 (a) Parallel lines
(b) Perpendicular lines
3. Which of the following is the other name for (c) Intersecting lines
a pair of linear equations in two variables? (d) Coincident lines
(a) Consistent equations
(b) Simultaneous equations 10. What is the nature of the graphs of a system
(c) Inconsistent equations of linear equations with exactly one
(d) Dependent equations solution?
(a) Parallel lines
4. What is the condition that a system of (b) Perpendicular lines
simultaneous equations a1x  b1 y  c1  0 and (c) Coincident lines
a2 x  b2 y  c2  0 must satisfy to have exactly (d) Intersecting lines
one solution?
JOIN @iitwale in Telegram
11. What is the number of solutions of the pair
a b a b
(a) 1  1 (b) 1  1 of linear equations 4p  6q  18  0 and
a2 b2 a2 b2
2p  3q  9  0 ?
a1 c1 b c
(c)  (d) 1  1 (a) 0
a2 c2 b2 c2 (b) 1
5. How many solutions do the equations (c) 2
a b c (d) Infinitely many
satisfying 1  1  1 have?
a2 b2 c2
(a) One (b) Two 12. Which of the following is a consistent system
(c) Three (d) Infinitely many of simultaneous equations?
(a) m  3n  6 (b) a  3b  6
6. What is a system of simultaneous equations 2m  6n  12 2a  3b  12
called if it has no solution? (c) x  4y  6 (d) l  2m  6
(a) Consistent system 2x  8y  12 3l  6m  12
(b) independent system
(c) Inconsistent system 13. Find the unique solution of the system of
(d) Dependent system simultaneous equations 2x  y  2 and
4x  y  4 .
7. How many solutions does the system of (a) x  0, y  1
equations p  2q  4 and 2p  4q  12  0
(b) x  0, y  0
have?
(c) x  1, y  0
(a) 0 (b) 1
(c) 2 (d) 3 (d) x  1, y  1
14. The sum of a two-digit number and the 2 2
(a) k  (b) k 
number obtained by reversing its digits is 3 3
154. If the digits differ by 4, find the 2 2
number. (c) k  (d) k 
9 9
(a) 95 (b) 73
(c) 84 (d) 62 21. If the length of a rectangle is increased by 2
m and breadth is reduced by 2 m, its area
15. Choose the dependent system from the decreases by 28 sq. m. If the length is
following. reduced by 1 m and the breadth is
(a) m  n  7 (b) 3x  2y  5 increased by 2 m, the area increases by 33
3m  3n  21 2x  3y  7 sq m. Find the actual measurements of the
(c) 3x  3y  18 (d) 2x  y  6 rectangle.
x  y  10 4x  2y  4 (a) l  13m,b  11 m
(b) l  23 m, b  11 m
16. Five years ago, a father's age was seven (c) l  23m,b  20m
times his son's age. Five years from now, the (d) l  12 m, b  10 m
father's age will be thrice the son's age.
What are the respective present ages of 22. The side of a square is 4m more than the
father and son? side of another square. The sum of their
(a) 40 years, 10 years areas is 208 sq. m. What is the side of the
(b) 10 years, 40 years larger square?
(c) 25 years, 5 years (a) 12m (b) 8m
(d) 30 years, 8 years (c) 9m (d) 5m
JOIN @iitwale in Telegram
17. Rajesh buys 7 books and 6 pens for `3800 23. Two numbers are in the ratio 2 :7. If 6 is
and Amar buys 3 books and 5 pens of the added to each of the numbers, the ratio
same kind for `1750. What are the becomes 1 :3. Find the numbers.
respective costs of a book and a pen? (a) 14, 49 (b) 16, 56
(a) `350, `50 (b) `500, `75 (c) 18, 63 (d) 24, 84
(c) `250, `100 (d) `500, `50
24. If the pair of equations 3x  5 y  k and
18. The system of simultaneous equations 9 x  12 y  6 has infinitely many solutions,
3m  n  1and(2k  1)m  (k 1)n  2k  1 , is
which of the following is true?
inconsistent. What is the value of 'k'? (a) k = 2 (b) k = 6
(a) 3 (b) 1 (c) k  6 (d) k = 3
(c) 2 (d) 0
25. If the pair of linear equations 3x  5 y  3 and
19. What type of a system of equations is the
6 x  ky  6 do not have any solution, which
pair of linear equations 2x - 3y = 8 and 4x -
6y = 9? of the following is true?
(a) Consistent system (a) k = 5 (b) k=10
(b) Inconsistent system (c) k  10 (d) k  5
(c) Dependent system
(d) Independent system 26. When is the pair of linear equations
k
7 x  3 y  4;3x y  4 consistent?
20. If the pair of linear equations 2x  ky  3  0 7
2 (a) k = 9 (b) k  9
and 6x  y  7  0 has a unique solution, (c) k  9 (d) k  7
3
which of the following is true?

3
27. When does the pair of linear equations 35. If 12a  3b  1 and 7b  2a  9 , find the
7 x  ky  k ;14x  2 y  k  1 have infinitely average (arithmetic mean) of 'a' and 'b'.
many solutions? (a) 2.5 (b) 1
(a) k = 1 (b) k  1 (c) 0.1 (d) 0.5
(c) k = 2 (d) k = 4
36. If 4 x  6 y  32 and 4 x  2 y  4 , find the value
28. The pair of linear equations x  y  3 ; of 8y.
2 x  5 y  12 has a unique solution x  x1 , (a) 24 (b) 28
y  y1 Find the value of x1 . (c) 36 (d) 42
(a) 1 (b) 2
(c) -1 (d) -2  1
37. For the equations ( p  2)  q    pq  5
 2
29. Which of the following solutions does the  1
and ( p  2)  q    pq  5 , find the solution
pair of linear equations x  2 y  5 ;  2
3x  12 y  10 have? set (p, q).
(a) A unique solution  1  1
(b) No solution (a)  10,   (b)  10, 
 2  2
(c) More than two solutions
 1  1
(d) Infinitely many solutions (c) 10,   (d) 10, 
 2  2
30. If the sum of the ages (in years) of a father
and his son is 65 and twice the difference of 38.
2 3
Identify the solution of x  y  1 and
their ages (in years) is 50, what is the age of JOIN @iitwale3 4 in Telegram
the father? 8x  9 y  16 .
(a) 45 years (b) 40 years (a) x  6, y  4
(c) 50 years (d) 55 years (b) Infinitely many solutions
(c) x  4, y  6
31. Three chairs and two tables cost `1850.Five
(d) No solution
chairs and three tables cost `2850. Find the
total cost of one chair and one table.
39. Find the values of 'x' and y for x  y  0.9 ;
(a) `800 (b) `850
(c) `900 (d) `950 11
2
x y
32. If a  b  5 and 3a  2b  20 , find 3a  b . (a) 3.2, 5.6
(a) 25 (b) 20 (b) 3.2, 2.3
(c) 15 (d) 10 (c) 5.6, 2.3
(d) 4.5, 6.4
33. Which of the respective values of 'x' and 'y'
satisfy the following equations I and II? 40. What is the solution of the equations,
(I) 3x  y  19 (II) 3x  y  1 2 x  y  2 3x  2 y  1
  ?
x y 9 3 5 6
(a) 7, 2 (b) 7, -2 (a) x  1, y  1
(c) -7, 2 (d) -7, -2 (b) x  1, y  1
(c) x  1, y  2
x 5 (d) x  2, y  1
34. If 3x  5 y  5 and  , what is the value
x y 7
of x  y ?
(a) 9 (b) 6
(c) 4 (d) 3

4
41. The course of an enemy submarine as 47. cm Find 'x' and 'y' for the equations
plotted on a set of rectangular axes gives the x y
 4
equation 2 x  3 y  5 . On the same axes. the 3 4
course of a destroyer is indicated by the 5x y
and   8 .
equation x  y  10 . Find the point ( x, y) at 3 4
which the submarine can be destroyed. (a) x  8, y  6 (b) x  3, y  4
(a) (-7, 3) (b) (7, -3) (c) x  6, y  8 (d) x  4, y  6
(c) (-3, 7) (d) (3, -7)
48. How many solutions does the system of
42. Find the solution of the equations equations, 3x  4 y  5 and 12 x  16 y  20
8x  9 y  6 xy 10 x  6 y  19 xy .
have?
3 3 2 3 (a) More than two solutions
(a) x  , y  (b) x  , y 
2 2 3 2 (b) Exactly two solutions
3
(c) x  , y 
2
(d) x  3, y  2 (c) Exactly one solution
2 3 (d) No solution

43. Find the values of 'x' and y, for the 49. Find the number of solutions of the
a 2 b2 a 2b b 2 a 1
equations x   2 and 2 xy  3 y  2 .
equations   0;   ab
x y x y y
where x, y  0 . (a) 0 (b) 1
(c) 2 (d) Infinitely many
(a) x  a 2 , y  b2 (b) x  b2 , y  a 2
b a 1 1 50. Which ofJOINthe following solutions do the
(c) x  , y  (d) x  , y  @iitwale in Telegram
a b b a system of equations 2 x  y  5 and x  2 y  4
have?
1 3 (a) Consistent and a unique solution
44. If x   5 and 2 x   13 , what is the
y y (b) Consistent and infinitely many solutions
value of (2 x  3 y) ? (c) Inconsistent
(a) 1 (b) 2 (d) No solution
(c) 3 (d) 5
51. If the equations 4 x  7 y  10 and
45. Which of the following is the solution of the 10 x  ky  25 represent coincident lines, what
4 is the value of 'k'?
system of equations  5 y  7 and
x 17
3 (a) 5 (b)
 4y  5 ? 2
x 27 35
1 1 (c) (d)
(a) x   , y  1 (b) x  , y  1 2 2
3 3
1 1 52. For what value of 'k', will the equations
(c) x   , y  1 (d) x  , y  1
3 3 4 x  6 y  11 and 2 x  ky  7 be inconsistent?
(a) 2 (b) 3
46. The solution of 2 x  3 y  2 and 3x  2 y  2 (c) 4 (d) 8
can be represented by a point. In which of
the following parts of the coordinate plane 53. For what value of 'k' will the system of
does the point lie? equations 3x  5 y  2 and kx  10 y  0 have a
(a) First quadrant (b) Second quadrant non zero solution?
(c) Third quadrant (d) Fourth quadrant (a) 0 (b) 2
(c) 6 (d) 8

5
54. If the cost of 3 audio cassettes and 2 VCDs (a) ` 900 (b) ` 350
is ` 350 and that of 2 audio cassettes and 3 (c) ` 650 (d) ` 700
VCDs is `425, what is the cost of a VCD?
(a) ` 140 (b) ` 125 61. The angles A, B, C and D in order in a
(c) ` 115 (d) ` 110 cyclic quadrilateral are (2 x  y)o ,(2( x  y))o ,
(3x  2 y)o , and (4 x  2 y)o . Find their
55. The difference between two numbers is 5
and the difference between their squares is measures in the same order.
65. Find the larger number. (a) 70, 110, 80, 100
(a) 9 (b) 10 (b) 70, 80, 110, 100
(c) 11 (d) 12 (c) 70, 80, 100, 110
(d) 80, 100, 110, 70
56. `49 was divided among 150 children. Each
girl got 50 paise and a boy 25 paise. How 62. The smallest angle of a triangle is one-fifth
many boys were there? the sum of the other two and the largest
(a) 100 (b) 102 angle exceeds the sum of the other two by
(c) 104 (d) 105 20 . Find the largest angle of the triangle.
(a) 100 (b) 90
57. The area of a rectangle increases by 76 (c) 120 (d) 110
square units, if the length and breadth are
each increased by 2 units. However, if the 63. The sum of Raju's age and half of Sameer's
length is increased by 3 units and breadth is age is 4. One-third Raju's age added to
decreased by 3 units, the area gets reduced twice Sameer's age is 5. Find the sum of
by 21 square units. Find the sum of the their ages.JOIN @iitwale in Telegram
length and breadth of the rectangle. (a) 7 years (b) 3 years
(a) 40 units (b) 42 units (c) 5 years (d) 2 years
(c) 4 units (d) 36 units

58. What number must be added to each of the


numbers, 5, 9, 17, 27 to make them
proportionate?
(a) 2 (b) 1
(c) 3 (d) 5

59. Two numbers differ by 3 and their product


is 54. Find the numbers.
(a) 9 and 6
(b) -9 and -6
(c) Both (a) and (b)
(d) 9 and -4

60. A part of the monthly expenses of a family is


constant and the remaining varies with the
price of wheat. When the price of wheat is `
250 per quintal, the monthly expenses of
the family is ` 1000 and when it is ` 240 per
quintal, the monthly expenses is ` 980. Find
the monthly expenses of the family on
wheat when the cost of wheat is ` 350 a
quintal.

6
Answer - Keys

1. A 2. C 3. B 4. B 5. D 6. C

7. A 8. B 9. D 10. D 11. D 12. B

13. C 14. A 15. A 16. A 17. D 18. C

19. B 20. D 21. B 22. A 23. D 24. A

25. B 26. C 27. A 28. A 29. A 30. A

JOIN @iitwale in Telegram


31. B 32. A 33. B 34. D 35. D 36. B

37. D 38. D 39. B 40. B 41. B 42. C

43. A 44. D 45. B 46. A 47. C 48. A

49. A 50. A 51. D 52. B 53. C 54. C

55. A 56. C 57. D 58. C 59. C 60. D

61. B 62. A 63. C

7
Solutions
1. (a) A linear equation is of degree 1. The To find if the system of simultaneous
standard form of a linear equation is equations is consistent check if it.
ax  by  c  0 . a b c
satisfies the condition 1  1  1
a2 b2 c2
2. (c) Substitute the given values of 'p' and 'q' 12. (b) Given, system of equations are a  3b  6
in 2p + 3q = 5 and check whether the and 2a  3b  12 .
equation is satisfied. a1 1 b1
Substituting p = 1 and q = 1 in 2p + 3q =
  ;  1;
a2 2 b2
5, makes it true.
a1 b1
 
a2 b2
3. (b) The pair of linear equations in two
Hence the system is consistent.
variables is also known as simultaneous
equations.
13. (c) 2x  y  2 ....... (1)
4. (b) The system of simultaneous equations 4x  y  4 ....... (2)
a1x  b1 y  c1  0 and 2 y
From (1), x 
a2 x  b2 y  c2  0 , have exactly one (unique) 2
 2 y 
solution if
a1 b1
 .  4  y0
  y  4@iitwale
a2 b2  2 JOIN
 in Telegram
20
 x 1
5. (d) Not available 2
x  1 and y = 0 is the unique solution of the
6. (c) Not available given system of simultaneous equations.

7. (a) Not available 14. (a) Not available

8. (b) Not available 15. (a) Not available

9. (d) Not available 16. (a) Not available

10. (d) Not available 17. (d) Not available

11. (d) Given: 4p  6q  18  0 and 18. (c) Not available


2p  3q  9  0
19. (b) Not available
a1 b c
  2; 1  2 and 1  2
a2 b2 c2
20. (d) Not available
a1 b1
 
a2 b2 21. (b) Not available
Thus, the given system of equations has
infinitely many solutions. 22. (a) Not available

8
23. (d) Not available 37. (d) Not available

24. (a) Not available 38. (d) Not available

25. (b) Not available 39. (b) Not available

26. (c) Not available 3x  y  1 2 x  y  2


40. (b) Given 
3 5
27. (a) Not available 3x  2 y  1

6
28. (a) Not available Equate any two equations:
29. (a) Given linear equations are 3x  y  1 2 x  y  2
 
x  2 y  5 and 3x  12 y  10 3 5
Here, a1  1, b1  2, c1  5  9x  8 y 1  0 …….(1)
and a2  3, b2  12, c2  10 2 x  y  2 3x  2 y  1
And 
a1 1 b1 1 c1 1 5 6
  ,  ,   3x  4 y  7  0 .....(2)
a2 3 b2 6 c2 2
a1 b1 c1 Solving eq (1) and (2), we get
   y = 1 and x  1 .
a2 b2 c2
The solutionJOIN @iitwale
of equations is in Telegram
Hence, the given system of linear equations
x  1, y  1 .
has a unique solution.

41. (b) Not available


30. (a) Let the age of father be 'x' years.
Let the age of son be 'y' years.
Given, x  y  65 ...... (1) 42. (c) Not available

and 2( x  y)  50
43. (a) Not available
x  y  25 ......(2)
Adding eq. (1) and eq. (2), we get 44. (d) Not available
 x  45
Hence, the age of father = 45 years. 45. (b) Not available

31. (b) Not available 46. (a) Not available

32. (a) Not available 47. (c) The given equations can be written as
4 x  3 y  48 ..... (i)
33. (b) Not available 20 x  3 y  96 ..... (ii)
Adding (i) and (ii), we get
34. (d) Not available
24 x  144  x6
Substituting x = 6 in (i). we get
35. (d) Not available
y 8.

36. (b) Not available

9
48. (a) Given equations are 3x  4 y  5
and 12 x  16 y  20 52. (b) Not available
Here a1  3, b1  4, c1  5
53. (c) Not available
and a2  12, b2  16, c2  20
a1 1 b1 1 c1 1 54. (c) Not available
  ;  ; 
a2 4 b2 4 c2 4


a1 b1 c1
  55. (a) Not available
a2 b2 c2
Hence, the given equations have infinitely 56. (c) Let the no. of girls be 'x'
many solutions i.e., more than two and the no. of boys be 'y'.
solutions. Given, 0.50 x  0.25y  49
and x  y  150 .... (1)
49. (a) Given equations become x x
1    49 …. (2)
x 2 .....(i) 2 y
y From (1) & (2) x  46, y  104
and 2 xy  3 y  2
Hence, number of boys (y) = 104.
2
 2x   3 ……(2)
y 57. (d) Let the length of the rectangle be 'x'
1 units, and the breadth be 'y' units. Then in
Let  z . then the equations are JOIN @iitwale in Telegram
y the first case,
x  z  2 and 2 x  2 z  3 . ( x  2)  ( y  2)  xy  76 i.e.,
Here, a1  1, b1  1, c1  2 and 2 x  2 y  4  76 ....(1)
a2  2, b2  2, c2  3 i.e., x y 4 ..... (1)
a1 1 b1 1 c1 2 In the second case,
  ;  ; 
a2 2 b2 2 c2 3 ( x  3)  ( y  3)  xy  21
a1 b1 1 c1 2 i.e., x y 4
   ; 
a2 b2 2 c2 3 .....(2)
Hence, the equations do not have any Adding eq. (1) and eq. (2), we get
solution. 2 x  40  x  20 units
x  y  4  y  20  4  16 units
50. (a) Given equations are Hence, the length of rectangle is 20 units
2x  y  5 and the breadth is 16 units.
and x  2y  4  Their sum =20+16
= 36 units
Here a1  2, b1  1, c1  5
and a2  1, b2  2, c2  4
58. (c) Four numbers are in proportion if

a1 b1 c1
  First  Fourth = Second  Third
a2 b2 c2 Let 'x' be added to each of the given
Hence, the given equations have a unique numbers to make the numbers
solution and are consistent. proportionate.
Then,
51. (d) Not available

10
(5  x)(27  x)  (9  x)(17  x)
 x3

59. (c) Not available

60. (d) Not available

61. (b) Not available

62. (a) Not available

63. (c) Not available

JOIN @iitwale in Telegram

11
Quadratic Equations
1. Identify the quadratic equation from the 7. What are the roots of
following. 17a 2  20a  10  10a 2  2a  7 ?
1
(a) p   1, p  0
1
(b) p 2   1, p  0 1 1
(a) ,3 (b) 3,
p p 7 7
1 1 1
(c) x 2   1, x  0 (d) x2  2 x  1  0 (c) , 3 (d) 3, 
x 7 7

2. Find the roots of the quadratic equation 4 x2


1 1 8. Identify the factors of  4x  5 .
(a) ,1 (b) 1, 5
2 2 2 2 5 5
1 1 (a) , (b) ,
(c) ,1 (d) 1, 5 5 2 2
2 2 5 5 2 2
(c) , (d) ,
2 2 5 5
3. Which of the following statements is correct?
(a) x  1 is a root of 2 x2  3x  1  0 . 9. The age of a man is the square of his son's
(b) x  2 is not a root of 6 x2  7 x  5  0 . age. A year ago, the man's age was eight
(c) x  1 is a root of 3x2  x  1  0 . times the age of his son. What is the present
2 age of the man?
(d) x   is not a root of 5x2  8x  4  0 . (a) 47 years (b) 49 years
5
1 (c) 36 years (d) 48 years
4. Find the value of 'p' for which m  is a JOIN @iitwale in Telegram
3
10. Find two consecutive even numbers whose
root of the equation product is double that of the greater
pm2  ( 3  2)m  1  0 number.
(a) 3 (b) 2 (a) 1, 3 (b) 4, 6
(c) 2, 4 (d) 6, 8
(c) 6 (d) 7
11. The length and breadth of a rectangle are
5. For what respective values of 'm' and 'n' are (3k + 1) cm and (2k - 1) cm respectively.
2 5 Find the perimeter of the rectangle if its area
x and x  the roots of
5 3 is 144cm2 .
mx  nx  10  0 ?
2
(a) 50 cm (b) 10 cm
(a) 15,-19 (c) 32 cm (d) 25 cm
(b) -19, 15
(c) 19,-15 12. The sum of squares of two consecutive
(d) -15, 19 positive even numbers is 340. Find them.
(a) 12, 14 (b) 12, 10
6. The sides of two square plots are (2 x  1)m (c) 10, 8 (d) 14, 16
and (5x  4)m . The area of the second
square plot is 9 times the area of the first 13. Find two consecutive positive odd numbers,
square plot. Find the side of the larger plot. the sum of whose squares is 514.
(a) 15m (a) 11, 13
(b) 13m (b) 15, 17
(c) 31 m (c) 11, 9
(d) 39m (d) 13, 15
14. The area of a rectangular cardboard is 21. The quadratic equation ax2  bx  c  0 has
80 cm2 . If its perimeter is 36 cm, find its no real root. Which of the following is true?
length. (a) b2  4ac  0 (b) b2  4ac  0
(a) 40 cm (b) 10 cm (c) b2  4ac  0 (d) b2  4ac  0
(c) 20 cm (d) 8 cm
22. What is the nature of the roots of the
15. Find two consecutive integers whose quadratic equation 25x2  49  0 ?
product is 600. (a) Real and distinct (b) Real and equal
(a) 30, 20 (b) 50, 12 (c) Irrational (d) No real roots
(c) 15, 40 (d) 24, 25
23. When are the roots of a quadratic equation
16. Find the present age of a boy whose age 12 real and equal?
years from now will be the square of his (a) When the discriminant is positive.
present age. (b) When the discriminant is zero.
(a) 5 years (b) 7 years (c) When the discriminant is negative.
(c) 4 years (d) 6 years (d) When the discriminant is non-negative.
17. Identify the correct statement.
24. How are the roots of 3x2  7 x  8  0 ?
(a) The roots of the quadratic equation
(a) Real and unequal
9
2 y 2  9 y  0 are 0 and . (b) Real and equal
2 (c) Not real
(b) The value of 'k' for which (d) Cannot be determined.
4m2  k  15  0 has a root m = 3 is 7.
(c) The quadratic equation (4 x  11)2  0 has 25. What is theJOIN
value @iitwale inthe
of 'k' for which Telegram
roots of
two distinct roots. the quadratic equation 3x  2kx  27  0 are
2

(d) 7 x2  12 x  18  0 is not a quadratic real and equal?


equation. (a) 9 only (b) -9 only
(c) 9 or-9 (d) Neither 9 nor-9.
18. Find the roots of 3x2  2 6 x  2  0 .
26. Find the sum of the roots of
2 2 2 2
(a) , (b) , x2  x  210  0
3 3 3 3 (a) -2 (b) 29
2 3 2 3 (c) 20 (d) -1
(c) , (d) ,
3 2 3 3
27. In the quadratic equation 9 x2   x  2  0 ,
19. Divide 63 into two parts such that their 1
product is 962. find the value of a for which x  is its
3
(a) 24, 3C (b) 28, 35 solution.
(c) 26, 37 (d) 27, 36 (a) -2 (b) 3
(c) -4 (d) 6
20. Which of the following is a quadratic
equation? 28. The ratio of the length and breadth of a
5
(a) x   x 2 rectangular photo frame is 3 : 2. Find its
x length if its area is 864 cm2 .
2 (a) 34 cm (b) 26 cm
(b) x 2  2  1
x (c) 24 cm (d) 36 cm
(c) 2 x  3 x  4  0
2

(d) x2  1  2 x2  4

3
29. A two digit number is 4 times the sum of its 36. Find the value of 'k' for which x2  4 x  k  0
digits and also 16 more than the product of has coincident roots.
digits. Find the number. (a) 4 (b) -4
(a) 48 (b) 36 (c) 0 (d) -2
(c) 44 (d) 32
37. If the roots of x2  4mx  4m2  m  1  0 are
30. A quadratic equation  x  5x    0 has
2
real, which of the following is true?
1 (a) m  1 (b) m  1
two roots x  and x  2 . Find the (c) m  1 (d) m  0
3
respective values of  and  .
(a) 3, 2 (b) 2, -5 38. What is the ratio of the sum and the product
(c)-3, 5 (d) 3, -2 of roots of 7 x2  12 x  18  0
(a) 7:12 (b) 2:3
31. Find the common root of the equations (c) 3:2 (d) 7:18
x2  7 x  10  0 and x2  10 x  16  0 .
(a) - 2 (b) 3 39. Which of the following is the quadratic
(c) 5 (d) 2 equation one of whose roots is 3  2 3 ?
(a) x2  6 x  3  0 (b) x2  6 x  3  0
32. If the product of the roots of (c) x2  6 x  3  0 (d) x2  6 x  3  0
x2  3x  k  10 is - 2, what is the value of' k'?
(a) -2 (b) 8 40. If a and Rare the roots of the equation
(c) 12 (d)-8 x2  8x  p  0 such that  2   2  40 , find
the value JOIN
of 'p'. @iitwale in Telegram
33. If 2a 2  a  2  1 and a >0, find 'a'. (a) 8 (b) 10
(a)
3
(b) 1 (c) 12 (d) 14
2
(c) 3 (d) -1 41. Which of the following quadratic
polynomials can be factorized into a product
10 of real linear factors?
34. Find 'a' if a  3  .
a (a) 2 x2  5x  9 (b) 2 x2  4 x  5
(a) 5, 2 (c) 3x  4 x  6
2
(d) 5x2  3x  2
(b)  7,7
42. If  and  are the roots of the equation
(c) 7,7
(d) 5,2 x2  3x  2  0 , which of the following is the
equation whose roots are (  1) and (   1) ?
35. Find the value of 'p' so that x2  5 px  16  0 (a) x2  5x  6  0
has no real root. (b) x2  5x  6  0
8 (c) x2  5x  6  0
(a) Greater than (d) x2  5x  6  0
5
8
(b) Less than 43. Which of the following equations has 2 as a
5
root?
8 8
(c) Lies between and (a) 2 x2  7 x  6  0
5 5
(b) x2  4 x  5  0
15
(d) Less than (c) 3x2  6 x  2  0
8
(d) x2  3x  12  0

4
44. If the equation ax  5x  c  0 has 10 as the
sum of the roots and also as the product of
the roots, which of the following is true?
(a) a  c  5
(b) a  2,c  3
(c) a  5, c  1
(d) a  3, c  2

45. Find the product of the roots of the


quadratic equation 9m2  24 m  16  0 .
4 9
(a) (b)
3 16
16 3
(c) (d)
9 4

46. What is the nature of the roots of


3x 2  x  6  0 ?
(a) Real and equal
(b) Real and distinct
(c) Not real
(d) Cannot be determined.
JOIN @iitwale in Telegram
47. The perimeter and area of a rectangular
park are 80 m and 400 m2 . What is its
length?
(a) 20m (b) 15m
(c) 30m (d) 40m

48. If a and P are the roots of the equation


x2  kx  12  0 such that     1 , what is
the value of 'k'?
(a) 0 (b) ± 5
(c) ±1 (d) ± 7

49. What is the value of 'k' for which


2x2  kx  k has equal roots?
(a) 4 only (b) 0 only
(c) 8 only (d) 0, 8

50. Which of the following statements is true?


(a) x2  x  1  0 has no real roots.
(b) x2  4 x  3  0 and x2  x  2  0 have two
common roots.
(c) x2  3x  4  0 have real and equal roots.
(d) The roots of ax2  bx  c  0, a  0 are
reciprocal to each other if a  c .

5
Answer - Keys

1. A 2. D 3. B 4. C 5. A 6. D

7. A 8. C 9. B 10. C 11. A 12. A

13. B 14. B 15. D 16. C 17. A 18. B

19. C 20. D 21. A 22. A 23. B 24. C

25. C 26. D 27. B 28. D JOIN


29. @iitwale
A in
30.Telegram
D

31. D 32. B 33. B 34. A 35. C 36. A

37. B 38. B 39. B 40. C 41. B 42. D

43. A 44. A 45. C 46. C 47. A 48. D

49. D 50. A

6
Solutions
1. (a) A quadratic equation has a degree 2.
In (b) and (c), the degree of the polynomial 10. (c) Not available
is 3.
In (d), x 2  2 x  1 is not a polynomial as 11. (a) Not available
x  x1/2 the power of the variable is not an
12. (a) Not available
integer.
1
In (a), p   1  p 2  p  1  0 is a 13. (b) Not available
p
quadratic equation. 14. (b) Not available

2. (d) 2m2  3m  1  0 15. (d) Let the two consecutive integers be 'x'
 (m  1)(2m  1)  0 and x  1 .
1 According to the problem,
 m  1 or 
2 x( x  1)  600
 x  25 or 24
3. (b) Not available  x  1  24  1  25
 The required numbers are 24 and 25.
4. (c) Not available JOIN @iitwale in Telegram
16. (c) Let the present age of the boy be 'x'
5. (a) Not available years.
12 years from now, his age will be ( x  12)
6. (d) Not available years.
According to the problem, ( x  12)  x 2
7. (a) Not available
 ( x  4)( x  3)  0

8. (c) Not available  x  4 or  3


Since age cannot be negative, the required
9. (b) Let the present age of the son be 'x' present age of the boy is 4 years.
years. Then the father's age is x 2 years.
One year ago, the son's age was ( x  1) years 17. (a) Not available

and the father's age was ( x 2  1) years.


18. (b) Not available
According to the problem,
( x2  1)  8( x  1) 19. (c) Not available
 ( x  7)( x  1)  0
 x  1or 7 20. (d) Not available
If x  1, x2  1
 the father's age is 1 year is ridiculous. 21. (a) Not available
If x  7 ,then x 2  49
22. (a) b2  4ac  4(25)(49)  4900  0
Hence, the present age of the father is 49
years.

7
 The roots of the given quadratic equation 8 8
 p lies between and .
are real and distinct. 5 5

23. (b) When the value of the discriminant is 36. (a) Since the roots are coincident,
zero, the roots of quadratic equation are real b2  4ac  0
and equal.  (4)2  (1)(k )  0  k  4

24. (c) Not available


37. (b) Given, x2  4mx  4m2  m  1  0 have
real roots.
25. (c) Not available
 b2  4ac  0
26. (d) Not available  (4m2 )  4(1)(4m2  m  1)  0
 4m  4  0  m  1
27. (b) Not available
28. (d) Not available 38. (b) Given, 7 x2  12 x  18  0
b 12
Sum of roots  
29. (a) Let the digits in the tens place and the a 7
ones place be x and y respectively. Then, c 18
Product of roots  
according to the problem, a 7
10 x  y  4( x  y)  y  2 x 12 2
 The required ratio    2 : 3
and 10 x  y  xy  16 JOIN @iitwale 18 3 in Telegram
 x  4 or 2
If x  2 ,then y  2 x  4 39. (b) Let   3  2 3 and   3  2 3
The number is 24.     3 2 3 3 2 3  6
If x  4 then y  8 .
  (3)2  (2 3)2  9  12  3
 The number is 48.
 The required quadratic equation is
Hence, the required number is 48.
x2  6 x  3  0 .
30. (d) Not available
40. (c) The given equation is x2  8x  p  0
31. (d) Not available     8,  p
 2   2  40 (Given)
32. (b) Not available  (   )2  2  40
 p  12
33. (b) Not available

41. (b) Except for the equation in option (b) all


34. (a) Not available
options have discriminant < 0 and hence
cannot have real linear factors.
35. (c) Given x2  5 px  16  0 has no real root
 b2  4ac  0 42. (d) Given  and  are the roots of
 (5 p 2 )  4(1)(16)  0
x 2  3x  2  0 .
 p
8     3 and   2
5

8
 (  1)  (  1)      2
 3 2  5
(   )(  1)        1
 2  3 1  6
Hence, the required equation is
x2  5x  6  0

43. (a) Not available

44. (a) Not available

45. (c) Not available

46. (c) Not available

47. (a) Not available

48. (d) Given equation is x2  kx  12  0 .


b
    k and   12
a
    1 (Given)   1   JOIN @iitwale in Telegram
  (1   )  12
  2    12  0
   3, 4
If   3,  4 , and   4,  3
    7 or 7  k

49. (d) Not available

50. (a) Not available

9
Areas Related to Circles
1. When the circumference and area of a circle 8. The difference between circumference and
are numerically equal, find the numerical radius of a circle is 37 m. Find the
value of its diameter. circumference of the circle.
 (a) 7 m (b) 44 m
(a) (b) 8
2 (c) 154 m (d) 186 m
(c) 2 (d) 4
9. If the ratio of areas of two circles is 16:25,
2. The radius of a circle is 14 m. Find the what is the respective ratio of their
circumference of the circle. circumferences?
(a) 616m (b) 88m (a) 25:16 (b) 5:4
(c) 154m (d) 176m (c) 4:5 (d) 3:5
7
3. If the radius of a circle is cm , find the 10. If the ratio of circumference of two circles is

4:9, what is the ratio of their respective
area of the circle. areas?
(a) 154cm2 (a) 9:4 (b) 16:81
49 2 (c) 4:9 (d) 2:3
(b) cm

(c) 22 cm2 11. If the area of a circle is A, radius is 'r' and
(d) 49 cm2 circumference is C. which of the following
relations is true?
JOIN @iitwale C in r Telegram
4. If the circumference of a circle is
30
, what is (a) rC  2 A (b) 
 A 2
2
the diameter of the circle? r A
(c) AC  (d)  C
15 4 r
(a) 60 (b)

30 12. Find the area of the sector of a circle, whose
(c) (d) 30 radius is 6 m when the angle at the centre is
2
42 .
5. If the circumference of a circle is 44 m, what (a) 13.2m2 (b) 14.2m2
is the area of the circle? (c) 13.4m2 (d) 14.4m2
(a) 6084.5 m2 (b) 276.5 m2
(c) 154m 2
(d) 44m2 13. What is The area of a sector of a circle of
radius 16 cm cut off by an arc which is 18.5
6. A circular grass lawn of 35 m radius, has a cm long?
path 7 m wide running around it on the (a) 168cm2 (b) 148cm2
outside. Find the area of the path. (c) 154cm2 (d) 176cm2
(a) 1496m2 (b) 1450m2
(c) 1576m2 (d) 1694m2 14. Find the area of a segment of a circle of
radius 21 cm if the angle made by the arc of
7. How many plants will be there in a circular the segment has a measure of 60 .
bed whose outer edge measures 30 cm (a) 45.27 cm2
allowing 4 cm2 for each plant? (b) 40.27 cm2
(a) 18 (b) 750 (c) 40.8 cm2
(d) 24 (d) 120 (d) 44.27 cm2
15. A sector of 120 cut out from a circle has an   4 x2
(a) sq.units
3 4
area of 9 . What is the radius of the circle?
7 (  4) 2
(a) 3cm (b) 2.5 cm (b) x sq.units
4
(c) 3.5 cm (d) 3.6 cm  8 2
(c)   x sq.units
16. The length of the minute hand of a wall  4 
clock is 21 cm long. What is the area swept (d) 2(  4) x2 sq.units
by it in 10 minutes?
(a) 231 cm2 (b) 221 cm2 22. If a square of side 's' units is carved out from
(c) 210cm3 (d) 200cm2 a circle, find the ratio of the areas of two
figures.
(a) 2 :  (b)  : 2
17. The diameters of the wheels of a car are
(c)  : 4 (d) 4 : 
each 63 cm. Find the distance travelled by
the car when the wheels make 1000
revolutions. 23. The figure shows an isosceles triangle and a
semicircle with centre O.
(a) 1890 m (b) 1980 m
(c) 1900 m (d) 1800 m

18. The radius of a circle is 20 cm. Three more


concentric circles are drawn inside it in such
a manner that it is divided into four parts of Given that the radius of the semicircle is 2.8
equal area. Find the radius of one of the cm, find the perimeter of the given figure.
three concentric circles. JOIN @iitwale
(a) 15.6 cm (b) 18.8incmTelegram
(a) 8 3 cm (b) 2 3 cm (c) 16.8 cm (d) 20.4 cm
(c) 10 3 cm (d) 14 3 cm 24. The given figure shows two identical circles
and a rectangle.
19. A wire bent in the form of a circle of radius
42 cm is cut and again bent in the form of a
square. What is the ratio of the regions
enclosed by the circle and the square?
(a) 11 : 12 (b) 21 : 33
(c) 22 : 33 (d) 14 : 11 What is the unshaded area of the figure in
cm2 ? (Take   3.14 .)
20. What is the area of the largest triangle that
(a) 40 (b) 43
can be inscribed in a semicircle whose
(c) 25 (d) 33
radius is 'r' cm?
(a) 2r cm2 (b) r 2 cm2 25. By The shaded part of the given figure
(c) 2r 2 cm2
r
(d) cm 2 shows a running track in a stadium.
2

21. The top of a dining table is rectangular, 2x


units long and x units wide, with two
semicircles along the breadth. Find the area
of the table. Find the area of the track.
22
(Take   .)
7

3
(a) 2058m2 Find the area of the remaining piece of
(b) 1546m2 22
paper. (Use   .)
7
(c) 3514m2
(a) 296.1 cm2 (b) 265.4 cm2
(d) 1923m2
(c) 221.5cm2 (d) 201.7cm2

26. The figure shows a right-angled triangle and


31. Mrs. Vidya bought a piece of cloth as shown
a semicircle. PQ is the diameter of the
in the figure. The portion of the cloth that is
semicircle.
not coloured consists of 6 identical
semicircles.

Find the perimeter of the whole figure. Find the area of the coloured portion.
(a) 13  8 (b) 30  5 (a) 144 cm2 (b) 126 cm2
(c) 18  6 (d) 18  10 (c) 195cm2 (d) 243 cm2

27. The outer diameter and the inner diameter 32. The given figure shows a plot of land, which
of a circular path are 728m and 700 m is made up of 2 semicircles, a rectangle and
respectively. Find the area of the 22 circular an isosceles triangle. The diameter of the
22 bigger semicircle is 7 m longer than that of
path. (Use   .)
7 22
the smallerJOIN (Use   in Telegram
@iitwale
semicircle. .)
(a) 45260m2 (b) 25012m2 7
(c) 31416m2 (d) 19541 m2

28. If the circumference of a circle is increased


by 50%, by what percent will its area be
increased?
(a) 75% (b) 100%
(c) 125% (d) 150% If the shaded region is covered with grass,
find the area of the land covered with grass.
29. Each wheel of a car makes 5 revolutions per (a) 502 m2 (b) 497 m2
second. If the diameter of a wheel is 84 cm, (c) 433 m 2
(d) 564 m2
find the speed of the car in km/h. (Give your
answer correct to the nearest km.) 33. The figure given is made up of a rectangle,
(a) 48 km / h (b) 32 km / h 2 identical semicircles and a quadrant.
(c) 41 km / h (d) 25 km / h

30. The figure given shows two identical


semicircles cut out from a piece of coloured
paper.
Find the unshaded area of the figure.
22
(Use   .)
7
(a) 1350cm2 (b) 1154cm2
(c) 1400cm2 (d) 1260cm2

4
34. In the given figure, a circle with centre B (Use   3.14 )
overlaps another circle with centre A and a (a) 383.68 cm2 (b) 197.59 cm2
square. The ratio of areas of P and Q is 5 :4 (c) 173.45 cm2 (d) 242.52 cm2
1
and the area of Q is the area of circle B.
8 38. The figure given shows two identical
The radii of circle A and circle B are 10 cm 22
and 8 cm respectively. semicircles inside a square. (Use   .)
7

Find the area of the unshaded part of the


figure. (Take   3.14 .) What is the shaded area of the region?
(a) 449.75 cm2 (b) 520.60 cm2 (a) 15cm2 (b) 21 cm2
(c) 563.72 cm2 (d) 450.92 cm2 (c) 16cm2 (d) 23cm2

35. In the given figure, O is the centre of the 39. A regular hexagon is inscribed in a circle of
circle whose diameter is 14 cm. radius 14 cm. Find the area of the circle
falling outside the hexagon.
(a) 106.79 cm2 (b) 241.8 cm2
(c) 79.27 cm 2
(d) 173.9 cm2

40. Kill In the JOIN


given figure, ABC is ainright-angled
@iitwale Telegram
Find the perimeter of the figure.
triangle in which ABC  90 , AB = 6 cm
22
(Use   .) and BC = 8 cm. 0 is the centre of the in
7 circle.
(a) 134cm (b) 124cm
(c) 112cm (d) 160cm

36. A bucket is pulled from a well by means of a


rope which is wound round a wheel of
diameter 77 cm. Given that the bucket Find the area of shaded region.
ascends in 1 minute 28 seconds with a 22
uniform speed of 1.1 m/s, find the number (Use   .)
7
of complete revolutions that the wheel
(a) 12.56cm2 (b) 11.42cm2
22
makes in raising the bucket. (Take   .) (c) 13.65cm2 (d) 10.57cm2
7
(a) 10 (b) 55
(c) 25 (d) 40 41. Arjun drew a figure as shown in figure,
where a circle is divided into 18 equal parts.
37. David cut out the given figure during one of He then shaded some of the parts. (Take
his art classes. The figure is made up of   3.14 .)
rectangles and quadrants.

If David wanted to colour the figure red.


Find the total area that he needs to colour.

5
Find the total area that Arjun shaded. 45. The figure given is made up of a circle and
(a) 25.12 cm2 (b) 29.25 cm2 3 identical semicircles. 0 is the centre of the
(c) 36.4 cm 2
(d) 45.2 cm2 circle and XY is the diameter of the circle.

42. In the adjoining figure, ABC is an equilateral


triangle of side 14 cm. M is the centre of the
circumcircle.

Given that XY is 28 cm, find the perimeter


of the shaded part of the figure.
22
(Use   .)
7
(a) 67 cm (b) 50 cm
(c) 80 cm (d) 15 cm
Find the area of the shaded region.
(a) 115.27 cm2 (b) 96.63 cm2 46. The figure given shows a rectangle with a
(c) 120.46 cm 2
(d) 146.72 cm2 semicircle and 2 identical quadrants inside
it.
43. 4 identical semicircles are drawn inside a big
square as shown. Each side of the big
square is 14cm long.

JOIN @iitwale in Telegram


What is the shaded area of the figure?
22
(Use   .)
7
Find the area of the shaded region. (a) 363 cm2 (b) 259 cm2
22 (c) 305cm2 (d) 216cm2
(Use   .)
7
(a) 125cm2 (b) 112 cm2 47. The minute hand of a clock is 7 cm long.
Find the area traced out by the minute hand
(c) 173cm2 (d) 159 cm2
of the clock between 4:15 p.m. and 4:35
p.m. on a day.
44. The figure given is made up of a square with (a) 59 cm2 (b) 65 cm2
a circle inscribed in it.
(c) 52 cm 2
(d) 45 cm2

48. A square shaped bus shelter is supported on


four circular poles-The circumference of
each pole is 'x' m and the length of each
side of the shelter is 'y' m. Find the area of
What is the area of the shaded region? the unsupported part of the shelter.
(Take   3.14 .)  y2   x2 
(a)  x 2   m2 (b)  y 2   m 2
(a) 6.28 cm2  p  p
(b) 1.42 cm2  y2   x2 
(c) 4.91 cm2 (c)  x 2   m2 (d)  y 2   m2
 p  p
(d) 7.36 cm2

6
49. The circumference of a circle exceeds its (c) 42 (d) 40
diameter by 30 cm. Find the radius of the
circle. 53. If the length of the arc of a circle having a
(a) 7 cm (b) 14 cm central angle 36° is 22 cm, what is the area
(c) 21 cm (d) 28 cm of the circle?
(a) 3580 cm2
50. The given figure shows 2 identical circles (b) 3850 cm2
inside a rectangle in cm2 . (c) 3058 cm2
(d) 3805 cm2

54. If the perimeter of an arc of a circle is 43 cm


Find the shaded area of the figure. and the radius of the circle is 21 cm, find the
(Take   3.14 .) angle subtended at the centre.
(a) 1300 (a) 3.72 (b) 2.73
(b) 1154 (c) 2.37 (d) 3.27
(c) 1092
(d) 1256 55. The given figure is made up of a rectangle
and 2 identical circles. Given that the length
51. A copper wire, when bent in the form of a of the rectangle is 28 cm.
square, encloses an area of 484 cm2 . If the
same wire is bent in the form of a circle, find
the area in cm2 .
(a) 606 (b) 661 JOIN
What is the area of@iitwale
the shaded in Telegram
region?
(c) 616 (d) 610
22
(Use   .)
52. Four equal circles are described about the 7
four corners of a square so that each of (a) 119 cm2 (b) 105 cm2
them touches two of the others. If each side (c) 152cm2 (d) 123 cm2
of the square measures 14 cm, find the area
of the remaining portion of the square apart
from four circles in cm2 .
(a) 20 (b) 24

7
Answer - Keys

1. D 2. B 3. D 4. C 5. C 6. D

7. A 8. B 9. C 10. B 11. A 12. A

13. B 14. B 15. A 16. A 17. B 18. C

19. D 20. B 21. C 22. A 23. B 24. B

25. A 26. C 27. C 28. C JOIN


29. @iitwale
A in
30.Telegram
C

31. B 32. B 33. D 34. D 35. A 36. D

37. A 38. B 39. A 40. B 41. A 42. C

43. B 44. A 45. C 46. B 47. C 48. D

49. A 50. D 51. C 52. C 53. B 54. B

55. A

8
Solutions
1. (d) Not available 10. (b) Not available

2. (b) Not available 11. (a) Not available

3. (d) Not available 12. (a) Not available


13. (b) Not available
4. (c) Not available
14. (b)
5. (c) Not available

6. (d)

x
Area of sector OAB   r2
360o
60o 22
   21 21  231cm2
360o 7
r  35m 3 2
R  35  7  42 m  Area of OAB  r  190.73 cm 2
4
Area of circular path  Area of JOIN
sector =@iitwale
231 - 190.73
in Telegram
  ( R  r )( R  r )  1694m2  40.27 cm2

7. (a) Circumference = 30 cm 15. (a) Area of sector


C 2
30  30 x 3
Area    71.6 cm2    r 2  9 (Given)
4 4  22 360 o
7
7 120o 22 2 66
A 71.6   r 
Number of plants   360o 7 7
4 4 66 360o 7
 r2    9
7 120o 22
 17.9  18
 r  9  3 cm
8. (b) 2 r  r  37
37 37
r   7m 16. (a) The angle swept by the minute hand in
2  1 2  22  1
7 60 minutes  360
Circumference  2 r  44 m  The angle swept by the minute hand in 10
o
 360 
minutes   10   60o
9. (c) We have,  60 
A1 : A2  16 : 25  Sector angle  60
C1 A1 16 4 The area swept by the minute hand in 10
   minutes = The area of a sector of a circle of
C2 A2 25 5
radius 21 cm and sector angle 60 .
The ratio of circumferences is 4 : 5.
60
o
 r2
360

9
60 22 Area of shaded region
   21 21cm2  231cm2
360o 7  2  3.14  5cm  5 cm  157 cm 2

Breadth of rectangle = Diameter of circle


17. (b) One revolution made by the wheel of  5 cm  2  10 cm
the car is equal to the circumference of the Area of rectangle = Diameter of circle
wheel.  20 cm 10 cm  200 cm2
Given diameter of the wheel,  Area of the unshaded region
d = 63 cm  200 cm2 157 cm2  43 cm2
d 63 Hence, the unshaded area of the figure is
 The radius, r   cm
2 2 43 cm2 .
Circumference, C  2 r
22 63 25. (a) Not available
 2    22  9  198 cm
7 2
 1.98 m 26. (c) Not available
 The distance travelled by the car in one 27. (c) Not available
revolution = 1.98 m
The distance travelled by the car in 1000 28. (c) Not available
revolutions
 1.98 1000 m  1980 m 29. (a) Not available

18. (c) Not available 30. JOIN


(c) Not available @iitwale in Telegram
19. (d) Not available 31. (b) Not available

20. (b) Not available 32. (b) Not available

21. (c) Not available 33. (d)

22. (a) Not available

23. (b)
1 22
Area of quadrant    42  42
4 7
 1386 cm2
Area of rectangle  42  (42  21)  2646 cm2
Circumference of circle  2 r
 Unshaded area
22
 2   2.8  17.6 cm  2846 cm2 1386 cm2  1260 cm2
7
Circumference of semicircle = 8.8 cm
34. (d) Not available
Perimeter of the given figure = (5 + 5 +
8.8) = 18.8 cm
35. (a) Radius of circle = 14 cm - 2 = 7 cm
One side of the figure opposite to 35 cm =
24. (b) Radius of each circle  20 cm  4  5 cm
35 cm - 7 cm = 28 cm

10
Perimeter of the two sectors of circle 1
Area of A  2m  2 cm   3.14
1 22 4
  14 cm  22 cm
2 7 2 cm  2cm  0.86 cm2
Total perimeter = 134 cm 1
Unshaded area  0.86 cm2  2  cm
The perimeter of the given figure is 134 cm. 2
4cm  4cm  9.72 cm 2

36. (d) Not available


Shaded area  4 cm  4 cm  9.72 cm2

37. (a) Not available  6.28 cm2

38. (b) Not available 45. (c) Radius of circle  28 cm  2  14 cm


22
Area of circle  14cm 14 cm
39. (a) Not available 7
 616 cm2
40. (b) Not available Radius of semicircle  14 cm  2  7 cm
41. (a) Not available 1 22
 ShadedJOIN
area  3@iitwale
   7 in 7 Telegram
231cm2
42. (c) Not available 2 7
 616 cm2  231 cm2  385 cm2
43. (b) Radius =14 cm  2  7 cm Perimeter of shaded part of figure
 1 22 
 7  7  3    14 
2 7 
 14 cm  66 cm  80 cm

1 22 46. (b) Area of rectangle  28 cm  23 cm


Area of a    7 cm  7cm
4 7
 644 cm2
 38.5cm2
Radius of semicircle  28 cm  2  14 cm
1 Radius of quadrant  23cm 16cm  7cm
Area of a   7 cm  7 cm
2 Area of unshaded region
 24.5 cm2 1 22
 (  14cm 14cm) 
2 7
Area of a  38.5 cm2  24.5 cm2 1 22
(2    7cm  7cm)  385 cm2
 14cm2 4 7
Area of shaded region  14 cm2  8 Shaded area  644 cm2  385 cm2  259 cm2
 112 cm2
47. (c) Not available
44. (a)
48. (d) Not available

11
49. (a) Not available

50. (d) Not available

51. (c) Not available

52. (c) Not available

53. (b) Not available

54. (b) Not available

55. (a) Combine the shaded parts in the given


figure as in the following figure.

Radius of circle  28 cm  4  7 cm
1 22
Area of quadrant    7  7  38.5 cm2
4 7
Area of shaded region
=(14 14)  38.5  38.5
JOIN @iitwale in Telegram
 196 cm2  77 cm2  119 cm2

12
Statistics

Statistics
1. Find the weighted mean of first 'n' natural 7. The mean age of a combined group of men
numbers, whose weights are and women is 25 years. If the mean age of
proportional to the corresponding numbers. the group of men is 26 years and that of the
2n  1 group of women is 21 years, find the
(a)
3 percentage of men and women respectively
n 1 in the group.
(b) (a) 70, 30
2
n(n  1) (b) 30, 70
(c) (c) 80, 20
2
(d) 60, 40
(n  1)(2n  1)
(d)
6 8. The mean of 8 numbers is 25. If 5 is
subtracted from each number, what will the
2. What is the mean of -8,-4, 4 and 8? new mean be?
(a) 1 (a) 20 (b) 15
(b) Zero (c) 160 (d) 2
(c) 8
(d) 2 9. If the mean of first 'n' odd natural numbers
is 'n' itself, what is the value of 'n'?
3. Find the mean of the first six multiples of 3. (a) 2
(a) 63 (b) 3
(b) 11.5 (c) 1 JOIN @iitwale in Telegram
(c) 10.5 (d) Any natural number
(d) 60
10. If the mean of 'n' observations,
4. The A.M. of a set of 50 numbers is 38. If 46n
two numbers of the set, namely 55 and 45 12 ,22 ,32 ,....,n 2 is , find the value of 'n'.
11
are discarded, find the mean of the
(a) 22 (b) 23
remaining observations.
(c) 12 (d) 11
(a) 36
(b) 37.5
11. The A.M. of 'n' observations is M. If the sum
(c) 36.5
of (n-4) observations is 'a', what is the mean
(d) 38.5
of remaining 4 observations?
nM  a
5. Find the mean of the first six prime (a) nM  a (b)
numbers. 2
(a) 68.3 (b) 6.71 nM  a nM  a
(c) (d)
(c) 7 (d) 6.83 2 4

6. The A.M. of 'n' numbers of a series is X. If 12. Identify the mode of the given distribution.
the sum of first (n - 1) terms is 'k'. what is the Marks 4 5 6 7 8
nth number? Number of Students 3 5 10 6 1
(a) nX  nk (a) 7
(b) nX  k (b) 1
(c) 8
(c) X  nk
(d) 6
(d) X  k
13. The hearts of 60 patients were examined 18. The mean weight of seven boys is 56 kg.
through X-ray and observations obtained Individual weights of six boys (in kg) are
are recorded. 52,57,55,60,59 and 55 respectively. Find
Length of the weight of the seventh boy.
No. of
heart (in (a) 52 kg
patients
mm) (b) 54 kg
120 7 (c) 57 kg
121 9 (d) 59 kg
122 15
123 12 19. The mean of 150 items was found to be 60.
124 6 Later on, it was discovered that the values
of two items were taken as 52 and 8 instead
125 11
of 152 and 88 respectively. Find the correct
Find their median. mean.
(a) 6.15 (b) 60.7
(a) 144 (b) 122 (c) 61.2 (d) 6.72
(c) 156 (d) 114
20. Given that x is the mean of x1 , x2 ,....xn , find
14. Marks obtained by 12 students in a test are
x x x
37, 23, 16, 19, 34, 23, 5, 27, 36, 23, 20 the mean of 1 , 2 ,.... n .
and 38. Find the modal marks. a a a
(a) 27 (b) 37 (a) x  a (b) x
(c) 23 (d) 5 x
(c) a x (d)
a
15. If 10, 13,15,18,( x  1),( x  3),30,32,35 and 41 21.
JOIN @iitwale in Telegram
In the frequency distribution, discrete data is
are ten observations in the ascending order given.
with median 24, find the value of 'x'. Variable ( x) 0 1 2 3 4 5
(a) 42 Frequency x 20 40 40 20 4
(b) 22 (f)
(c) 10
(d) 32 If the mean is 2.5, what is the value of
frequency ' x ' ?
16. The mean of six numbers is 23. If one of the (a) 0 (b) 1
numbers is excluded, the mean of the (c) 3 (d) 4
remaining numbers is 20. Find the excluded
number. 22. If x1 , x2 ,.....xn are 'n' observations such that
(a) 138 n n
(b) 100  ( xi  3)  120 and  ( x  5)  160
i , find
(c) 20 1 1

(d) 38 'n'.
(a) 40 (b) 20
17. If x is the mean of x1 , x2 ,....xn , find the mean (c) 60 (d) 70
of ( x1  2a),( x2  2a),( x3  2a),.....( xn  2a) .
23. The arithmetic mean of the scores of a
(a) x  2a
group of students in a test was 52%. The
(b) x  2a
brightest 20% of them secured a mean score
x
(c)  2a of 80 and the dullest 25%, a mean score of
2 31 %. Which of the following is the mean
x score of the remaining 55%?
(d)  2a
2 (a) 45% (b) 50%
(c) 51.4%(approx) (d) 54.6% (approx)

3
24. The mean of 10 numbers is 7. If each 30. What is the mean of the first 'n' natural
number is multiplied by 12, find the mean numbers?
of new set of numbers. 1
(a) n  (b) n(n  1)
(a) 82 (b) 48 2
(c) 78 (d) 84 1 (n  1)
(c) (n  1) (d)
2 2n
25. The mean of the marks in Statistics of 100
students in class X was 72. The mean of 31. What is the arithmetic mean of 20 fours, 40
marks for boys was 75, while their number fives, 30 sixes and 10 tens?
was 70. What is the mean of marks of girls (a) 50 (b) 25
in the class? (c) 5.6 (d) 33
(a) 35 (b) 65
(c) 68 (d) 86 32. Find the value of ' x ' if the mean of
x  2,2 x  3, 3x  4 and 4 x  5 is x  2 .
26. The given data are the times (in minutes), it
(a) 2 (b) 1
takes seven students to go to school from
(c) 3 (d) -1
their homes.
11 6 22 7 10 6 15
33. What is the value of 'n' if the mean of first 9
Which statement about the data is false?
5n
(a) Their median is 11. natural numbers is ?
(b) Their mean is 11. 9
(c) Their range is 16. (a) 7
(d) Their mode is 6. (b) 8
(c) 9
(d) 11 JOIN @iitwale in Telegram
27. From a series of 50 observations, an
observation 45 is dropped, but the mean
remains the same. What was the mean of 50 34. Which of the following is true about the
observations? mode of a given data?
(a) 50 (b) 49 (a) It may or may not exist for a given data.
(c) 45 (d) 40 (b) It is always unique.
(c) It is very difficult to compute mode.
28. A person made 165 telephone calls in the (d) We cannot calculate mode without the
month of May in a year. It was Friday on 1st empirical formula.
May of the year. The average of telephone
calls on Sundays of the month was 7. What 35. The A.M. of 12 observations is 15. If an
was the average of the telephone calls per observation 20 is removed, what is the
day on the rest of the days of the month? arithmetic mean of the remaining
165 observations?
(a) (b) 5 (a) 14.5 (b) 13
31
(c) 15 (d) 13.5
137
(c) 7 (d)
27 36. The mean weight of a group of 10 students
is 25 kg and the mean weight of another
29. The numbers 4 and 9 have frequencies x group of 10 students is 35 kg. What is the
and ( x  1) respectively. If their arithmetic mean weight of all the 20 students?
mean is 6, what is the value of ' x ' ? (a) 30 kg
(a) 2 (b) 35 kg
(b) 3 (c) 25 kg
(c) 4 (d) 20 kg
(d) 5

4
37. The mean of the scores x1 , x2 ,....., x6 is x . 44. What is the mode of the data 3,6,3,4,6,4,
What is the mean of the scores 3,5,6,5, x and x 2 ?
5x1,5 x2 ,.....,5x6 ? (a) 4 or 5 only (b) 3 or 6 only
x (c) 3 or 5 only (d) 3, 4 or 6
(a) x  5 (b)
5
(c) x  5 (d) 5x 1 2 3 1 7
45. What is the median of , , , and ?
2 3 4 6 12
x 3 7
38. Find where 6 is the median of the scores (a) (b)
6 4 12
x x x x x 2 1
, , , and . (c) (d)
2 3 4 5 6 3 6
(a) 12 (b) 4
(c) 24 (d) 6 46. What is the median of the first 100 natural
numbers?
39. The mode of a data exceeds its mean by 12. (a) 50.5 (b) 50
By how much does its mode exceed the (c) 52 (d) 51
median?
(a) 8 (b) 12 47. If the difference of mode and median of a
(c) 0 (d) 10 data is 24, what is the difference of median
and mean?
40. In a class of 19 students, 7 boys failed in a (a) 24 (b) 6
math test. The scores of those who passed (c) 12 (d) 30
are 12,15,17,15,16,15,19,19,17,18,18 and
19 marks. What is the median marks of the 48. JOIN
What is the @iitwale
arithmetic mean ofin
30Telegram
20 32 16
19 students in the class? and 27?
(a) 15 (b) 12 (a) 23 (b) 24
(c) 16 (d) 19 (c) 25 (d) 26

x x x x 49. Find the mode of 32, 20, 32, 16, 27 and 32.
41. If the median of , x, , and (where (a) 20 (b) 27
5 4 2 3
(c) 30 (d) 32
x  0 ) is 8, what is the value of ' x ' ?
(a) 6 (b) 8
2 1
(c) 4 (d) 24 50. Find the median of 15 .15.03,15,15 and
3 3
42. The mode of the observations 5, 4, 4, 3, 5, 15.3.
x , 3, 4, 3, 5, 4, 3, and 5 is 3. What is their (a) 15
1
(b) 15.3
median? 3
(a) 3 (b) 4 2
(c) 15 (d) 15.03
(c) 5 (d) 4.5 3

43. If the ratio of mean and median of a certain 51. If for a given data median is 125.6 and
data is 2:3, what is the ratio of its mode and mean is 128, find mode.
mean? (a) 120.8 (b) 128.0
(a) 3 : 2 (c) 108.2 (d) 180.2
(b) 5 : 2
(c) 3 : 5 52. what is the arithmetic mean of a  2 , a and
(d) 2 : 3 a2?
(a) a  2 (b) a
(c) a  2 (d) 3a

5
53. The mean of 9, 11, 13, p, 18 and 19 is p. (c) 14 (d) 8
Find the value of 'p'.
(a) 12 (b) 13 55. Which of the following is calculated using
(c) 14 (d) 15 mid-values of classes?
(a) Mean (b) Median
54. What is the mode of 10, 2, 8, 6, 7, 8, 9, 10, (c) Mode (d) Range
10, 11 and10?
(a) 10 (b) 12

JOIN @iitwale in Telegram

6
Answer - Keys

1. A 2. B 3. C 4. B 5. D 6. B

7. C 8. A 9. D 10. D 11. D 12. D

13. B 14. C 15. B 16. D 17. A 18. B

19. C 20. D 21. D 22. B 23. C 24. D

25. B 26. A 27. C 28. B JOIN


29. @iitwale
B in
30.Telegram
C

31. C 32. D 33. C 34. A 35. A 36. A

37. D 38. B 39. A 40. A 41. D 42. B

43. B 44. C 45. B 46. A 47. C 48. C

49. D 50. B 51. A 52. B 53. C 54. A

55. A

7
Solutions
1. (a) Not available 15. (b) Not available

2. (b) Not available 16. (d) Not available

3. (c) Not available 17. (a) Not available


18. (b) Not available
4. (b) Not available
19. (c) Not available
5. (d) Not available
20. (d) Not available
6. (b) Let the numbers be
x1 , x2 , x3.....xn .Then 21. (d) Not available
1 n
X  X1
n i 1 22. (b) Not available
x1  x2  x3  .....  xn1  xn
 X 23. (c) Not available
n
k  xn
 [ x1  x2  x3  .....  xn1  k ] 24. (d) Total of 10 numbers
n
 10  7  70
JOIN @iitwale in Telegram
 xn  cX  k
If each number is multiplied by 12, New
total  70 12
7. (c) Not available 70 12
 New mean   84
10
8. (a) Not available
25. (b) Mean marks of girls
9. (d) Not available Total marks of all students

10. (d) Not available -Total marks of boys



Total number of girls
11. (d) Not available
1950
  65
30
12. (d) Mode is 6 as it has the highest
frequency.
26. (a) Arranging the given data in ascending
order, we get, 6, 6, 7, 10, 11, 15, 22
13. (b) Not available
6  6  7  10  11  15  22
mean 
7
14. (c) The given data arranged in ascending
order is 5,16,19,20,23,23,23,27, 34, 36, 77
  11
37,38. 7
By observation, we find that 23 occurs the Mode = 6
most number of times. Median = 4th value = 10
Mode = 23 or Modal marks = 23 Range = 22 - 6 = 16

8
27. (c) Not available 40. (a) Not available

28. (b) Not available 41. (d) Not available

29. (b) Not available 42. (b) Not available

30. (c) Not available 43. (b) Not available

31. (c) Not available 44. (d) Not available

32. (d) Given that the mean of x  2 , 45. (b) Not available
2 x  3,3x  4 and 4x + 5 is x  2
10 x  14 46. (a) Not available
  x2
4
 6 x  6  x  1 47. (c) Not available

33. (c) Mean of first 9 natural numbers 48. (c) Not available
1  2  .....  9 45
  5 49. (d) Not available
9 9
5n
Given mean of first 9 natural numbers is . 50. (b) Not available
9 JOIN @iitwale in Telegram
5n 95
 5  n 9 51. (a) Given median = 125.6 and mean
9 5
=128.
34. (a) Mode of a given data may or may not Mode = 3 Median - 2 Mean
 (3 125.6)  (2 128)
exist sometimes.
 376.8  256
35. (a) The A.M. of 12 observations is 15.  120.8
 Sum of 12 observations
 12 15  180 a  2  a  a  2 3a
52. (b) Mean   a
An observation 20 is removed 3 3
 Mean of the remaining observations
180  20 160 53. (c) Given mean = p
   14.5 9  11  16  p  18  19
(12  1) 11  p
6
 p  14
36. (a) Not available

37. (d) Not available 54. (a) Mode = Observation with the highest
frequency =10
38. (b) Not available
55. (a) Mean is calculated using the mid-values
39. (a) Not available of classes.

9
Triangles
1. In the figure, ABC  DAC  90 , BC = 3 5. ABCD is a parallelogram in which EF is
cm, AD = 2 cm and AB = 1 cm. What is drawn parallel to BC such that
the measure of AC + CD? AE  (4 x  3)cm , AF  (8x  7)cm ,
BE  (3x  1)cm and CF (5x  3)cm .

What is the value of 'x'?


(a) 1 (b) -1
(a) 2( 5  7)cm 1
(c) -5 (d)
(b) 5( 2  7)cm 2
(c) 2( 7  5)cm
AD AE
(d) 7( 2  2)cm 6. In ABC , if  and ADE  ACB ,
DB EC
what type of triangle is ABC ?
2. A paper is cut in the following shape. (a) Right triangle
(b) Acute angled triangle
(c) Isosceles triangle
(d) ObtuseJOIN
angled@iitwale
triangle in Telegram

7. The altitudes of two similar triangles are 4


cm and 6 cm. If the area of one triangle is
Which of the following is the correct 36 cm2 , what is the area of the other?
statement? (a) 16cm2 (b) 36cm2
(a) AE 2  AD2  ED2 (c) 49 cm2 (d) 25 cm2
(b) AC 2  AD2  DE 2
(c) AC 2  AB2  AD2 8. The ratio of areas of two similar triangles is
(d) AE 2  AB2  BC 2 81 :49. If the median of one triangle is 4.9
cm, what is the median of the other?
3. In the given figure what is the value of ' x ' if (a) 4.9 cm (b) 6.3 cm
AB || CD ? (c) 7 cm (d) 9 cm

9. The perimeters of two similar triangles ABC


and DEF are 24 cm and 32 cm respectively.
If DE = 12 cm, find AB.
(a) 9cm (b) 35cm
(c) 18cm (d) 28cm
(a) 2 units (b) 4 units
(c) 3 units (d) 5 units
10. BC and EF are the corresponding sides of
two similar triangles ABC and DEF. If BC =
AD 3
4. In ABC , DE || BC and  . What is the 9.1 cm, EF = 6.5 cm and the perimeter of
DB 5 AEF is 35 cm, find the perimeter of ABC .
length of AE if AC  4.8 cm ? (a) 15cm (b) 49cm
(a) 1.4cm (b) 1.8cm (c) 45 cm (d) 35 cm
(c) 2.2 cm (d) 3.6 cm
11. A man cycles 15 m towards east, turns right 18. In similar triangles ABC and FDE .DE =
and cycles 8 m. How far is he from the 4 cm, BC = 8 cm and area of
starting point? FDE  25 cm2 . What is the area of ABC ?
(a) 7m (b) 17m (a) 144cm2 (b) 121 cm2
(c) 23m (d) 19m
(c) 100cm2 (d) 81 cm2
12. At what height does the tip of a 34m long
ladder placed at a distance of 16 m from a AO BO 1
19. Given   and AB  8 cm ,find DC
wall, touch the wall? OC OD 4
(a) 29 m (b) 34 m
(c) 30m (d)18 m

13. At some point of time on a summer


evening, an 8 m tall flag post casts a 15 m
long shadow. What is the distance between
the tips of the flag post and its image?
(a) 15m (b) 28m (a) 10cm (b) 32cm
(c) 23m (d) 17m (c) 6 cm (d) 2 cm

14. What is the area of a square of diagonal 20. Given that PB  AB and QA  AB PO=4
12cm? cm and QO  7cm , if area of QAO is
(a) 36cm2 (b) 9cm2 245 cm2 , what is the area of PBO ?
(c) 72cm 2
(d) 144 cm 2
(a) 60 cm2 (b) 40 cm2
(c) 125cmJOIN @iitwale
2
(d) 80cmin2 Telegram
15. Two 15 m strings are tied to a peg between
two poles 9 m and 12 m long from their 21. A girl of height 90 cm is walking away from
types. What is the distance between the the base of a lamp post at a speed of 1.2
poles? m/sec. If the lamp is 3.6 m above the
(a) 18m (b) 21 m ground, find the length of the girls shadow
(c) 20 m (d) 23 m after 4 seconds.
(a) 4.8m (b) 1.2m
16. The areas of two similar triangles are 81 (c) 1.6m (d) 3.6m
cm2 and 49 cm2 respectively. If the altitude
of the bigger triangle is 4.5 cm, find the AD CE
corresponding altitude of the smaller 22. In the given figure, PD || BC and 
DC BE
triangle.
. Identify the correct statement.
(a) 3 cm (b) 2.5 cm
(c) 4 cm (d) 3.5 cm

17. X and Y are points on the sides AB and AC


respectively of ABC

(a) AD  DC
If AB = 5.6 cm, AX = 1.4 cm, AC = 7.2 (b) PE | | AC
cm, and XY || BC , what is the measure of (c) PB  CE
AY? (d) PD  AC
(a) 2.4 cm (b) 1.6cm
(c) 1.8 cm (d) 3.6cm

3
23. In the given figure, AB  BC , DE  AC 28. In the given figure, ABE~DCE .
and
GF  BC .

Find the difference between ABE and


 AEB .
(a) 130 (b) 15
(c) 35 (d) 115

Which of the following is correct? 29. In ABC, B  90 and AD  CB


(a) GFC  DAE (produced). Identify the correct statement.
(b) ADE GCF (a) AC 2  AB2  BC 2  2BC. AD
(c) GCF  ABC (b) AC 2  AB2  BC 2  2BC. AB
(d) AED EGF
(c) AB2  AC 2  BC 2  2BC.BD
24. Lf ABC ~ DEF , BC=3cm,EF  4cm and
30. In the figure, DE | | BC . Find AE.
area of ABC  54cm2 , find the area of
DEF .
(a) 96m2 (b) 16cm2
(c) 96cm2 (d) 69cm2
JOIN @iitwale in Telegram
25. In ABC, A  90 , AN  BC , BC = 12
cm and AC = 5 cm. Find the ratio
(a) 6 cm (b) 4 cm
ar (ANC ) : ar (ABC )
(c) 5 cm (d) 3 cm
(a) 122 : 52 (b) 32 :122
(c) 52 :122 (d) 32 : 52 31. Two triangles BAC and BDC, right angled at
A and D respectively, are drawn on the
26. In trapezium ABCD, AB || CD . If OA  x  4 , same base BC and on the same side of BC.
OB  3x 19, OC  4 and OD  x  3 , find If AC and DB intersect at P, which of the
'x'. following statements is true?
(a) 10 units (b) 9 units (a) AP  PC  DP  PB
(c) 12 units (d) 8 units (b)
AP DP

PC PB
27. In the given figure, if ABE  ACD , (c) AP  PB  DP  PC
identify the correct statement. (d) Both (b) and (c)

32. In ABC , D is a point on AB and E is a


point on BC such that DE | | AC and area of
1 AD
DBE  area of ABC . Find .
2 AB
1 2 2 1
(a) ADE~BED (a) (b)
2 2
(b) ABC~ADE
(c) DEC ~ ABC 2 1 2 1
(c) (d)
(d) BDE ~ BAE 2 2

4
33. In the given figure, RSP~RPQ .Identify 37. In the given figure, ABC is right angled at
the true statement. A.

RS What is the length of AD in terms of 'b' and


(a)  Rp 2 (b) RP  RQ  RS 2
RQ 'c'?
(c) RS  RQ  RP2 (d) RQ  RS  RP AC bc
(a) (b)
b2  c2 b  c2
2

34. In ABC, A  90 and AD±BC. If AB= 5 bc bc


(c) (d)
cm, BC = a cm and AC = b cm, find the b  c2
2
c2  b2
length of BD in cm.
 b 2  a 2  25   a 2  b 2  25  38. In the given figure, quad. ABCD ~ quad
(a)   (b)  
 a   2a  PQRS. Find the value of 'z'.
 a 2  b 2  25   a 2  b 2  25 
(c)   (d)  
 2b   2a 

35. If ABC is an equilateral triangle of side 'a' JOIN @iitwale in Telegram


and D is a point on BC such that BD = -
BC, what is the length of AD?
7 3 2
(a) a (b) a
3 7 4 45
(a) 5 (b)
7 7 2 6 4
(c) a (d) a
3 3 5 35
(c) 5 (d)
6 3
36. In the given figure, AD -L BC, BC = a, CA
= b, and AB = c. 39. ABC is an isosceles triangle right- angled
at B. Similar triangles ACD and ABE are
constructed on sides AC and AB. Find the
ratio between the areas of ABC and ACD
.
(a) 1 :2 (b) 3 :1
(c)1:3 (d) 4:1

40. If ABC ~ DFE, A  30o , C  50o , AB  5cm,


AC  8cm and DF  7.5 cm . Which of the
1 following is true ?
If BD  CD , what is the value of 2b 2 ?
3 (a) DE  12cm, F  50
(a) a  2c 2
2
(b) DE  12cm, F  100
(b) 2a 2  c 2 (c) EF  12cm, D  100
(c) c 2  2a 2 (d) EF  12cm, D  30
(d) 2a 2  2c2

5
41. In the given figure, AD is the bisector of 46. In ABC,DE | |BC,AD  6cm,BD  9cm and
BAC . If AB= 10 cm, AC = 14cm and BC AE  8cm .
= 6 cm, find the length of DC.

(a) 2 cm (b) 3.5 cm Find the length of AC.


(a) 20 cm (b) 12 cm
(c) 2.5 cm (d) 4 cm
(c) 15 cm (d) 18 cm
AO BO 1
42. In the given figure,   and AB = 47. The lengths of the diagonals of a rhombus
OC OD 2 are 16 cm and 12 cm. What is the length of
5 cm. Find the value of DC. the side of the rhombus?
(a) 9cm (b) 10cm
(c) 8 cm (d) 20 cm

48. In the given figure, two line segments AC


and BD intersect each other at point P, such
(a) 5 cm (b) 15cm that PA = 6 cm, PB = 3 cm, PC = 2.5 cm,
(c) 10 cm (d) 8 cm PD = 5 cm,
JOINAPB  50 and in
@iitwale CDP  30 .
Telegram
Find PBA .
43. In the given figure, AABC is right angled at
A. DEFG is a square, BD = 12 cm and EC
= 27 cm. What is the length of the side of
square?

(a) 70 (b) 80


(c) 90 (d) 100

49. If ABC and PQR are similar and


BC 1 ar (PQR)
(a) 27 cm (b) 19 cm  , find .
QR 3 ar (BCA)
(c) 12 cm (d) 18 cm
(a) 9 (b) 3
44. ABC is right-angled at C. If p is the length 1 1
(c) (d)
of perpendicular from C to AB and AB = c, 3 9
BC = a and CA = b, which of the following
is true? 50. Aman drives 13 km in the north west
(a) pc  ab (b) pb  ab direction, turns left and drives 5 km. How
(c) pc  bc (d) pb  ac far is he from the starting point?
(a) 10km
(b) 18km
45. ABC is right angled at C and AC  3BC . (c) 12 km
What is the value of ABC ? (d) 11 km
(a) 30 (b) 90
(c) 60 (d) 45

6
51. Given xy  4 cm , QR  8 cm and area of
PXY  25 sq.cm , what is the area of PQR ?

(a) 49 cm2 (b) 81 cm2


(c) 64cm2 (d) 100cm2

52. What is the ratio of areas of two similar


triangles whose corresponding sides are in
the ratio 15:19?
(a) 15 : 19 (b) 15:19
(c) 225:361 (d) 125:144

53. Identify the incorrect statement.


(a) A right angled triangle may have 1,1 and
2 as its sides.
(b) 1, 2, 3 are the sides of a right angled
triangle.
(c) The ratio of corresponding sides of two JOIN @iitwale in Telegram
squares whose areas are in the ratio 4:1 is 2
:1.
(d) 17,8 and 15 are the sides of a right
angled triangle.

54. Points P and Q on the sides AB and AC of


ABC are such that PQ||BC , AP: PB = 2 :3
and AQ = 4 cm. What is the measure of
AC?
(a) 12cm (b) 16cm
(c) 10cm (d) 15cm

55. The side of an equilateral triangle is 10 cm.


What is its altitude?
(a) 5 2cm
(b) 5 3cm
(c) 12cm
(d) 5cm

56. The diagonals of a rhombus are 16 cm and


12 cm. What is the measure of its side?
(a) 10cm
(b) 12cm
(c) 8cm
(d) 16cm

7
Answer - Keys

1. A 2. A 3. C 4. B 5. A 6. C

7. A 8. B 9. A 10. B 11. B 12. C

13. D 14. C 15. B 16. D 17. C 18. C

19. B 20. D 21. C 22. B 23. B 24. C

25. C 26. D 27. B 28. D JOIN


29. @iitwale
A in
30.Telegram
D

31. A 32. B 33. C 34. B 35. C 36. D

37. B 38. C 39. A 40. B 41. B 42. C

43. D 44. A 45. C 46. A 47. B 48. D

49. A 50. C 51. D 52. C 53. A 54. C

55. B 56. A

8
Solutions
1. (a) In Perimeter of ABC is 49 cm.
ABC, AC  AB 2  BC 2  10 cm
11. (b) Not available
In ACD, CD  AD2  AC 2
 14cm
 AC  CD  10  14 cm
 2( 5  7)cm Let the starting point of the man be A, AB is
the distance cycled due east, BC is the
2. (a) Not available distance cycled after taking a right turn.
 The required distance is AC, which is
3. (c) Not available given by
152  82  289  17 m
4. (b) Not available

12. (c)
5. (a) Not available

6. (c) Not available

JOIN @iitwale in Telegram


7. (a) If ABC ~ DEF , then Let ' x ' m be the height at which the ladder
ar (ABC ) AP 2 touches the wall. According to Pythagoras'
 where AP and DQ are the
ar (DEF ) DQ 2 theorem,
corresponding altitudes. x2  342 162
Area of the other triangle  16 cm2
 x  342  162  30 m

8. (b) Let the median of the other triangle be


13. (d) Not available
' x ' cm.
Then 81 : 49  (4.9)2 : x2
14. (c) Not available
9
 x   4.9  6.3
7 15. (b) Not available
The other median = 6.3 cm
16. (d) Not available
9. (a) Let AB be ' x ' cm.

Then
32 12
  x  9 cm 17. (c) Not available
24 x
18. (c) Since ABC ~ DEF ,
10. (b) Given BC = 9.1 cm, EF = 6.5 cm and
area of ABC  BC 
2

perimeter of ADEF is 35 cm.  


area of DEF  DE 
Let the perimeter of ABC be ' x ' cm
2
35 6.5 8
   x  49 cm  area of ABC     25  100 cm2
x 9.1 4

9
x  4 3x  19
 
19. (b) In AOD and DOC , 4 x 3
AO BO 1  x  11or 8
  and AB  8 cm
OC OD 4  The required value of ' x ' is 8 units.
AOB  DOC
(Vertically opposite angles) 27. (b) Not available
 AOB ~ DOC . (S.A.S. similarity)
 Corresponding sides are proportional 28. (d) Not available
AO BO AB 1
   
OC OD DC 4 29. (a) Not available
 DC  4 AB
 4  8cm  32cm 30. (d) Not available

20. (d) Given QA  AB and PB  AB , PO = 4 31. (a) Not available


cm, QO = 7 cm and area of
QAO  245 cm . 2

According to the given problem,


APB ~ DPC
JOIN @iitwale in
AP PB AB
Telegram
In QAO and PBO ,
  
DP PC DC
QAO  BPO, AQO  PBO  AP  PC  PB  DP
 QAO ~ PBO (A.A. corollary)
area (QAO) (QO)2 49 32. (b) Not available
  
area(PBO) ( PO)2 16
33. (c) Not available
16
 area PBO  245   80 cm2
49
34. (b) Not available

21. (c) Not available


35. (c) Not available

22. (b) Not available


36. (d) Not available

23. (b) Not available


1 2 2 1
37. (b) b  c  AD  bc
2 2
24. (c) Not available
bc
 AD 
25. (c) Not available b  c2
2

Hence, the length of AD in terms of 'b' and


26. (d) Since the diagonals of a trapezium bc
'c' is
divide each other proportionally, b  c2
2

OA OB

OC OD 38. (c) Not available

10
39. (a) Not available

40. (b) Not available

According to the problem, BC2  BA2  AC2


41. (b) Not available
 A man is 12 km from his starting point.
42. (c) Not available
51. (d) Not available
43. (d) Not available
52. (c) Not available
44. (a) Not available
53. (a) Not available
45. (c) Not available
54. (c) Not available
46. (a) Not available
55. (b) Not available

56. (a) The diagonals of a rhombus bisect each


47. (b) The diagonals of a rhombus are other perpendicularly.
perpendicular to each other JOIN @iitwale in Telegram

 AOB is a right angled triangle with OA


= 8 cm and OB = 6 cm.

side of rhombus  64  36  10cm  AB  OA2  OB2  64  36


 100  10cm
PB AP
48. (d) 
PC DP
 APB ~ DPC
 B  180o  (50o  30o )
 B  100o

49. (a) ABC ~ PQR (Given)


ar (ABC ) BC 2 1
  
ar (PQR) QR 2 9
ar (PQR)
 9
ar (BCA)

50. (c)

11
Introduction to Trigonometry
1. ABC is right angled at A. If AC = 8 cm and 8. ABC is an isosceles triangle with the unequal
AB = 6 cm, what is the value of cosec B? side measuring 12 cm. If both the equal sides
5 3 measure 19 cm, what is the measure of BAC ?
(a) (b)
4 4 (a) 36.8 (b) 68
4 4 (c) 38 (d) 60
(c) (d)
3 5
9. ABCD is a trapezium in which AB = 8 cm, AD
= 4 cm and CD = 3 cm.
2. ABC is right angled at A .lf BC  2 and AB
= AC =1, what is the measure of B ?
(a) 60 (b) 45
(c) 30 (d) 90

3. What is the value of  tor which tan   cot  ?


(a) 60 (b) 45
(c) 90 (d) 0

1 What is the length of BC to the nearest whole


4. Given sin    4 , what is the value number?
sin 
(a) 20 (b) 16 (a) 5 cm (b) 41cm
(c) 14 (d) 4 (c) 8 cm (d) 7 cm

1 10. The figure JOIN


shows an@iitwale in Telegram
isosceles triangle ABC. Find
5. Given sin   , what is the value of
2
the length of the perpendicular from A to BC.
sin 2 
sin   cos ?
25 31
(a) (b)
31 25
24 31
(c) (d)
25 24

6. Find the value of


cos30 cos45  sin30 sin45 .
(a) 5.45 cm (b) 4.55 cm
6 1 (c) 5.6 cm (d) 4.54 cm
(a)
2
6 2 11. If sec  2p and tan  
2
find the value
(b)
4 p
6 2  1 
(c) 2  p2  2  .
8  p 
2( 3  1) 1 1
(d) (a) (b)
4 2 2
1 1
7. What is the value of (c) (d)
tan 7 tan 23 tan 60 tan 67 tan 83 ? 4 8
1
(a) (b) 3 12. What is the value of  if 3 tan 2  3  0 ?
3
(a) 45 (b) 90
(c) 1 (d) 
(c) 30 (d) 60
13. Given sin   cos  3 , what is the value of 19. Graphs of y  sin x and y  cos x , where
tan   cot  ? 
0 x intersect at a point. Find abscissa.
1 2 2
(a) (b)
2 3  
(a) (b)
3 6 4
(c) (d) 1 
2 (c) (d) 0
3
tan 26o  tan19o
14. If  cos60o , what is the
x(1  tan 26 tan19 )
o o
20. Find the value of sin 15 .
value of x ?
3 1 3 1
(a) 1 (b) 2 (a) (b)
2 2
(c) 2 (d) 3
3 1 3 1
(c) (d)
2 2 2 2
2 3 4 x 1 
15. If sin 2  . . ....   1, 0  x  100 , find
1 2 3 x2 21. What is the value of sin0  cos30  tan45 
the value of x . cosec 60  cot90 ?
(a) 91 (b) 80
7 3 6 67 3
(c) 49 (d) 46 (a) (b)
6 6
16. What is the angle between the hour and minute (c) 0 (d) 2
hands of a clock at 02 :15 hours?
1
(a) 15 (b) 7
1o 22. If sin   JOIN
, what@iitwale
are the respective possible
in Telegram
2 2
values of 9 between 0 and 2 ?
1o
(c) 22 (d) 30 (a) 210 and 300 (b) 240 and 330
2 (c) 240 and 300 (d) 210 and 330
17. C is the centre of a circle of radius 3 units, and
23. In terms of radians. what is the equivalent of 45
 is the angle as shown in the given figure. If
?
x2  1 (a) 25 (b) 0.25
sin   cos 2   ,find the value of x.
x2 180o  45o
(c) (d)
45o 

1  cos A
24. Find the value of
1  cos A
(a) sec A  cot A (b) cosec A  cot A
(a) 2 (b) 4 (c) 0 (d) 1
(c) 6 (d) 8
25. Find the value of
 5cos   4 3  5sin 
18. If tan   sec  2 , 0    find the value of 
2 3  5sin  4  5cos 
the tan  . (a) - 1 (b) 5
3 5 (c) 1 (d) 0
(a) (b)
4 4
26. If x  3cos A cos B, y  3cos Asin B and z  3 sin A
3 5
(c) (d)
, find the value of x  y  z .
2 2 2
2 2
(a) 3 (b) 6
(c) 12 (d) 9

3
1  tan 75o 34. If sec  b tan   p , what is the value of cos ?
27. Find the value of
1  tan 75o p2  1 p2 1
(a) (b)
2 P2 1 ( P 2  1) 2
(a)  (b) 3
3 2p 4 p2
(c) 2 (d)
(c)  3 (d)
1 P 1 ( P 2  1) 2
3 35. What is the numerical value of the expression
sin 9o cos81o
 ?
sinA  cosA sin 48o cos 42o
28. If 8 tan A =15, find the value of
sinA  cosA 1
(a) 1 (b)
7 11 2
(a) (b)
23 23 (c) 0 (d) -1
13 17
(c) (d) 36. Find the value of the expression
23 23
[cos ec(75o   )  sec(15o   ) 
sec2  tan(55o   )  cot(35o   )]
29. If 4sin   3cos ,find .
4(1  tan 2  ) (a) - 1 (b) 0
25 25 3
(a) (b) (c) 1 (d)
16 28 2
1 5
(c) (d) 37. If cos(   )  0 , what is the value of
4 6 sin(   ) ?
30. Find the value of (a) cos  (b) cos 2
(c) sin 
JOIN @iitwale in Telegram
(d) sin 2
cos1 cos2cos3.....cos89cos90 .
o o

1
(a) 1 (b) 38. If cos9  sin  a and 9  90o , what is the
2
value of tan 5 a?
1
(c) (d) 0 1
2 (a) (b) 3
3
1
x (c) 1 (d)
31. If tan x  , where x and y are whole numbers, 2
y
find sin x . 39. If ABC is right angled at C, find the value of
y x cos( A  B) .
(a) (b)
y 2  x2 x2  y 2 (a) 0 (b) 1
y x 1 3
(c) (d) (c) (d)
x2  y 2 y 2  x2 2 2

40. If sin A  sin2 A  1 , find the value of the


sin 3   cos3  expression (cos2 A  cos4 A) .
32. Find
sin   cos 
1
(a) 1  sin cos (b) 1  sinc cos (a) 1 (b)
2
(c) 1  sin tan (d) 1 (c) 2 (d) 3

33. If x  a sec  b tan and y  b sec  a tan , 1 1


41. Given that sin  and cos = , find the
find x 2  y 2 . 2 2
(a) 4ab sec  a tan  (b) a  b
2 2 value of (   ) .
(a) 0 (b) 30
(c) b2  a 2 (d) a 2  b2
(c) 60 (d) 90

4
42. Find the value of the expression 50. If 3 tan   3sin  , find the value of
sin 2 22o  sin 2 68o sin   cos2  .
2
 sin 2 63o  cos63o sin 27o
cos 2 22o  cos 2 68o
(a) 3 (b) 2
(c) 1 (d) 0

43. If sin   cos  0 , find the value of


(sin 4   cos4  )
3
(a) 1 (b) 2 1
4 (a) (b)
1 1 3 3
(c) (d)
2 4 1 1
(c) (d)
2 3
44. Find the value of sin(45o   )  cos(45o   ) .
(a) 2cos (b) 0 13
51. If cos ec  , find the value of
(c) 2sin  (d) 1 12
2sin   3cos 
45. What is the value of
4sin   9cos 
sin 2 5  sin 210  sin 2 80  sin 2 85 ?
(a) 0 (b) 1
(c) 2 (d) 3
1
46. If sin B  , find the value of 3cosB  4cos3B .
2
1 JOIN @iitwale in Telegram
(a) (b) 1
2
(c) 2 (d) 0 (a) 0 (b) 1
(c) 3 (d) 2
47. If tan x  sin 45cos 45  sin30 , find the value
of x . 52. Find the value sin 2 30cos2 45  4tan 2 30
(a) 30 o
(b) 60 1 1
 sin 2 90o  2cos 2 90o 
(c) 45 (d) 90 2 24
(a) 1 (b) 2
3 (c) 2 (d) 3
48. If sin(A  B)  1 and cos(A-B)  , find A
2
and B. 2
(a) 45,45 53. Find the value of (cos 4 30o  sin 4 45o )
3
(b) 90,45 1
(c) 45, 30 3(sin 2 60o  sec2 45o )  cot 2 30o
4
(d) 60, 30
15 3
(a) (b)
4 4
4sin   2cos
49. If 4tan   1 , find the value of 65 17
4sin   3cos  (c) 2 (d) 4
4 24
1 1
(a) (b)
2 6 54. If cos3x  cos 30 sin 60  sin 30 cos 60
(c)
2
(d)
1 , find the value of x.
3 3 (a) 60 (b) 45
(c) 20 (d) 30

5
55. In ABC , if B  90,AB  5 cm and AC =10
cm, find A and C 57. If 2(cos  sec )  5 , what is the value of
cos2   sec2  ?
25 5
(a) (b)
2 4
17 4
(c) (d)
4 17

58. How is cot  expressed in terms of sin  ?


(a) 90, 45 (b) 60, 30
(c) 45, 0 (d) 90, 60 1 1  sin 2 
(a) (b)
1  sin 2  sin 
sin   cos 3 1 1 sin 
56. lf  find the acute angle  . (c) (d)
sin   cos 3 1 sin  1  sin 2 
(a) 90 (b) 45
(c) 30 (d) 60

JOIN @iitwale in Telegram

6
Answer - Keys

1. A 2. B 3. B 4. C 5. B 6. D

7. B 8. A 9. B 10. A 11. A 12. C

13. D 14. C 15. D 16. C 17. A 18. A

19. B 20. D 21. A 22. D 23. D 24. C

25. D 26. D 27. C 28. A JOIN


29. @iitwale
B in
30.Telegram
D

31. B 32. A 33. B 34. C 35. C 36. B

37. B 38. C 39. A 40. A 41. D 42. B

43. C 44. B 45. C 46. D 47. C 48. D

49. B 50. B 51. C 52. B 53. D 54. C

55. B 56. D 57. D 58. B

7
Solutions
1. (a) In ABC ,by BD 6
sin       18.41o
Pythagoras' theorem, AB 19
AC  82  62  10 cm  BAC  2(BAD)  2 18.41o
 36.82  36.8

9. (b)

Hypotenuse 5
 cos ec  
sideoppositeto 4

BC  CE 2  EB2
AC 1
2. (b) sin B    16  25  41cm
BC 2

10. (a) The perpendicular from A to BC bisects


BC at D.
 The length of
AD  AB 2  BD2  62 (2.5)2
1 JOIN @iitwale in Telegram
From the tables, sin 45  .  29.75  5.45
2
Therefore sin45  sinB
11. (a) Not available
 B  45

12. (c) Not available


3. (b) Not available

13. (d) Not available


4. (c) Not available

14. (c) Not available


5. (b) Not available

15. (d) Not available


6. (d) Not available

16. (c) The minute hand moves 6 in one


7. (b) Not available
minute.
 It will move 30 in 5 minutes (from 2 to 3).
8. (a) In ABC , AB = AC = 19 cm and BD =
Also the hour hand moves 30 in one hour.
DC = 6 cm.
 30 15 
o

 It will move   in 15 minutes, i.e.,


 60 
So, the angle between the two hands is
o
 1
o
1
 30o   7   22 .
 2 2
In ABD ,

8
17. (a)   30 (The angle subtended in the arc 25. (d) Not available
is half the angle subtended at the centre.)
x2  1 26. (d) x  3cos A cos B, y  3cos Asin B and
sin   cos 2   (Given)
x2 z  3sin A
x 1
2
x2  y 2  z 2  (3cos A cos B)2 
  sin 30o  cos 2 30o
x2 (3cos Asin B)2  (3sin A)2
2
1  3 1 3  9[cos2 A 1  sin 2 A]  9 1  9
     
2  2  2 4
1 5 1 tan 45o  tan 75o
  1   x  2 27. (c)
x 2
4 4 1  tan 45o tan 75o
18. (a) Solving the question from the options,  tan(45o  75o )  tan120o   3
we get
15
28. (a) tan A 
8
sin A  cos A tan A  1

3 5 sin A  cos A tan A  1
tan    sec  15  8 7
4 4  
(Pythagoras theorem) 15  8 23
3 5 8
 tan   sec     2 29. (b) JOIN @iitwale in Telegram
4 4 4
Which satisfies the given equation.


19. (b) When 0    ,
2 4sin   3cos
  1 3 5
sin  cos   tan    sec 
4 4 2 4 4
 5
2
The graphs intersect at .
4 sec 
2  
  4
4(1  tan 2  )   3 2 
20. (d) sin 15  sin(45  30) 4 1    
 4 
 
 sin 45  cos 30  cos 45 sin 30 
25
3 1 3 1 25 25
    16  
2 2 2 2 2 2  9 4(7) 28
4 1  
 16 
21. (a) Not available
30. (d) cos1cos2cos3.....cos45....
22. (d) Not available ... (cos 90  2)cos(90  l)cos 90
coslcos2.....cos45...
23. (d) Not available ...sin2sin lsin0  0

24. (b) Not available


31. (b) Not available

9
 cos2 85  cos2 80  sin 2 80  sin 2 85
32. (a) Not available  11  2

33. (b) Not available 46. (d) Not available

34. (c) Not available 47. (c) Not available

35. (c) Not available 48. (d) Not available

36. (b) cosec(75   )  sec(15   ) 49. (b) We have 4 tan   3


tan(55   )  cot(35   ) sin 
4
 sec(15   )  sec(l5   ) 4sin   2cos  cos
Now, 
4sin   3cos 4 sin   3
cot(35   )  cot  35     0
cos
3 2 1
37. (b) cos(   )  0  
33 6
     90
   90   50. (b) Not available
sin(   )  sin(90     )
o

51. (c) Not available


 sin(90o  2 )  cos 2
JOIN @iitwale in Telegram
52. (b) Not available
38. (c) cos9  sin 
53. (d) Not available
cos(9 )  cos(90   )
 9    90o 54. (c) Not available
 10  90o
   9o 55. (b) In ABC, B  90 AB = 5 cm,
 tan5  tan 45  1 AC = 10 cm
Now, in right angled ABC
39. (a) A  B  C  180 AB 5 1
cos A     cos60o
A  B  90 [ Since C  90 ] AC 10 2
 cos(A  B)  cos 90  0  A  60o
 C  180o  (A  B)
40. (a) Not available  180  (60  90)
 180 150  30
41. (d) Not available

sin   cos 3 1
42. (b) Not available 56. (d) 
sin   cos 3 1
43. (c) Not available (sin   cos )  (sin   cos )

(sin   cos )  (sin   cos )
44. (b) Not available ( 3  1)  ( 3  1)

( 3  1)  ( 3  1)
45. (c) sin 2 5  sin 210  sin 2 80  sin 2 85

10
[Applying Componendo-Dividendo on both
sides]
2sin  2 3
   tan   3
2cos 2
 tan   tan 60o    60o

57. (c) Given 2(cos  sec )  5


5
 cos  sec 
2
2
5
 cos   sec   2cos sec   
2 2

2
25 17
 cos2   sec2   2
4 4
17
 cos 2   sec2  
4

cos cos 2  1  sin 2 


58. (b) cot    
sin  sin  sin 
JOIN @iitwale in Telegram

11
SEARCH { CYBERWORLDFORU.BLOGSPOT.COM } ON GOOGE

ALL STUDY MATERIALS IN FREE OF COST


HERE YOU WILL GET FOLLOWING MATERIALS :-
 HANDWRITTEN NOTES
 PRINTED NOTES { DETAILED , REVISION , SHORT }
 IMPORTANT QUESTIONS
 BOOKS PDF
 BOOKS SOLUTIONS
 TEST PAPERS
 CHAPTERWISH PREVIOUS YEAR PAPERS
 SAMPLE PAPERS
 CONCEPT MAPS
 HOT QUESTIONS
 NEWS AND UPDATES JOIN @iitwale in Telegram

MATERIALS ARE AVILABLE FOR FOLLOWING :-


 CLASS 9TH
 CLASS 10TH
 CLASS 11TH
 CLASS 12TH
 IIT-JEE
 NEET
 NTSE
 KVPY
 OTHER COMPETITIVE EXAMS

You might also like